Slot System
Featured Buckets
Featured Buckets Admin
Reverse Chronological Sort
Allow Teaser Image

Pustular Eruption on the Face

Article Type
Changed
Wed, 01/31/2024 - 11:08
Display Headline
Pustular Eruption on the Face

The Diagnosis: Eczema Herpeticum

The patient’s condition with worsening facial edema and notable pain prompted a bedside Tzanck smear using a sample from the base of a deroofed forehead vesicle. In addition, a swab of a deroofed lesion was sent for herpes simplex virus and varicella-zoster virus (VZV) polymerase chain reaction (PCR) testing. The Tzanck smear demonstrated ballooning multinucleated syncytial giant cells and eosinophilic inclusion bodies (Figure), which are characteristic of certain herpesviruses including herpes simplex virus and VZV. He was started on intravenous acyclovir while PCR results were pending; the PCR test later confirmed positivity for herpes simplex virus type 1. Treatment was transitioned to oral valacyclovir once the lesions started crusting over. Notable healing and epithelialization of the lesions occurred during his hospital stay, and he was discharged home 5 days after starting treatment. He was counseled on autoinoculation, advised that he was considered infectious until all lesions had crusted over, and encouraged to employ frequent handwashing. Complete resolution of eczema herpeticum (EH) was noted at 3-week follow-up.

Obinwanne.jpg
%3Cp%3EA%20Tzanck%20smear%20of%20a%20forehead%20vesicle%20revealed%20multinucleated%20giant%20cells%20and%20eosinophilic%20inclusion%20bodies%20(original%20magnification%20%C3%9740).%3C%2Fp%3E

Eczema herpeticum (also known as Kaposi varicelliform eruption) is a potentially life-threatening disseminated cutaneous infection caused by herpes simplex virus types 1 and 2 in patients with pre-existing skin disease.1 It typically presents as a complication of atopic dermatitis (AD) but also has been identified as a rare complication in other conditions that disrupt the normal skin barrier, including mycosis fungoides, pemphigus foliaceus, pemphigus vulgaris, Darier disease, pityriasis rubra pilaris, contact dermatitis, and seborrheic dermatitis.1-4

The pathogenesis of EH is multifactorial. Disruption of the stratum corneum; impaired natural killer cell function; early-onset, untreated, or severe AD; disrupted skin microbiota with skewed colonization by Staphylococcus aureus; immunosuppressive AD therapies such as calcineurin inhibitors; eosinophilia; and helper T cell (TH2) cytokine predominance all have been suggested to play a role in the development of EH.5-8

As seen in our patient, EH presents with a sudden eruption of painful or pruritic, grouped, monomorphic, domeshaped vesicles with background swelling and erythema typically on the head, neck, and trunk. Vesicles then progress to punched-out erosions with overlying hemorrhagic crusting that can coalesce to form large denuded areas susceptible to superinfection with bacteria.9 Other accompanying symptoms include high fever, chills, malaise, and lymphadenopathy. Associated inflammation, classically described as erythema, may be difficult to discern in patients with darker skin and appears as hyperpigmentation; therefore, identification of clusters of monomorphic vesicles in areas of pre-existing dermatitis is particularly important for clinical diagnosis in people with darker skin types.

Various tests are available to confirm diagnosis in ambiguous cases. Bedside Tzanck smears can be performed rapidly and are considered positive if characteristic multinucleated giant cells are noted; however, they do not differentiate between the various herpesviruses. Direct fluorescent antibody testing of scraped lesions and viral cultures of swabbed vesicular fluid are equally effective in distinguishing between herpes simplex virus type 1, herpes simplex virus type 2, and VZV; PCR confirms the diagnosis with high specificity and sensitivity.10

In our patient, the initial differential diagnosis included EH, acute generalized exanthematous pustulosis, allergic contact dermatitis, and Orthopoxvirus infection. The positive Tzanck smear reduced the likelihood of a nonviral etiology. Additionally, worsening of the rash despite discontinuation of medications and utilization of topical steroids argued against acute generalized exanthematous pustulosis and allergic contact dermatitis. The laboratory findings reduced the likelihood of drug reaction with eosinophilia and systemic symptoms (DRESS) syndrome, and PCR findings ultimately ruled out Orthopoxvirus infections. Additional differential diagnoses for EH include dermatitis herpetiformis; primary VZV infection; hand, foot, and mouth disease; disseminated zoster infection; disseminated molluscum contagiosum; and eczema coxsackium.

Complications of EH include scarring; herpetic keratitis due to corneal infection, which if left untreated can progress to blindness; and rarely death due to multiorgan failure or septicemia.11 The traditional smallpox vaccine (ACAM2000) is contraindicated in patients with AD and EH, even when AD is in remission. These patients should avoid contact with recently vaccinated individuals.12 An alternative vaccine—Jynneos (Bavarian Nordic)—is available for these patients and their family members.13 Clinicians should be aware of this guideline, especially given the recent mpox (monkeypox) outbreaks.

Mild cases of EH are more common, may sometimes go unnoticed, and self-resolve in healthy patients. Severe cases may require systemic antiviral therapy. Acyclovir and its prodrug valacyclovir are standard treatments for EH. Alternatively, foscarnet or cidofovir can be used in the treatment of acyclovir-resistant thymidine kinase– deficient herpes simplex virus and other acyclovirresistant cases.14 Any secondary bacterial superinfections, usually due to staphylococcal or streptococcal bacteria, should be treated with antibiotics. A thorough ophthalmologic evaluation should be performed for patients with periocular involvement of EH. Empiric treatment should be started immediately, given a relative low toxicity of systemic antiviral therapy and high morbidity and mortality associated with untreated widespread EH.

It is important to maintain a high index of clinical suspicion for EH, especially in patients with pre-existing conditions such as AD who present with systemic symptoms and facial vesicles, pustules, or erosions to ensure prompt diagnosis and appropriate treatment.

References
  1. Baaniya B, Agrawal S. Kaposi varicelliform eruption in a patient with pemphigus vulgaris: a case report and review of the literature. Case Rep Dermatol Med. 2020;2020:6695342. doi:10.1155/2020/6695342
  2. Tayabali K, Pothiwalla H, Lowitt M. Eczema herpeticum in Darier’s disease: a topical storm. J Community Hosp Intern Med Perspect. 2019;9:347. doi:10.1080/20009666.2019.1650590
  3. Cavalié M, Giacchero D, Cardot-Leccia N, et al. Kaposi’s varicelliform eruption in a patient with pityriasis rubra pilaris (pityriasis rubra pilaris herpeticum). J Eur Acad Dermatol Venereol. 2013;27:1585-1586. doi:10.1111/JDV.12120
  4. Lee GH, Kim YM, Lee SY, et al. A case of eczema herpeticum with Hailey-Hailey disease. Ann Dermatol. 2009;21:311-314. doi:10.5021/ad.2009.21.3.311
  5. Seegräber M, Worm M, Werfel T, et al. Recurrent eczema herpeticum— a retrospective European multicenter study evaluating the clinical characteristics of eczema herpeticum cases in atopic dermatitis patients. J Eur Acad Dermatol Venereol. 2020;34:1074-1079. doi:10.1111/JDV.16090
  6. Kawakami Y, Ando T, Lee J-R, et al. Defective natural killer cell activity in a mouse model of eczema herpeticum. J Allergy Clin Immunol. 2017;139:997-1006.e10. doi:10.1016/j.jaci.2016.06.034
  7. Beck L, Latchney L, Zaccaro D, et al. Biomarkers of disease severity and Th2 polarity are predictors of risk for eczema herpeticum. J Allergy Clin Immunol. 2008;121:S37-S37. doi:10.1016/j.jaci.2007.12.152
  8. Kim M, Jung M, Hong SP, et al. Topical calcineurin inhibitors compromise stratum corneum integrity, epidermal permeability and antimicrobial barrier function. Exp Dermatol. 2010; 19:501-510. doi:10.1111/J.1600-0625.2009.00941.X
  9. Karray M, Kwan E, Souissi A. Kaposi varicelliform eruption. StatPearls [Internet]. StatPearls Publishing; 2023. https://www.ncbi.nlm.nih.gov/books/NBK482432/
  10. Dominguez SR, Pretty K, Hengartner R, et al. Comparison of herpes simplex virus PCR with culture for virus detection in multisource surface swab specimens from neonates [published online September 25, 2018]. J Clin Microbiol. doi:10.1128/JCM.00632-18
  11. Feye F, De Halleux C, Gillet JB, et al. Exacerbation of atopic dermatitis in the emergency department. Eur J Emerg Med. 2004;11:49-52. doi:10.1097/00063110-200412000-00014
  12. Casey C, Vellozzi C, Mootrey GT, et al; Vaccinia Case Definition Development Working Group; Advisory Committee on Immunization Practices-Armed Forces Epidemiological Board Smallpox Vaccine Safety Working Group. Surveillance guidelines for smallpox vaccine (vaccinia) adverse reactions. MMWR Recomm Rep. 2006;55:1-16.
  13. Rao AK, Petersen BW, Whitehill F, et al. Use of JYNNEOS (Smallpox and Monkeypox Vaccine, Live, Nonreplicating) for preexposure vaccination of persons at risk for occupational exposure to orthopoxviruses: recommendations of the Advisory Committee on Immunization Practices—United States, 2022. MMWR Morb Mortal Wkly Rep. 2022;71:734-742. doi:10.15585 /MMWR.MM7122E1
  14. Piret J, Boivin G. Resistance of herpes simplex viruses to nucleoside analogues: mechanisms, prevalence, and management. Antimicrob Agents Chemother. 2011;55:459. doi:10.1128/AAC.00615-10
Article PDF
Author and Disclosure Information

Vera Obinwanne is from the Western Michigan Homer Stryker School of Medicine, Kalamazoo. Drs. Tung and James are from the Department of Dermatology, University of Pittsburgh Medical Center, Pennsylvania.

The authors report no conflict of interest.

Correspondence: Alaina J. James, MD, PhD, University of Pittsburgh Medical Center, Medical Arts Bldg, 3708 Fifth Ave, Pittsburgh, PA 15213 (alainajjames@gmail.com).

Issue
Cutis - 112(5)
Publications
Topics
Page Number
E10-E12
Sections
Author and Disclosure Information

Vera Obinwanne is from the Western Michigan Homer Stryker School of Medicine, Kalamazoo. Drs. Tung and James are from the Department of Dermatology, University of Pittsburgh Medical Center, Pennsylvania.

The authors report no conflict of interest.

Correspondence: Alaina J. James, MD, PhD, University of Pittsburgh Medical Center, Medical Arts Bldg, 3708 Fifth Ave, Pittsburgh, PA 15213 (alainajjames@gmail.com).

Author and Disclosure Information

Vera Obinwanne is from the Western Michigan Homer Stryker School of Medicine, Kalamazoo. Drs. Tung and James are from the Department of Dermatology, University of Pittsburgh Medical Center, Pennsylvania.

The authors report no conflict of interest.

Correspondence: Alaina J. James, MD, PhD, University of Pittsburgh Medical Center, Medical Arts Bldg, 3708 Fifth Ave, Pittsburgh, PA 15213 (alainajjames@gmail.com).

Article PDF
Article PDF
Related Articles

The Diagnosis: Eczema Herpeticum

The patient’s condition with worsening facial edema and notable pain prompted a bedside Tzanck smear using a sample from the base of a deroofed forehead vesicle. In addition, a swab of a deroofed lesion was sent for herpes simplex virus and varicella-zoster virus (VZV) polymerase chain reaction (PCR) testing. The Tzanck smear demonstrated ballooning multinucleated syncytial giant cells and eosinophilic inclusion bodies (Figure), which are characteristic of certain herpesviruses including herpes simplex virus and VZV. He was started on intravenous acyclovir while PCR results were pending; the PCR test later confirmed positivity for herpes simplex virus type 1. Treatment was transitioned to oral valacyclovir once the lesions started crusting over. Notable healing and epithelialization of the lesions occurred during his hospital stay, and he was discharged home 5 days after starting treatment. He was counseled on autoinoculation, advised that he was considered infectious until all lesions had crusted over, and encouraged to employ frequent handwashing. Complete resolution of eczema herpeticum (EH) was noted at 3-week follow-up.

Obinwanne.jpg
%3Cp%3EA%20Tzanck%20smear%20of%20a%20forehead%20vesicle%20revealed%20multinucleated%20giant%20cells%20and%20eosinophilic%20inclusion%20bodies%20(original%20magnification%20%C3%9740).%3C%2Fp%3E

Eczema herpeticum (also known as Kaposi varicelliform eruption) is a potentially life-threatening disseminated cutaneous infection caused by herpes simplex virus types 1 and 2 in patients with pre-existing skin disease.1 It typically presents as a complication of atopic dermatitis (AD) but also has been identified as a rare complication in other conditions that disrupt the normal skin barrier, including mycosis fungoides, pemphigus foliaceus, pemphigus vulgaris, Darier disease, pityriasis rubra pilaris, contact dermatitis, and seborrheic dermatitis.1-4

The pathogenesis of EH is multifactorial. Disruption of the stratum corneum; impaired natural killer cell function; early-onset, untreated, or severe AD; disrupted skin microbiota with skewed colonization by Staphylococcus aureus; immunosuppressive AD therapies such as calcineurin inhibitors; eosinophilia; and helper T cell (TH2) cytokine predominance all have been suggested to play a role in the development of EH.5-8

As seen in our patient, EH presents with a sudden eruption of painful or pruritic, grouped, monomorphic, domeshaped vesicles with background swelling and erythema typically on the head, neck, and trunk. Vesicles then progress to punched-out erosions with overlying hemorrhagic crusting that can coalesce to form large denuded areas susceptible to superinfection with bacteria.9 Other accompanying symptoms include high fever, chills, malaise, and lymphadenopathy. Associated inflammation, classically described as erythema, may be difficult to discern in patients with darker skin and appears as hyperpigmentation; therefore, identification of clusters of monomorphic vesicles in areas of pre-existing dermatitis is particularly important for clinical diagnosis in people with darker skin types.

Various tests are available to confirm diagnosis in ambiguous cases. Bedside Tzanck smears can be performed rapidly and are considered positive if characteristic multinucleated giant cells are noted; however, they do not differentiate between the various herpesviruses. Direct fluorescent antibody testing of scraped lesions and viral cultures of swabbed vesicular fluid are equally effective in distinguishing between herpes simplex virus type 1, herpes simplex virus type 2, and VZV; PCR confirms the diagnosis with high specificity and sensitivity.10

In our patient, the initial differential diagnosis included EH, acute generalized exanthematous pustulosis, allergic contact dermatitis, and Orthopoxvirus infection. The positive Tzanck smear reduced the likelihood of a nonviral etiology. Additionally, worsening of the rash despite discontinuation of medications and utilization of topical steroids argued against acute generalized exanthematous pustulosis and allergic contact dermatitis. The laboratory findings reduced the likelihood of drug reaction with eosinophilia and systemic symptoms (DRESS) syndrome, and PCR findings ultimately ruled out Orthopoxvirus infections. Additional differential diagnoses for EH include dermatitis herpetiformis; primary VZV infection; hand, foot, and mouth disease; disseminated zoster infection; disseminated molluscum contagiosum; and eczema coxsackium.

Complications of EH include scarring; herpetic keratitis due to corneal infection, which if left untreated can progress to blindness; and rarely death due to multiorgan failure or septicemia.11 The traditional smallpox vaccine (ACAM2000) is contraindicated in patients with AD and EH, even when AD is in remission. These patients should avoid contact with recently vaccinated individuals.12 An alternative vaccine—Jynneos (Bavarian Nordic)—is available for these patients and their family members.13 Clinicians should be aware of this guideline, especially given the recent mpox (monkeypox) outbreaks.

Mild cases of EH are more common, may sometimes go unnoticed, and self-resolve in healthy patients. Severe cases may require systemic antiviral therapy. Acyclovir and its prodrug valacyclovir are standard treatments for EH. Alternatively, foscarnet or cidofovir can be used in the treatment of acyclovir-resistant thymidine kinase– deficient herpes simplex virus and other acyclovirresistant cases.14 Any secondary bacterial superinfections, usually due to staphylococcal or streptococcal bacteria, should be treated with antibiotics. A thorough ophthalmologic evaluation should be performed for patients with periocular involvement of EH. Empiric treatment should be started immediately, given a relative low toxicity of systemic antiviral therapy and high morbidity and mortality associated with untreated widespread EH.

It is important to maintain a high index of clinical suspicion for EH, especially in patients with pre-existing conditions such as AD who present with systemic symptoms and facial vesicles, pustules, or erosions to ensure prompt diagnosis and appropriate treatment.

The Diagnosis: Eczema Herpeticum

The patient’s condition with worsening facial edema and notable pain prompted a bedside Tzanck smear using a sample from the base of a deroofed forehead vesicle. In addition, a swab of a deroofed lesion was sent for herpes simplex virus and varicella-zoster virus (VZV) polymerase chain reaction (PCR) testing. The Tzanck smear demonstrated ballooning multinucleated syncytial giant cells and eosinophilic inclusion bodies (Figure), which are characteristic of certain herpesviruses including herpes simplex virus and VZV. He was started on intravenous acyclovir while PCR results were pending; the PCR test later confirmed positivity for herpes simplex virus type 1. Treatment was transitioned to oral valacyclovir once the lesions started crusting over. Notable healing and epithelialization of the lesions occurred during his hospital stay, and he was discharged home 5 days after starting treatment. He was counseled on autoinoculation, advised that he was considered infectious until all lesions had crusted over, and encouraged to employ frequent handwashing. Complete resolution of eczema herpeticum (EH) was noted at 3-week follow-up.

Obinwanne.jpg
%3Cp%3EA%20Tzanck%20smear%20of%20a%20forehead%20vesicle%20revealed%20multinucleated%20giant%20cells%20and%20eosinophilic%20inclusion%20bodies%20(original%20magnification%20%C3%9740).%3C%2Fp%3E

Eczema herpeticum (also known as Kaposi varicelliform eruption) is a potentially life-threatening disseminated cutaneous infection caused by herpes simplex virus types 1 and 2 in patients with pre-existing skin disease.1 It typically presents as a complication of atopic dermatitis (AD) but also has been identified as a rare complication in other conditions that disrupt the normal skin barrier, including mycosis fungoides, pemphigus foliaceus, pemphigus vulgaris, Darier disease, pityriasis rubra pilaris, contact dermatitis, and seborrheic dermatitis.1-4

The pathogenesis of EH is multifactorial. Disruption of the stratum corneum; impaired natural killer cell function; early-onset, untreated, or severe AD; disrupted skin microbiota with skewed colonization by Staphylococcus aureus; immunosuppressive AD therapies such as calcineurin inhibitors; eosinophilia; and helper T cell (TH2) cytokine predominance all have been suggested to play a role in the development of EH.5-8

As seen in our patient, EH presents with a sudden eruption of painful or pruritic, grouped, monomorphic, domeshaped vesicles with background swelling and erythema typically on the head, neck, and trunk. Vesicles then progress to punched-out erosions with overlying hemorrhagic crusting that can coalesce to form large denuded areas susceptible to superinfection with bacteria.9 Other accompanying symptoms include high fever, chills, malaise, and lymphadenopathy. Associated inflammation, classically described as erythema, may be difficult to discern in patients with darker skin and appears as hyperpigmentation; therefore, identification of clusters of monomorphic vesicles in areas of pre-existing dermatitis is particularly important for clinical diagnosis in people with darker skin types.

Various tests are available to confirm diagnosis in ambiguous cases. Bedside Tzanck smears can be performed rapidly and are considered positive if characteristic multinucleated giant cells are noted; however, they do not differentiate between the various herpesviruses. Direct fluorescent antibody testing of scraped lesions and viral cultures of swabbed vesicular fluid are equally effective in distinguishing between herpes simplex virus type 1, herpes simplex virus type 2, and VZV; PCR confirms the diagnosis with high specificity and sensitivity.10

In our patient, the initial differential diagnosis included EH, acute generalized exanthematous pustulosis, allergic contact dermatitis, and Orthopoxvirus infection. The positive Tzanck smear reduced the likelihood of a nonviral etiology. Additionally, worsening of the rash despite discontinuation of medications and utilization of topical steroids argued against acute generalized exanthematous pustulosis and allergic contact dermatitis. The laboratory findings reduced the likelihood of drug reaction with eosinophilia and systemic symptoms (DRESS) syndrome, and PCR findings ultimately ruled out Orthopoxvirus infections. Additional differential diagnoses for EH include dermatitis herpetiformis; primary VZV infection; hand, foot, and mouth disease; disseminated zoster infection; disseminated molluscum contagiosum; and eczema coxsackium.

Complications of EH include scarring; herpetic keratitis due to corneal infection, which if left untreated can progress to blindness; and rarely death due to multiorgan failure or septicemia.11 The traditional smallpox vaccine (ACAM2000) is contraindicated in patients with AD and EH, even when AD is in remission. These patients should avoid contact with recently vaccinated individuals.12 An alternative vaccine—Jynneos (Bavarian Nordic)—is available for these patients and their family members.13 Clinicians should be aware of this guideline, especially given the recent mpox (monkeypox) outbreaks.

Mild cases of EH are more common, may sometimes go unnoticed, and self-resolve in healthy patients. Severe cases may require systemic antiviral therapy. Acyclovir and its prodrug valacyclovir are standard treatments for EH. Alternatively, foscarnet or cidofovir can be used in the treatment of acyclovir-resistant thymidine kinase– deficient herpes simplex virus and other acyclovirresistant cases.14 Any secondary bacterial superinfections, usually due to staphylococcal or streptococcal bacteria, should be treated with antibiotics. A thorough ophthalmologic evaluation should be performed for patients with periocular involvement of EH. Empiric treatment should be started immediately, given a relative low toxicity of systemic antiviral therapy and high morbidity and mortality associated with untreated widespread EH.

It is important to maintain a high index of clinical suspicion for EH, especially in patients with pre-existing conditions such as AD who present with systemic symptoms and facial vesicles, pustules, or erosions to ensure prompt diagnosis and appropriate treatment.

References
  1. Baaniya B, Agrawal S. Kaposi varicelliform eruption in a patient with pemphigus vulgaris: a case report and review of the literature. Case Rep Dermatol Med. 2020;2020:6695342. doi:10.1155/2020/6695342
  2. Tayabali K, Pothiwalla H, Lowitt M. Eczema herpeticum in Darier’s disease: a topical storm. J Community Hosp Intern Med Perspect. 2019;9:347. doi:10.1080/20009666.2019.1650590
  3. Cavalié M, Giacchero D, Cardot-Leccia N, et al. Kaposi’s varicelliform eruption in a patient with pityriasis rubra pilaris (pityriasis rubra pilaris herpeticum). J Eur Acad Dermatol Venereol. 2013;27:1585-1586. doi:10.1111/JDV.12120
  4. Lee GH, Kim YM, Lee SY, et al. A case of eczema herpeticum with Hailey-Hailey disease. Ann Dermatol. 2009;21:311-314. doi:10.5021/ad.2009.21.3.311
  5. Seegräber M, Worm M, Werfel T, et al. Recurrent eczema herpeticum— a retrospective European multicenter study evaluating the clinical characteristics of eczema herpeticum cases in atopic dermatitis patients. J Eur Acad Dermatol Venereol. 2020;34:1074-1079. doi:10.1111/JDV.16090
  6. Kawakami Y, Ando T, Lee J-R, et al. Defective natural killer cell activity in a mouse model of eczema herpeticum. J Allergy Clin Immunol. 2017;139:997-1006.e10. doi:10.1016/j.jaci.2016.06.034
  7. Beck L, Latchney L, Zaccaro D, et al. Biomarkers of disease severity and Th2 polarity are predictors of risk for eczema herpeticum. J Allergy Clin Immunol. 2008;121:S37-S37. doi:10.1016/j.jaci.2007.12.152
  8. Kim M, Jung M, Hong SP, et al. Topical calcineurin inhibitors compromise stratum corneum integrity, epidermal permeability and antimicrobial barrier function. Exp Dermatol. 2010; 19:501-510. doi:10.1111/J.1600-0625.2009.00941.X
  9. Karray M, Kwan E, Souissi A. Kaposi varicelliform eruption. StatPearls [Internet]. StatPearls Publishing; 2023. https://www.ncbi.nlm.nih.gov/books/NBK482432/
  10. Dominguez SR, Pretty K, Hengartner R, et al. Comparison of herpes simplex virus PCR with culture for virus detection in multisource surface swab specimens from neonates [published online September 25, 2018]. J Clin Microbiol. doi:10.1128/JCM.00632-18
  11. Feye F, De Halleux C, Gillet JB, et al. Exacerbation of atopic dermatitis in the emergency department. Eur J Emerg Med. 2004;11:49-52. doi:10.1097/00063110-200412000-00014
  12. Casey C, Vellozzi C, Mootrey GT, et al; Vaccinia Case Definition Development Working Group; Advisory Committee on Immunization Practices-Armed Forces Epidemiological Board Smallpox Vaccine Safety Working Group. Surveillance guidelines for smallpox vaccine (vaccinia) adverse reactions. MMWR Recomm Rep. 2006;55:1-16.
  13. Rao AK, Petersen BW, Whitehill F, et al. Use of JYNNEOS (Smallpox and Monkeypox Vaccine, Live, Nonreplicating) for preexposure vaccination of persons at risk for occupational exposure to orthopoxviruses: recommendations of the Advisory Committee on Immunization Practices—United States, 2022. MMWR Morb Mortal Wkly Rep. 2022;71:734-742. doi:10.15585 /MMWR.MM7122E1
  14. Piret J, Boivin G. Resistance of herpes simplex viruses to nucleoside analogues: mechanisms, prevalence, and management. Antimicrob Agents Chemother. 2011;55:459. doi:10.1128/AAC.00615-10
References
  1. Baaniya B, Agrawal S. Kaposi varicelliform eruption in a patient with pemphigus vulgaris: a case report and review of the literature. Case Rep Dermatol Med. 2020;2020:6695342. doi:10.1155/2020/6695342
  2. Tayabali K, Pothiwalla H, Lowitt M. Eczema herpeticum in Darier’s disease: a topical storm. J Community Hosp Intern Med Perspect. 2019;9:347. doi:10.1080/20009666.2019.1650590
  3. Cavalié M, Giacchero D, Cardot-Leccia N, et al. Kaposi’s varicelliform eruption in a patient with pityriasis rubra pilaris (pityriasis rubra pilaris herpeticum). J Eur Acad Dermatol Venereol. 2013;27:1585-1586. doi:10.1111/JDV.12120
  4. Lee GH, Kim YM, Lee SY, et al. A case of eczema herpeticum with Hailey-Hailey disease. Ann Dermatol. 2009;21:311-314. doi:10.5021/ad.2009.21.3.311
  5. Seegräber M, Worm M, Werfel T, et al. Recurrent eczema herpeticum— a retrospective European multicenter study evaluating the clinical characteristics of eczema herpeticum cases in atopic dermatitis patients. J Eur Acad Dermatol Venereol. 2020;34:1074-1079. doi:10.1111/JDV.16090
  6. Kawakami Y, Ando T, Lee J-R, et al. Defective natural killer cell activity in a mouse model of eczema herpeticum. J Allergy Clin Immunol. 2017;139:997-1006.e10. doi:10.1016/j.jaci.2016.06.034
  7. Beck L, Latchney L, Zaccaro D, et al. Biomarkers of disease severity and Th2 polarity are predictors of risk for eczema herpeticum. J Allergy Clin Immunol. 2008;121:S37-S37. doi:10.1016/j.jaci.2007.12.152
  8. Kim M, Jung M, Hong SP, et al. Topical calcineurin inhibitors compromise stratum corneum integrity, epidermal permeability and antimicrobial barrier function. Exp Dermatol. 2010; 19:501-510. doi:10.1111/J.1600-0625.2009.00941.X
  9. Karray M, Kwan E, Souissi A. Kaposi varicelliform eruption. StatPearls [Internet]. StatPearls Publishing; 2023. https://www.ncbi.nlm.nih.gov/books/NBK482432/
  10. Dominguez SR, Pretty K, Hengartner R, et al. Comparison of herpes simplex virus PCR with culture for virus detection in multisource surface swab specimens from neonates [published online September 25, 2018]. J Clin Microbiol. doi:10.1128/JCM.00632-18
  11. Feye F, De Halleux C, Gillet JB, et al. Exacerbation of atopic dermatitis in the emergency department. Eur J Emerg Med. 2004;11:49-52. doi:10.1097/00063110-200412000-00014
  12. Casey C, Vellozzi C, Mootrey GT, et al; Vaccinia Case Definition Development Working Group; Advisory Committee on Immunization Practices-Armed Forces Epidemiological Board Smallpox Vaccine Safety Working Group. Surveillance guidelines for smallpox vaccine (vaccinia) adverse reactions. MMWR Recomm Rep. 2006;55:1-16.
  13. Rao AK, Petersen BW, Whitehill F, et al. Use of JYNNEOS (Smallpox and Monkeypox Vaccine, Live, Nonreplicating) for preexposure vaccination of persons at risk for occupational exposure to orthopoxviruses: recommendations of the Advisory Committee on Immunization Practices—United States, 2022. MMWR Morb Mortal Wkly Rep. 2022;71:734-742. doi:10.15585 /MMWR.MM7122E1
  14. Piret J, Boivin G. Resistance of herpes simplex viruses to nucleoside analogues: mechanisms, prevalence, and management. Antimicrob Agents Chemother. 2011;55:459. doi:10.1128/AAC.00615-10
Issue
Cutis - 112(5)
Issue
Cutis - 112(5)
Page Number
E10-E12
Page Number
E10-E12
Publications
Publications
Topics
Article Type
Display Headline
Pustular Eruption on the Face
Display Headline
Pustular Eruption on the Face
Sections
Questionnaire Body

A 52-year-old man developed a sudden eruption of small pustules on background erythema and edema covering the forehead, nasal bridge, periorbital region, cheeks, and perioral region on day 3 of hospitalization in the intensive care unit for management of septic shock secondary to a complicated urinary tract infection. He had a medical history of benign prostatic hyperplasia, sarcoidosis, and atopic dermatitis. He initially presented to the emergency department with fever, chills, and dysuria of 2 days’ duration. Because he received ceftriaxone, vancomycin, ciprofloxacin, and tamsulosin while hospitalized for the infection, the primary medical team suspected a drug reaction and empirically started applying hydrocortisone cream 2.5%. The rash continued to spread over the ensuing day, prompting a dermatology consultation to rule out a drug eruption and to help guide further management. The patient was in substantial distress and pain. Physical examination revealed numerous discrete and confluent monomorphic pustules on background erythema with faint collarettes of scale covering most of the face. Substantial periorbital and facial edema forced the eyes closed. There was no mucous membrane involvement. A review of systems was negative for dyspnea and dysphagia, and the rash was not present elsewhere on the body. Ophthalmologic evaluation revealed no ocular involvement or vision changes. Laboratory studies demonstrated neutrophilia (17.27×109 cells/L [reference range, 2.0–6.9×109 cells/L]). The eosinophil count, blood urea nitrogen/creatinine, and liver function tests were within reference range.

Obinwanne_Quiz.jpg

Disallow All Ads
Content Gating
No Gating (article Unlocked/Free)
Alternative CME
Disqus Comments
Default
Gate On Date
Wed, 11/08/2023 - 14:15
Un-Gate On Date
Wed, 11/08/2023 - 14:15
Use ProPublica
CFC Schedule Remove Status
Wed, 11/08/2023 - 14:15
Hide sidebar & use full width
render the right sidebar.
Conference Recap Checkbox
Not Conference Recap
Clinical Edge
Display the Slideshow in this Article
Medscape Article
Display survey writer
Reuters content
Disable Inline Native ads
WebMD Article
Article PDF Media
Image
Teambase ID
18002628.SIG
Disable zoom
Off

Clustered Vesicles on the Neck

Article Type
Changed
Wed, 04/10/2024 - 09:17
Display Headline
Clustered Vesicles on the Neck

The Diagnosis: Microcystic Lymphatic Malformation

A punch biopsy demonstrated anastomosing fluidfilled spaces within the papillary and reticular dermal layers (Figure), confirming the diagnosis of microcystic lymphatic malformation (LM)(formerly known as lymphangioma circumscriptum), a congenital vascular malformation composed of slow-flow lymphatic channels.1 The patient underwent serial excisions with improvement of the LM, though the treatment course was complicated by hypertrophic scar formation.

Kayishunge.jpg
%3Cp%3EAn%20unencapsulated%20proliferation%20of%20anastomosing%20vascular%20spaces%20within%20the%20papillary%20and%20reticular%20dermis%20(H%26amp%3BE%2C%20original%20magnification%20%C3%9720).%3C%2Fp%3E

The classic clinical presentation of microcystic LM includes a crop of vesicles containing clear or hemorrhagic fluid with associated oozing or bleeding.2 When cutaneous lesions resembling microcystic LM develop in response to lymphatic damage and resulting stasis, such as from prior radiotherapy or surgery, the term lymphangiectasia is used to distinguish this entity from congenital microcystic LM.3

Microcystic LMs are histologically indistinguishable from macrocystic LMs; however, macrocystic LMs typically are clinically evident at birth as ill-defined subcutaneous masses.2,4-6 Dermatitis herpetiformis, a dermatologic manifestation of gluten sensitivity, causes intensely pruritic vesicles in a symmetric distribution on the elbows, knees, and buttocks. Histopathology shows neutrophilic microabscesses in the dermal papillae with subepidermal blistering. Direct immunofluorescence demonstrates the deposition of IgA along the basement membrane with dermal papillae aggregates.6 The underlying dermis also may contain a lymphohistiocytic infiltrate rich in neutrophils. The vesicles of herpes zoster virus are painful and present in a dermatomal distribution. A viral cytopathic effect often is observed in keratinocytes, specifically with multinucleation, molding, and margination of chromatin material. The lesions are accompanied by variable lymphocytic inflammation and epithelial necrosis resulting in intraepidermal blistering.7 Extragenital lichen sclerosus presents as polygonal white papules merging to form plaques and may include hemorrhagic blisters in some instances. Histopathology shows hyperkeratosis, epidermal atrophy with flattened rete ridges, vacuolar interface changes, loss of elastic fibers, and hyalinization of the lamina propria with lymphocytic infiltrate.8

Endothelial cells in LM exhibit activating mutations in the phosphatidylinositol-4,5-bisphosphate 3-kinase catalytic subunit alpha gene, PIK3CA, which may lead to proliferation and overgrowth of the lymphatic vasculature, as well as increased production of cyclic guanosine monophosphate.9,10 Phosphodiesterase 5 (PDE5) is expressed in the perivascular smooth muscle adjacent to lymphatic spaces in LMs but not in the their vasculature. 10 This pattern of PDE5 expression may cause perilesional vasculature to constrict, preventing lymphatic fluid from draining into the veins.11 It is theorized that the PDE5 inhibitor sildenafil leads to relaxation of the vasculature adjacent to LMs, allowing the outflow of the accumulated lymphatic fluid and thus decompression.11-13

Management of LM should not only take into account the depth and location of involvement but also any associated symptoms or complications, such as pruritus, pain, bleeding, or secondary infections. Magnetic resonance imaging (MRI) typically has been considered the gold standard for determining the size and depth of involvement of the malformation.1,3,4 However, ultrasonography with Doppler flow may be considered an initial diagnostic and screening test, as it can distinguish between macrocystic and microcystic components and provide superior images of microcystic lesions, which are below the resolution capacity of MRI.4 Notably, our patient’s LM was undetectable on ultrasonography and was found to be largely superficial in nature on MRI.

Serial excision of the microcystic LM was conducted in our patient, but there currently is no consensus on optimal treatment of LM, and many treatment options are complicated by high recurrence rates or complications.5 Procedural approaches may include excision, cryotherapy, radiotherapy, sclerotherapy, or laser therapy, while pharmacologic approaches may include sildenafil for its inhibition of PDE5 or sirolimus (oral or topical) for its inhibition of mammalian target of rapamycin.5,12-14 Because recurrence is highly likely, patients may require repeat treatments or a combination approach to therapy.1,5 The development of targeted therapies may lead to a shift in management of LMs in the future, as successful use of the PIK3CA inhibitor alpelisib recently has been reported to lead to clinical improvement of PIK3CA-related LMs, including in patients with PIK3CA-related overgrowth syndromes.15

References
  1. Garzon MC, Huang JT, Enjolras O, et al. Vascular malformations: part I. J Am Acad Dermatol. 2007;56:353-374. doi:10.1016/j.jaad.2006.05.069
  2. Alrashdan MS, Hammad HM, Alzumaili BAI, et al. Lymphangioma circumscriptum of the tongue: a case with marked hemorrhagic component. J Cutan Pathol. 2018;45:278-281. doi:10.1111/cup.13101
  3. Osborne GE, Chinn RJ, Francis ND, et al. Magnetic resonance imaging in the investigation of penile lymphangioma circumscriptum. Br J Dermatol. 2000;143:467-468. doi:10.1046/j.1365-2133.2000.03695.x
  4. Davies D, Rogers M, Lam A, et al. Localized microcystic lymphatic malformations—ultrasound diagnosis. Pediatr Dermatol. 1999;16: 423-429. doi:10.1046/j.1525-1470.1999.00110.x
  5. García-Montero P, Del Boz J, Baselga-Torres E, et al. Use of topical rapamycin in the treatment of superficial lymphatic malformations. J Am Acad Dermatol. 2019;80:508-515. doi:10.1016/j.jaad.2018.09.050
  6. Clarindo MV, Possebon AT, Soligo EM, et al. Dermatitis herpetiformis: pathophysiology, clinical presentation, diagnosis and treatment. An Bras Dermatol. 2014;89:865-875; quiz 876-877. doi:10.1590/abd1806-4841.20142966
  7. Leinweber B, Kerl H, Cerroni L. Histopathologic features of cutaneous herpes virus infections (herpes simplex, herpes varicella/zoster): a broad spectrum of presentations with common pseudolymphomatous aspects. Am J Surg Pathol. 2006;30:50-58.
  8. Shiver M, Papasakelariou C, Brown JA, et al. Extragenital bullous lichen sclerosus in a pediatric patient: a case report and literature review. Pediatr Dermatol. 2014;31:383-385. doi:10.1111 /pde.12025
  9. Blesinger H, Kaulfuß S, Aung T, et al. PIK3CA mutations are specifically localized to lymphatic endothelial cells of lymphatic malformations [published online July 9, 2018]. PLoS One. 2018;13:E0200343. doi:10.1371/journal.pone.0200343
  10. Green JS, Prok L, Bruckner AL. Expression of phosphodiesterase-5 in lymphatic malformation tissue. JAMA Dermatol. 2014;150:455-456. doi:10.1001/jamadermatol.2013.7002
  11. Swetman GL, Berk DR, Vasanawala SS, et al. Sildenafil for severe lymphatic malformations. N Engl J Med. 2012;366:384-386. doi:10.1056 /NEJMc1112482
  12. Tu JH, Tafoya E, Jeng M, et al. Long-term follow-up of lymphatic malformations in children treated with sildenafil. Pediatr Dermatol. 2017;34:559-565. doi:10.1111/pde.13237
  13. Maruani A, Tavernier E, Boccara O, et al. Sirolimus (rapamycin) for slow-flow malformations in children: the Observational-Phase Randomized Clinical PERFORMUS Trial. JAMA Dermatol. 2021;157:1289-1298. doi:10.1001/jamadermatol.2021.3459
  14. Delestre F, Venot Q, Bayard C, et al. Alpelisib administration reduced lymphatic malformations in a mouse model and in patients. Sci Transl Med. 2021;13:eabg0809. doi:10.1126/scitranslmed .abg0809
  15. Garreta Fontelles G, Pardo Pastor J, Grande Moreillo C. Alpelisib to treat CLOVES syndrome, a member of the PIK3CA-related overgrowth syndrome spectrum [published online February 21, 2022]. Br J Clin Pharmacol. 2022;88:3891-3895. doi:10.1111/bcp.15270
Article PDF
Author and Disclosure Information

From the University of Arkansas for Medical Sciences, Little Rock. Dr. Kayishunge is from the College of Medicine, Drs. Rollins and Gonzalez-Krellwitz are from the Department of Pathology, and Dr. Evans is from the Department of Dermatology.

The authors report no conflict of interest.

Correspondence: Delice Kayishunge, MD, MSc, 4301 W Markham St, Slot 576, Little Rock, AR 72205 (dkayishunge@uams.edu).

Issue
Cutis - 112(5)
Publications
Topics
Page Number
245-247
Sections
Author and Disclosure Information

From the University of Arkansas for Medical Sciences, Little Rock. Dr. Kayishunge is from the College of Medicine, Drs. Rollins and Gonzalez-Krellwitz are from the Department of Pathology, and Dr. Evans is from the Department of Dermatology.

The authors report no conflict of interest.

Correspondence: Delice Kayishunge, MD, MSc, 4301 W Markham St, Slot 576, Little Rock, AR 72205 (dkayishunge@uams.edu).

Author and Disclosure Information

From the University of Arkansas for Medical Sciences, Little Rock. Dr. Kayishunge is from the College of Medicine, Drs. Rollins and Gonzalez-Krellwitz are from the Department of Pathology, and Dr. Evans is from the Department of Dermatology.

The authors report no conflict of interest.

Correspondence: Delice Kayishunge, MD, MSc, 4301 W Markham St, Slot 576, Little Rock, AR 72205 (dkayishunge@uams.edu).

Article PDF
Article PDF
Related Articles

The Diagnosis: Microcystic Lymphatic Malformation

A punch biopsy demonstrated anastomosing fluidfilled spaces within the papillary and reticular dermal layers (Figure), confirming the diagnosis of microcystic lymphatic malformation (LM)(formerly known as lymphangioma circumscriptum), a congenital vascular malformation composed of slow-flow lymphatic channels.1 The patient underwent serial excisions with improvement of the LM, though the treatment course was complicated by hypertrophic scar formation.

Kayishunge.jpg
%3Cp%3EAn%20unencapsulated%20proliferation%20of%20anastomosing%20vascular%20spaces%20within%20the%20papillary%20and%20reticular%20dermis%20(H%26amp%3BE%2C%20original%20magnification%20%C3%9720).%3C%2Fp%3E

The classic clinical presentation of microcystic LM includes a crop of vesicles containing clear or hemorrhagic fluid with associated oozing or bleeding.2 When cutaneous lesions resembling microcystic LM develop in response to lymphatic damage and resulting stasis, such as from prior radiotherapy or surgery, the term lymphangiectasia is used to distinguish this entity from congenital microcystic LM.3

Microcystic LMs are histologically indistinguishable from macrocystic LMs; however, macrocystic LMs typically are clinically evident at birth as ill-defined subcutaneous masses.2,4-6 Dermatitis herpetiformis, a dermatologic manifestation of gluten sensitivity, causes intensely pruritic vesicles in a symmetric distribution on the elbows, knees, and buttocks. Histopathology shows neutrophilic microabscesses in the dermal papillae with subepidermal blistering. Direct immunofluorescence demonstrates the deposition of IgA along the basement membrane with dermal papillae aggregates.6 The underlying dermis also may contain a lymphohistiocytic infiltrate rich in neutrophils. The vesicles of herpes zoster virus are painful and present in a dermatomal distribution. A viral cytopathic effect often is observed in keratinocytes, specifically with multinucleation, molding, and margination of chromatin material. The lesions are accompanied by variable lymphocytic inflammation and epithelial necrosis resulting in intraepidermal blistering.7 Extragenital lichen sclerosus presents as polygonal white papules merging to form plaques and may include hemorrhagic blisters in some instances. Histopathology shows hyperkeratosis, epidermal atrophy with flattened rete ridges, vacuolar interface changes, loss of elastic fibers, and hyalinization of the lamina propria with lymphocytic infiltrate.8

Endothelial cells in LM exhibit activating mutations in the phosphatidylinositol-4,5-bisphosphate 3-kinase catalytic subunit alpha gene, PIK3CA, which may lead to proliferation and overgrowth of the lymphatic vasculature, as well as increased production of cyclic guanosine monophosphate.9,10 Phosphodiesterase 5 (PDE5) is expressed in the perivascular smooth muscle adjacent to lymphatic spaces in LMs but not in the their vasculature. 10 This pattern of PDE5 expression may cause perilesional vasculature to constrict, preventing lymphatic fluid from draining into the veins.11 It is theorized that the PDE5 inhibitor sildenafil leads to relaxation of the vasculature adjacent to LMs, allowing the outflow of the accumulated lymphatic fluid and thus decompression.11-13

Management of LM should not only take into account the depth and location of involvement but also any associated symptoms or complications, such as pruritus, pain, bleeding, or secondary infections. Magnetic resonance imaging (MRI) typically has been considered the gold standard for determining the size and depth of involvement of the malformation.1,3,4 However, ultrasonography with Doppler flow may be considered an initial diagnostic and screening test, as it can distinguish between macrocystic and microcystic components and provide superior images of microcystic lesions, which are below the resolution capacity of MRI.4 Notably, our patient’s LM was undetectable on ultrasonography and was found to be largely superficial in nature on MRI.

Serial excision of the microcystic LM was conducted in our patient, but there currently is no consensus on optimal treatment of LM, and many treatment options are complicated by high recurrence rates or complications.5 Procedural approaches may include excision, cryotherapy, radiotherapy, sclerotherapy, or laser therapy, while pharmacologic approaches may include sildenafil for its inhibition of PDE5 or sirolimus (oral or topical) for its inhibition of mammalian target of rapamycin.5,12-14 Because recurrence is highly likely, patients may require repeat treatments or a combination approach to therapy.1,5 The development of targeted therapies may lead to a shift in management of LMs in the future, as successful use of the PIK3CA inhibitor alpelisib recently has been reported to lead to clinical improvement of PIK3CA-related LMs, including in patients with PIK3CA-related overgrowth syndromes.15

The Diagnosis: Microcystic Lymphatic Malformation

A punch biopsy demonstrated anastomosing fluidfilled spaces within the papillary and reticular dermal layers (Figure), confirming the diagnosis of microcystic lymphatic malformation (LM)(formerly known as lymphangioma circumscriptum), a congenital vascular malformation composed of slow-flow lymphatic channels.1 The patient underwent serial excisions with improvement of the LM, though the treatment course was complicated by hypertrophic scar formation.

Kayishunge.jpg
%3Cp%3EAn%20unencapsulated%20proliferation%20of%20anastomosing%20vascular%20spaces%20within%20the%20papillary%20and%20reticular%20dermis%20(H%26amp%3BE%2C%20original%20magnification%20%C3%9720).%3C%2Fp%3E

The classic clinical presentation of microcystic LM includes a crop of vesicles containing clear or hemorrhagic fluid with associated oozing or bleeding.2 When cutaneous lesions resembling microcystic LM develop in response to lymphatic damage and resulting stasis, such as from prior radiotherapy or surgery, the term lymphangiectasia is used to distinguish this entity from congenital microcystic LM.3

Microcystic LMs are histologically indistinguishable from macrocystic LMs; however, macrocystic LMs typically are clinically evident at birth as ill-defined subcutaneous masses.2,4-6 Dermatitis herpetiformis, a dermatologic manifestation of gluten sensitivity, causes intensely pruritic vesicles in a symmetric distribution on the elbows, knees, and buttocks. Histopathology shows neutrophilic microabscesses in the dermal papillae with subepidermal blistering. Direct immunofluorescence demonstrates the deposition of IgA along the basement membrane with dermal papillae aggregates.6 The underlying dermis also may contain a lymphohistiocytic infiltrate rich in neutrophils. The vesicles of herpes zoster virus are painful and present in a dermatomal distribution. A viral cytopathic effect often is observed in keratinocytes, specifically with multinucleation, molding, and margination of chromatin material. The lesions are accompanied by variable lymphocytic inflammation and epithelial necrosis resulting in intraepidermal blistering.7 Extragenital lichen sclerosus presents as polygonal white papules merging to form plaques and may include hemorrhagic blisters in some instances. Histopathology shows hyperkeratosis, epidermal atrophy with flattened rete ridges, vacuolar interface changes, loss of elastic fibers, and hyalinization of the lamina propria with lymphocytic infiltrate.8

Endothelial cells in LM exhibit activating mutations in the phosphatidylinositol-4,5-bisphosphate 3-kinase catalytic subunit alpha gene, PIK3CA, which may lead to proliferation and overgrowth of the lymphatic vasculature, as well as increased production of cyclic guanosine monophosphate.9,10 Phosphodiesterase 5 (PDE5) is expressed in the perivascular smooth muscle adjacent to lymphatic spaces in LMs but not in the their vasculature. 10 This pattern of PDE5 expression may cause perilesional vasculature to constrict, preventing lymphatic fluid from draining into the veins.11 It is theorized that the PDE5 inhibitor sildenafil leads to relaxation of the vasculature adjacent to LMs, allowing the outflow of the accumulated lymphatic fluid and thus decompression.11-13

Management of LM should not only take into account the depth and location of involvement but also any associated symptoms or complications, such as pruritus, pain, bleeding, or secondary infections. Magnetic resonance imaging (MRI) typically has been considered the gold standard for determining the size and depth of involvement of the malformation.1,3,4 However, ultrasonography with Doppler flow may be considered an initial diagnostic and screening test, as it can distinguish between macrocystic and microcystic components and provide superior images of microcystic lesions, which are below the resolution capacity of MRI.4 Notably, our patient’s LM was undetectable on ultrasonography and was found to be largely superficial in nature on MRI.

Serial excision of the microcystic LM was conducted in our patient, but there currently is no consensus on optimal treatment of LM, and many treatment options are complicated by high recurrence rates or complications.5 Procedural approaches may include excision, cryotherapy, radiotherapy, sclerotherapy, or laser therapy, while pharmacologic approaches may include sildenafil for its inhibition of PDE5 or sirolimus (oral or topical) for its inhibition of mammalian target of rapamycin.5,12-14 Because recurrence is highly likely, patients may require repeat treatments or a combination approach to therapy.1,5 The development of targeted therapies may lead to a shift in management of LMs in the future, as successful use of the PIK3CA inhibitor alpelisib recently has been reported to lead to clinical improvement of PIK3CA-related LMs, including in patients with PIK3CA-related overgrowth syndromes.15

References
  1. Garzon MC, Huang JT, Enjolras O, et al. Vascular malformations: part I. J Am Acad Dermatol. 2007;56:353-374. doi:10.1016/j.jaad.2006.05.069
  2. Alrashdan MS, Hammad HM, Alzumaili BAI, et al. Lymphangioma circumscriptum of the tongue: a case with marked hemorrhagic component. J Cutan Pathol. 2018;45:278-281. doi:10.1111/cup.13101
  3. Osborne GE, Chinn RJ, Francis ND, et al. Magnetic resonance imaging in the investigation of penile lymphangioma circumscriptum. Br J Dermatol. 2000;143:467-468. doi:10.1046/j.1365-2133.2000.03695.x
  4. Davies D, Rogers M, Lam A, et al. Localized microcystic lymphatic malformations—ultrasound diagnosis. Pediatr Dermatol. 1999;16: 423-429. doi:10.1046/j.1525-1470.1999.00110.x
  5. García-Montero P, Del Boz J, Baselga-Torres E, et al. Use of topical rapamycin in the treatment of superficial lymphatic malformations. J Am Acad Dermatol. 2019;80:508-515. doi:10.1016/j.jaad.2018.09.050
  6. Clarindo MV, Possebon AT, Soligo EM, et al. Dermatitis herpetiformis: pathophysiology, clinical presentation, diagnosis and treatment. An Bras Dermatol. 2014;89:865-875; quiz 876-877. doi:10.1590/abd1806-4841.20142966
  7. Leinweber B, Kerl H, Cerroni L. Histopathologic features of cutaneous herpes virus infections (herpes simplex, herpes varicella/zoster): a broad spectrum of presentations with common pseudolymphomatous aspects. Am J Surg Pathol. 2006;30:50-58.
  8. Shiver M, Papasakelariou C, Brown JA, et al. Extragenital bullous lichen sclerosus in a pediatric patient: a case report and literature review. Pediatr Dermatol. 2014;31:383-385. doi:10.1111 /pde.12025
  9. Blesinger H, Kaulfuß S, Aung T, et al. PIK3CA mutations are specifically localized to lymphatic endothelial cells of lymphatic malformations [published online July 9, 2018]. PLoS One. 2018;13:E0200343. doi:10.1371/journal.pone.0200343
  10. Green JS, Prok L, Bruckner AL. Expression of phosphodiesterase-5 in lymphatic malformation tissue. JAMA Dermatol. 2014;150:455-456. doi:10.1001/jamadermatol.2013.7002
  11. Swetman GL, Berk DR, Vasanawala SS, et al. Sildenafil for severe lymphatic malformations. N Engl J Med. 2012;366:384-386. doi:10.1056 /NEJMc1112482
  12. Tu JH, Tafoya E, Jeng M, et al. Long-term follow-up of lymphatic malformations in children treated with sildenafil. Pediatr Dermatol. 2017;34:559-565. doi:10.1111/pde.13237
  13. Maruani A, Tavernier E, Boccara O, et al. Sirolimus (rapamycin) for slow-flow malformations in children: the Observational-Phase Randomized Clinical PERFORMUS Trial. JAMA Dermatol. 2021;157:1289-1298. doi:10.1001/jamadermatol.2021.3459
  14. Delestre F, Venot Q, Bayard C, et al. Alpelisib administration reduced lymphatic malformations in a mouse model and in patients. Sci Transl Med. 2021;13:eabg0809. doi:10.1126/scitranslmed .abg0809
  15. Garreta Fontelles G, Pardo Pastor J, Grande Moreillo C. Alpelisib to treat CLOVES syndrome, a member of the PIK3CA-related overgrowth syndrome spectrum [published online February 21, 2022]. Br J Clin Pharmacol. 2022;88:3891-3895. doi:10.1111/bcp.15270
References
  1. Garzon MC, Huang JT, Enjolras O, et al. Vascular malformations: part I. J Am Acad Dermatol. 2007;56:353-374. doi:10.1016/j.jaad.2006.05.069
  2. Alrashdan MS, Hammad HM, Alzumaili BAI, et al. Lymphangioma circumscriptum of the tongue: a case with marked hemorrhagic component. J Cutan Pathol. 2018;45:278-281. doi:10.1111/cup.13101
  3. Osborne GE, Chinn RJ, Francis ND, et al. Magnetic resonance imaging in the investigation of penile lymphangioma circumscriptum. Br J Dermatol. 2000;143:467-468. doi:10.1046/j.1365-2133.2000.03695.x
  4. Davies D, Rogers M, Lam A, et al. Localized microcystic lymphatic malformations—ultrasound diagnosis. Pediatr Dermatol. 1999;16: 423-429. doi:10.1046/j.1525-1470.1999.00110.x
  5. García-Montero P, Del Boz J, Baselga-Torres E, et al. Use of topical rapamycin in the treatment of superficial lymphatic malformations. J Am Acad Dermatol. 2019;80:508-515. doi:10.1016/j.jaad.2018.09.050
  6. Clarindo MV, Possebon AT, Soligo EM, et al. Dermatitis herpetiformis: pathophysiology, clinical presentation, diagnosis and treatment. An Bras Dermatol. 2014;89:865-875; quiz 876-877. doi:10.1590/abd1806-4841.20142966
  7. Leinweber B, Kerl H, Cerroni L. Histopathologic features of cutaneous herpes virus infections (herpes simplex, herpes varicella/zoster): a broad spectrum of presentations with common pseudolymphomatous aspects. Am J Surg Pathol. 2006;30:50-58.
  8. Shiver M, Papasakelariou C, Brown JA, et al. Extragenital bullous lichen sclerosus in a pediatric patient: a case report and literature review. Pediatr Dermatol. 2014;31:383-385. doi:10.1111 /pde.12025
  9. Blesinger H, Kaulfuß S, Aung T, et al. PIK3CA mutations are specifically localized to lymphatic endothelial cells of lymphatic malformations [published online July 9, 2018]. PLoS One. 2018;13:E0200343. doi:10.1371/journal.pone.0200343
  10. Green JS, Prok L, Bruckner AL. Expression of phosphodiesterase-5 in lymphatic malformation tissue. JAMA Dermatol. 2014;150:455-456. doi:10.1001/jamadermatol.2013.7002
  11. Swetman GL, Berk DR, Vasanawala SS, et al. Sildenafil for severe lymphatic malformations. N Engl J Med. 2012;366:384-386. doi:10.1056 /NEJMc1112482
  12. Tu JH, Tafoya E, Jeng M, et al. Long-term follow-up of lymphatic malformations in children treated with sildenafil. Pediatr Dermatol. 2017;34:559-565. doi:10.1111/pde.13237
  13. Maruani A, Tavernier E, Boccara O, et al. Sirolimus (rapamycin) for slow-flow malformations in children: the Observational-Phase Randomized Clinical PERFORMUS Trial. JAMA Dermatol. 2021;157:1289-1298. doi:10.1001/jamadermatol.2021.3459
  14. Delestre F, Venot Q, Bayard C, et al. Alpelisib administration reduced lymphatic malformations in a mouse model and in patients. Sci Transl Med. 2021;13:eabg0809. doi:10.1126/scitranslmed .abg0809
  15. Garreta Fontelles G, Pardo Pastor J, Grande Moreillo C. Alpelisib to treat CLOVES syndrome, a member of the PIK3CA-related overgrowth syndrome spectrum [published online February 21, 2022]. Br J Clin Pharmacol. 2022;88:3891-3895. doi:10.1111/bcp.15270
Issue
Cutis - 112(5)
Issue
Cutis - 112(5)
Page Number
245-247
Page Number
245-247
Publications
Publications
Topics
Article Type
Display Headline
Clustered Vesicles on the Neck
Display Headline
Clustered Vesicles on the Neck
Sections
Questionnaire Body

A 6-year-old girl presented to the dermatology clinic with a rash on the right side of the neck that was noted at birth as a small raised lesion but slowly increased over time in size and number of lesions. She reported pruritus and irritation, particularly when rubbed or scratched. There was no family history of similar skin abnormalities. Her medical history was notable for a left-sided cholesteatoma on tympanomastoidectomy. Physical examination revealed clustered vesicles on the right side of the neck with underlying erythema. The vesicles contained mostly clear fluid with a few focal areas of hemorrhagic fluid. Ultrasonography was unremarkable, and magnetic resonance imaging revealed superficial T2 hyperintense nonenhancing cutaneous and subcutaneous lesions overlying the right lateral neck with minimal extension into the superficial right supraclavicular soft tissues.

Kayishunge_quiz.jpg

Disallow All Ads
Content Gating
No Gating (article Unlocked/Free)
Alternative CME
Disqus Comments
Default
Gate On Date
Mon, 10/30/2023 - 09:45
Un-Gate On Date
Mon, 10/30/2023 - 09:45
Use ProPublica
CFC Schedule Remove Status
Mon, 10/30/2023 - 09:45
Hide sidebar & use full width
render the right sidebar.
Conference Recap Checkbox
Not Conference Recap
Clinical Edge
Display the Slideshow in this Article
Medscape Article
Display survey writer
Reuters content
Disable Inline Native ads
WebMD Article
Article PDF Media
Image
Teambase ID
180025EC.SIG
Disable zoom
Off

Blue Nodules on the Forearms in an Active-Duty Military Servicemember

Article Type
Changed
Mon, 10/30/2023 - 17:32
Display Headline
Blue Nodules on the Forearms in an Active-Duty Military Servicemember

The Diagnosis: Glomangiomyoma

A punch biopsy of the right forearm revealed a collection of vascular and smooth muscle components with small and spindled bland cells containing minimal eosinophilic cytoplasm (Figure 1), confirming the diagnosis of glomangiomyoma. Immunohistochemical stains also supported the diagnosis and were positive for smooth muscle actin, desmin, and CD34 (Figure 2). Magnetic resonance imaging from a prior attempt at treatment with sclerotherapy demonstrated scattered vascular malformations with no notable internal derangement. There was no improvement with sclerotherapy. Given the number and vascular nature of the lesions, a trial of pulsed dye laser (PDL) therapy was administered and tolerated by the patient. He subsequently moved to a new military duty station. On follow-up, he reported no noticeable clinical improvement in the lesions after PDL and opted not to continue with laser treatment.

CT112004026_e_Fig1_AB.jpg
%3Cp%3E%3Cstrong%3EFIGURE%201.%3C%2Fstrong%3E%20A%20and%20B%2C%20Histopathology%20revealed%20a%20collection%20of%20dilated%20and%20variably%20sized%20vascular%20spaces%20in%20the%20dermis%20surrounded%20by%20small%20bland%20cells%20with%20little%20cytoplasm%20as%20well%20as%20some%20foci%20between%20the%20vascular%20spaces%20containing%20cells%20that%20were%20more%20spindled%20and%20had%20increased%20amounts%20of%20eosinophilic%20cytoplasm%20(H%26amp%3BE%2C%20original%20magnifications%20%C3%9720%20and%20%C3%9780).%20Reference%20bars%20indicate%201%20mm%20and%20300%20%CE%BCm%2C%20respectively.%3C%2Fp%3E

Glomangiomyoma is a rare and benign glomus tumor variant that demonstrates differentiation into the smooth muscle and potentially can result in substantial complications.1 Glomus tumors generally are benign neoplasms of the glomus apparatus, and glomus cells function as thermoregulators in the reticular dermis.2 Glomus tumors comprise less than 2% of soft tissue neoplasms and generally are solitary nodules; only 10% of glomus tumors occur with multiple lesions, and among them, glomangiomyoma is the rarest subtype, presenting in only 15% of cases.2,3 The 3 main subtypes of glomus tumors are solid, glomangioma, and glomangiomyoma.4 Clinically, the lesions may present as small blue nodules with associated pain and cold or pressure sensitivity. Although there appears to be variation of the nomenclature depending on the source in the literature, glomangiomas are characterized by their predominant vascular malformations on biopsy. Glomangiomyomas are a subset of glomus tumors with distinct smooth muscle differentiation.4 Given their pathologic presentation, our patient’s lesions were most consistent with the diagnosis of glomangiomyoma.

CT112004026_e_Fig2_ABC.jpg
%3Cp%3E%3Cstrong%3EFIGURE%202.%3C%2Fstrong%3E%20A%2C%20Immunohistochemistry%20revealed%20small%20cells%20that%20were%20decorated%20with%20smooth%20muscle%20actin%20(original%20magnification%20%C3%9750).%20B%2C%20Spindled%20cells%20were%20highlighted%20with%20desmin%20(original%20magnification%20%C3%97200).%20C%2C%20CD34%20highlighted%20the%20endothelial%20cells%20lining%20the%20spaces%20(original%20magnification%20%C3%9720).%20Reference%20bars%20indicate%20100%20%CE%BCm%2C%20100%20%CE%BCm%2C%20and%201%20mm%2C%20respectively.%3C%2Fp%3E

The small size of the lesions may result in difficulty establishing a clinical diagnosis, particularly if there is no hand involvement, where lesions most commonly occur.2 Therefore, histopathologic evaluation is essential and is the best initial step in evaluating glomangiomyomas.4 Biopsy is the most reliable means of confirming a diagnosis2,4,5; however, diagnostic imaging such as a computed tomography also should be performed if considering blue rubber bleb nevus syndrome due to the primary site of involvement. Surgical excision is the treatment of choice after confirming the diagnosis in most cases of symptomatic glomangiomyomas, particularly with painful lesions.6

Neurilemmomas (also known as schwannomas) are benign lesions that generally present as asymptomatic, soft, smooth nodules most often on the neck; however, they also may present on the flexor extremities or in internal organs. Although primarily asymptomatic, the tumors may be associated with pain and paresthesia as they enlarge and affect surrounding structures. Neurilemmomas may occur spontaneously or as part of a syndrome, such as neurofibromatosis type 2 or Carney complex.7

Hereditary hemorrhagic telangiectasia (formerly known as Osler-Weber-Rendu syndrome) is an autosomal-dominant disease that presents with arteriovenous malformations and telangiectases. Patients generally present in the third decade of life, with the main concern generally being epistaxis.8

Kaposi sarcoma is a viral infection secondary to human herpesvirus 8 that results in red-purple lesions commonly on mucocutaneous sites. Kaposi sarcoma can be AIDS associated and non-HIV associated. Although clinically indistinguishable, a few subtle histologic features can assist in differentiating the 2 etiologies. In addition to a potential history of immunodeficiency, evaluating for involvement of the lymphatic system, respiratory tract, or gastrointestinal tract can aid in differentiating this entity from glomus tumors.9

Leiomyomas are smooth muscle lesions divided into 3 subcategories: angioleiomyoma, piloleiomyoma, and genital leiomyoma. The clinical presentation and histopathology will vary depending on the subcategory. Although cutaneous leiomyomas are benign, further workup for piloleiomyoma may be required given the reported association with hereditary leiomyomatosis and renal cell cancer (Reed syndrome).10

Imaging can be helpful when the clinical diagnosis of a glomus tumor vs other painful neoplasms of the skin is unclear, such as in blue rubber bleb nevus syndrome, angioleiomyomas, neuromas, glomus tumors, leiomyomas, eccrine spiradenomas, congenital vascular malformations, schwannomas, or hemangiomas.4 Radiologic findings for glomus tumors may demonstrate cortical or cystic osseous defects. Magnetic resonance imaging and ultrasonography can help provide additional information on the lesion size and depth of involvement.1 Additionally, deeper glomangiomyomas have been associated with malignancy,2 potentially highlighting the benefit of early incorporation of imaging in the workup for this condition. Malignant transformation is rare and has been reported in less than 1% of cases.6

Treatment of glomus tumors predominantly is directed to the patient’s symptoms; asymptomatic lesions may be monitored.4 For symptomatic lesions, therapeutic options include wide local excision; sclerotherapy; and incorporation of various lasers, including Nd:YAG, CO2, and flashlamp tunable dye laser.4,5 One case report documented use of a PDL that successfully eliminated the pain associated with glomangiomyoma; however, the lesion in that report was not biopsy proven.11

Our case highlights the need to consider glomus tumors in patients presenting with multiple small nodules given the potential for misdiagnosis, impact on quality of life with associated psychological distress, and potential utility of incorporating PDL in treatment. Although our patient did not report clinical improvement in the appearance of the lesions with PDL therapy, additional treatment sessions may have helped,11 but he opted to discontinue. Follow-up for persistently symptomatic or changing lesions is necessary, given the minimal risk for malignant transformation.6

References
  1. Lee DY, Hwang SC, Jeong ST, et al. The value of diagnostic ultrasonography in the assessment of a glomus tumor of the subcutaneous layer of the forearm mimicking a hemangioma: a case report. J Med Case Rep. 2015;9:191. doi:10.1186/s13256-015-0672-y
  2. Li L, Bardsley V, Grainger A, et al. Extradigital glomangiomyoma of the forearm mimicking peripheral nerve sheath tumour and thrombosed varicose vein. BMJ Case Rep. 2021;14:E241221. doi: 10.1136 /bcr-2020-241221
  3. Calduch L, Monteagudo C, Martínez-Ruiz E, et al. Familial generalized multiple glomangiomyoma: report of a new family, with immunohistochemical and ultrastructural studies and review of the literature. Pediatr Dermatol. 2002;19:402-408. doi:10.1046/j.1525-1470.2002.00114.x
  4. Mohammadi O, Suarez M. Glomus cancer. StatPearls [Internet]. StatPearls Publishing; 2021.
  5. Maxey ML, Houghton CC, Mastriani KS, et al. Large prepatellar glomangioma: a case report [published online July 10, 2015]. Int J Surg Case Rep. 2015;14:80-84. doi:10.1016/j.ijscr.2015.07.002
  6. Brathwaite CD, Poppiti RJ Jr. Malignant glomus tumor. a case report of widespread metastases in a patient with multiple glomus body hamartomas. Am J Surg Pathol. 1996;20:233-238. doi:10.1097/00000478-199602000-00012
  7. Davis DD, Kane SM. Neurilemmoma. StatPearls [Internet]. StatPearls Publishing; 2022.
  8. Kühnel T, Wirsching K, Wohlgemuth W, et al. Hereditary hemorrhagic telangiectasia. Otolaryngol Clin North Am. 2018;51:237-254. doi:10.1016/j.otc.2017.09.017
  9. Radu O, Pantanowitz L. Kaposi sarcoma. Arch Pathol Lab Med. 2013;137:289-294. doi:10.5858/arpa.2012-0101-RS
  10. Bernett CN, Mammino JJ. Cutaneous leiomyomas. StatPearls [Internet]. StatPearls Publishing; 2022.
  11. Antony FC, Cliff S, Cowley N. Complete pain relief following treatment of a glomangiomyoma with the pulsed dye laser. Clin Exp Dermatol. 2003;28:617-619. doi:10.1046/j.1365-2230.2003.01403.x
Article PDF
Author and Disclosure Information

Dr. Eubanks is from the San Antonio Uniformed Services Health Education Consortium, Brooke Army Medical Center, Joint Base San Antonio–Fort Sam Houston, Texas. Drs. Mescher and McGinley Simpson are from the Department of Dermatology, Walter Reed National Military Medical Center, Bethesda, Maryland. Dr. Royer is from the Division of Dermatopathology, The Joint Pathology Center, Silver Spring, Maryland.

The authors report no conflict of interest.

The views expressed herein are those of the authors and do not reflect the official policy or position of Brooke Army Medical Center, Walter Reed National Military Medical Center, the US Army Medical Department, the Defense Health Agency, the US Army Office of the Surgeon General, the Department of the Army, the Department of the Air Force, the Department of the Navy, the Department of Defense, or the US Government. This case was presented at the Uniformed Services University of the Health Sciences 2022 Research Day; May 3, 2022.

Correspondence: Bianca N. Eubanks, MD, 3551 Roger Brooke Dr, JBSA–Fort Sam Houston, TX 78234 (bianca.n.eubanks@gmail.com).

Issue
Cutis - 112(4)
Publications
Topics
Page Number
E26-E28
Sections
Author and Disclosure Information

Dr. Eubanks is from the San Antonio Uniformed Services Health Education Consortium, Brooke Army Medical Center, Joint Base San Antonio–Fort Sam Houston, Texas. Drs. Mescher and McGinley Simpson are from the Department of Dermatology, Walter Reed National Military Medical Center, Bethesda, Maryland. Dr. Royer is from the Division of Dermatopathology, The Joint Pathology Center, Silver Spring, Maryland.

The authors report no conflict of interest.

The views expressed herein are those of the authors and do not reflect the official policy or position of Brooke Army Medical Center, Walter Reed National Military Medical Center, the US Army Medical Department, the Defense Health Agency, the US Army Office of the Surgeon General, the Department of the Army, the Department of the Air Force, the Department of the Navy, the Department of Defense, or the US Government. This case was presented at the Uniformed Services University of the Health Sciences 2022 Research Day; May 3, 2022.

Correspondence: Bianca N. Eubanks, MD, 3551 Roger Brooke Dr, JBSA–Fort Sam Houston, TX 78234 (bianca.n.eubanks@gmail.com).

Author and Disclosure Information

Dr. Eubanks is from the San Antonio Uniformed Services Health Education Consortium, Brooke Army Medical Center, Joint Base San Antonio–Fort Sam Houston, Texas. Drs. Mescher and McGinley Simpson are from the Department of Dermatology, Walter Reed National Military Medical Center, Bethesda, Maryland. Dr. Royer is from the Division of Dermatopathology, The Joint Pathology Center, Silver Spring, Maryland.

The authors report no conflict of interest.

The views expressed herein are those of the authors and do not reflect the official policy or position of Brooke Army Medical Center, Walter Reed National Military Medical Center, the US Army Medical Department, the Defense Health Agency, the US Army Office of the Surgeon General, the Department of the Army, the Department of the Air Force, the Department of the Navy, the Department of Defense, or the US Government. This case was presented at the Uniformed Services University of the Health Sciences 2022 Research Day; May 3, 2022.

Correspondence: Bianca N. Eubanks, MD, 3551 Roger Brooke Dr, JBSA–Fort Sam Houston, TX 78234 (bianca.n.eubanks@gmail.com).

Article PDF
Article PDF
Related Articles

The Diagnosis: Glomangiomyoma

A punch biopsy of the right forearm revealed a collection of vascular and smooth muscle components with small and spindled bland cells containing minimal eosinophilic cytoplasm (Figure 1), confirming the diagnosis of glomangiomyoma. Immunohistochemical stains also supported the diagnosis and were positive for smooth muscle actin, desmin, and CD34 (Figure 2). Magnetic resonance imaging from a prior attempt at treatment with sclerotherapy demonstrated scattered vascular malformations with no notable internal derangement. There was no improvement with sclerotherapy. Given the number and vascular nature of the lesions, a trial of pulsed dye laser (PDL) therapy was administered and tolerated by the patient. He subsequently moved to a new military duty station. On follow-up, he reported no noticeable clinical improvement in the lesions after PDL and opted not to continue with laser treatment.

CT112004026_e_Fig1_AB.jpg
%3Cp%3E%3Cstrong%3EFIGURE%201.%3C%2Fstrong%3E%20A%20and%20B%2C%20Histopathology%20revealed%20a%20collection%20of%20dilated%20and%20variably%20sized%20vascular%20spaces%20in%20the%20dermis%20surrounded%20by%20small%20bland%20cells%20with%20little%20cytoplasm%20as%20well%20as%20some%20foci%20between%20the%20vascular%20spaces%20containing%20cells%20that%20were%20more%20spindled%20and%20had%20increased%20amounts%20of%20eosinophilic%20cytoplasm%20(H%26amp%3BE%2C%20original%20magnifications%20%C3%9720%20and%20%C3%9780).%20Reference%20bars%20indicate%201%20mm%20and%20300%20%CE%BCm%2C%20respectively.%3C%2Fp%3E

Glomangiomyoma is a rare and benign glomus tumor variant that demonstrates differentiation into the smooth muscle and potentially can result in substantial complications.1 Glomus tumors generally are benign neoplasms of the glomus apparatus, and glomus cells function as thermoregulators in the reticular dermis.2 Glomus tumors comprise less than 2% of soft tissue neoplasms and generally are solitary nodules; only 10% of glomus tumors occur with multiple lesions, and among them, glomangiomyoma is the rarest subtype, presenting in only 15% of cases.2,3 The 3 main subtypes of glomus tumors are solid, glomangioma, and glomangiomyoma.4 Clinically, the lesions may present as small blue nodules with associated pain and cold or pressure sensitivity. Although there appears to be variation of the nomenclature depending on the source in the literature, glomangiomas are characterized by their predominant vascular malformations on biopsy. Glomangiomyomas are a subset of glomus tumors with distinct smooth muscle differentiation.4 Given their pathologic presentation, our patient’s lesions were most consistent with the diagnosis of glomangiomyoma.

CT112004026_e_Fig2_ABC.jpg
%3Cp%3E%3Cstrong%3EFIGURE%202.%3C%2Fstrong%3E%20A%2C%20Immunohistochemistry%20revealed%20small%20cells%20that%20were%20decorated%20with%20smooth%20muscle%20actin%20(original%20magnification%20%C3%9750).%20B%2C%20Spindled%20cells%20were%20highlighted%20with%20desmin%20(original%20magnification%20%C3%97200).%20C%2C%20CD34%20highlighted%20the%20endothelial%20cells%20lining%20the%20spaces%20(original%20magnification%20%C3%9720).%20Reference%20bars%20indicate%20100%20%CE%BCm%2C%20100%20%CE%BCm%2C%20and%201%20mm%2C%20respectively.%3C%2Fp%3E

The small size of the lesions may result in difficulty establishing a clinical diagnosis, particularly if there is no hand involvement, where lesions most commonly occur.2 Therefore, histopathologic evaluation is essential and is the best initial step in evaluating glomangiomyomas.4 Biopsy is the most reliable means of confirming a diagnosis2,4,5; however, diagnostic imaging such as a computed tomography also should be performed if considering blue rubber bleb nevus syndrome due to the primary site of involvement. Surgical excision is the treatment of choice after confirming the diagnosis in most cases of symptomatic glomangiomyomas, particularly with painful lesions.6

Neurilemmomas (also known as schwannomas) are benign lesions that generally present as asymptomatic, soft, smooth nodules most often on the neck; however, they also may present on the flexor extremities or in internal organs. Although primarily asymptomatic, the tumors may be associated with pain and paresthesia as they enlarge and affect surrounding structures. Neurilemmomas may occur spontaneously or as part of a syndrome, such as neurofibromatosis type 2 or Carney complex.7

Hereditary hemorrhagic telangiectasia (formerly known as Osler-Weber-Rendu syndrome) is an autosomal-dominant disease that presents with arteriovenous malformations and telangiectases. Patients generally present in the third decade of life, with the main concern generally being epistaxis.8

Kaposi sarcoma is a viral infection secondary to human herpesvirus 8 that results in red-purple lesions commonly on mucocutaneous sites. Kaposi sarcoma can be AIDS associated and non-HIV associated. Although clinically indistinguishable, a few subtle histologic features can assist in differentiating the 2 etiologies. In addition to a potential history of immunodeficiency, evaluating for involvement of the lymphatic system, respiratory tract, or gastrointestinal tract can aid in differentiating this entity from glomus tumors.9

Leiomyomas are smooth muscle lesions divided into 3 subcategories: angioleiomyoma, piloleiomyoma, and genital leiomyoma. The clinical presentation and histopathology will vary depending on the subcategory. Although cutaneous leiomyomas are benign, further workup for piloleiomyoma may be required given the reported association with hereditary leiomyomatosis and renal cell cancer (Reed syndrome).10

Imaging can be helpful when the clinical diagnosis of a glomus tumor vs other painful neoplasms of the skin is unclear, such as in blue rubber bleb nevus syndrome, angioleiomyomas, neuromas, glomus tumors, leiomyomas, eccrine spiradenomas, congenital vascular malformations, schwannomas, or hemangiomas.4 Radiologic findings for glomus tumors may demonstrate cortical or cystic osseous defects. Magnetic resonance imaging and ultrasonography can help provide additional information on the lesion size and depth of involvement.1 Additionally, deeper glomangiomyomas have been associated with malignancy,2 potentially highlighting the benefit of early incorporation of imaging in the workup for this condition. Malignant transformation is rare and has been reported in less than 1% of cases.6

Treatment of glomus tumors predominantly is directed to the patient’s symptoms; asymptomatic lesions may be monitored.4 For symptomatic lesions, therapeutic options include wide local excision; sclerotherapy; and incorporation of various lasers, including Nd:YAG, CO2, and flashlamp tunable dye laser.4,5 One case report documented use of a PDL that successfully eliminated the pain associated with glomangiomyoma; however, the lesion in that report was not biopsy proven.11

Our case highlights the need to consider glomus tumors in patients presenting with multiple small nodules given the potential for misdiagnosis, impact on quality of life with associated psychological distress, and potential utility of incorporating PDL in treatment. Although our patient did not report clinical improvement in the appearance of the lesions with PDL therapy, additional treatment sessions may have helped,11 but he opted to discontinue. Follow-up for persistently symptomatic or changing lesions is necessary, given the minimal risk for malignant transformation.6

The Diagnosis: Glomangiomyoma

A punch biopsy of the right forearm revealed a collection of vascular and smooth muscle components with small and spindled bland cells containing minimal eosinophilic cytoplasm (Figure 1), confirming the diagnosis of glomangiomyoma. Immunohistochemical stains also supported the diagnosis and were positive for smooth muscle actin, desmin, and CD34 (Figure 2). Magnetic resonance imaging from a prior attempt at treatment with sclerotherapy demonstrated scattered vascular malformations with no notable internal derangement. There was no improvement with sclerotherapy. Given the number and vascular nature of the lesions, a trial of pulsed dye laser (PDL) therapy was administered and tolerated by the patient. He subsequently moved to a new military duty station. On follow-up, he reported no noticeable clinical improvement in the lesions after PDL and opted not to continue with laser treatment.

CT112004026_e_Fig1_AB.jpg
%3Cp%3E%3Cstrong%3EFIGURE%201.%3C%2Fstrong%3E%20A%20and%20B%2C%20Histopathology%20revealed%20a%20collection%20of%20dilated%20and%20variably%20sized%20vascular%20spaces%20in%20the%20dermis%20surrounded%20by%20small%20bland%20cells%20with%20little%20cytoplasm%20as%20well%20as%20some%20foci%20between%20the%20vascular%20spaces%20containing%20cells%20that%20were%20more%20spindled%20and%20had%20increased%20amounts%20of%20eosinophilic%20cytoplasm%20(H%26amp%3BE%2C%20original%20magnifications%20%C3%9720%20and%20%C3%9780).%20Reference%20bars%20indicate%201%20mm%20and%20300%20%CE%BCm%2C%20respectively.%3C%2Fp%3E

Glomangiomyoma is a rare and benign glomus tumor variant that demonstrates differentiation into the smooth muscle and potentially can result in substantial complications.1 Glomus tumors generally are benign neoplasms of the glomus apparatus, and glomus cells function as thermoregulators in the reticular dermis.2 Glomus tumors comprise less than 2% of soft tissue neoplasms and generally are solitary nodules; only 10% of glomus tumors occur with multiple lesions, and among them, glomangiomyoma is the rarest subtype, presenting in only 15% of cases.2,3 The 3 main subtypes of glomus tumors are solid, glomangioma, and glomangiomyoma.4 Clinically, the lesions may present as small blue nodules with associated pain and cold or pressure sensitivity. Although there appears to be variation of the nomenclature depending on the source in the literature, glomangiomas are characterized by their predominant vascular malformations on biopsy. Glomangiomyomas are a subset of glomus tumors with distinct smooth muscle differentiation.4 Given their pathologic presentation, our patient’s lesions were most consistent with the diagnosis of glomangiomyoma.

CT112004026_e_Fig2_ABC.jpg
%3Cp%3E%3Cstrong%3EFIGURE%202.%3C%2Fstrong%3E%20A%2C%20Immunohistochemistry%20revealed%20small%20cells%20that%20were%20decorated%20with%20smooth%20muscle%20actin%20(original%20magnification%20%C3%9750).%20B%2C%20Spindled%20cells%20were%20highlighted%20with%20desmin%20(original%20magnification%20%C3%97200).%20C%2C%20CD34%20highlighted%20the%20endothelial%20cells%20lining%20the%20spaces%20(original%20magnification%20%C3%9720).%20Reference%20bars%20indicate%20100%20%CE%BCm%2C%20100%20%CE%BCm%2C%20and%201%20mm%2C%20respectively.%3C%2Fp%3E

The small size of the lesions may result in difficulty establishing a clinical diagnosis, particularly if there is no hand involvement, where lesions most commonly occur.2 Therefore, histopathologic evaluation is essential and is the best initial step in evaluating glomangiomyomas.4 Biopsy is the most reliable means of confirming a diagnosis2,4,5; however, diagnostic imaging such as a computed tomography also should be performed if considering blue rubber bleb nevus syndrome due to the primary site of involvement. Surgical excision is the treatment of choice after confirming the diagnosis in most cases of symptomatic glomangiomyomas, particularly with painful lesions.6

Neurilemmomas (also known as schwannomas) are benign lesions that generally present as asymptomatic, soft, smooth nodules most often on the neck; however, they also may present on the flexor extremities or in internal organs. Although primarily asymptomatic, the tumors may be associated with pain and paresthesia as they enlarge and affect surrounding structures. Neurilemmomas may occur spontaneously or as part of a syndrome, such as neurofibromatosis type 2 or Carney complex.7

Hereditary hemorrhagic telangiectasia (formerly known as Osler-Weber-Rendu syndrome) is an autosomal-dominant disease that presents with arteriovenous malformations and telangiectases. Patients generally present in the third decade of life, with the main concern generally being epistaxis.8

Kaposi sarcoma is a viral infection secondary to human herpesvirus 8 that results in red-purple lesions commonly on mucocutaneous sites. Kaposi sarcoma can be AIDS associated and non-HIV associated. Although clinically indistinguishable, a few subtle histologic features can assist in differentiating the 2 etiologies. In addition to a potential history of immunodeficiency, evaluating for involvement of the lymphatic system, respiratory tract, or gastrointestinal tract can aid in differentiating this entity from glomus tumors.9

Leiomyomas are smooth muscle lesions divided into 3 subcategories: angioleiomyoma, piloleiomyoma, and genital leiomyoma. The clinical presentation and histopathology will vary depending on the subcategory. Although cutaneous leiomyomas are benign, further workup for piloleiomyoma may be required given the reported association with hereditary leiomyomatosis and renal cell cancer (Reed syndrome).10

Imaging can be helpful when the clinical diagnosis of a glomus tumor vs other painful neoplasms of the skin is unclear, such as in blue rubber bleb nevus syndrome, angioleiomyomas, neuromas, glomus tumors, leiomyomas, eccrine spiradenomas, congenital vascular malformations, schwannomas, or hemangiomas.4 Radiologic findings for glomus tumors may demonstrate cortical or cystic osseous defects. Magnetic resonance imaging and ultrasonography can help provide additional information on the lesion size and depth of involvement.1 Additionally, deeper glomangiomyomas have been associated with malignancy,2 potentially highlighting the benefit of early incorporation of imaging in the workup for this condition. Malignant transformation is rare and has been reported in less than 1% of cases.6

Treatment of glomus tumors predominantly is directed to the patient’s symptoms; asymptomatic lesions may be monitored.4 For symptomatic lesions, therapeutic options include wide local excision; sclerotherapy; and incorporation of various lasers, including Nd:YAG, CO2, and flashlamp tunable dye laser.4,5 One case report documented use of a PDL that successfully eliminated the pain associated with glomangiomyoma; however, the lesion in that report was not biopsy proven.11

Our case highlights the need to consider glomus tumors in patients presenting with multiple small nodules given the potential for misdiagnosis, impact on quality of life with associated psychological distress, and potential utility of incorporating PDL in treatment. Although our patient did not report clinical improvement in the appearance of the lesions with PDL therapy, additional treatment sessions may have helped,11 but he opted to discontinue. Follow-up for persistently symptomatic or changing lesions is necessary, given the minimal risk for malignant transformation.6

References
  1. Lee DY, Hwang SC, Jeong ST, et al. The value of diagnostic ultrasonography in the assessment of a glomus tumor of the subcutaneous layer of the forearm mimicking a hemangioma: a case report. J Med Case Rep. 2015;9:191. doi:10.1186/s13256-015-0672-y
  2. Li L, Bardsley V, Grainger A, et al. Extradigital glomangiomyoma of the forearm mimicking peripheral nerve sheath tumour and thrombosed varicose vein. BMJ Case Rep. 2021;14:E241221. doi: 10.1136 /bcr-2020-241221
  3. Calduch L, Monteagudo C, Martínez-Ruiz E, et al. Familial generalized multiple glomangiomyoma: report of a new family, with immunohistochemical and ultrastructural studies and review of the literature. Pediatr Dermatol. 2002;19:402-408. doi:10.1046/j.1525-1470.2002.00114.x
  4. Mohammadi O, Suarez M. Glomus cancer. StatPearls [Internet]. StatPearls Publishing; 2021.
  5. Maxey ML, Houghton CC, Mastriani KS, et al. Large prepatellar glomangioma: a case report [published online July 10, 2015]. Int J Surg Case Rep. 2015;14:80-84. doi:10.1016/j.ijscr.2015.07.002
  6. Brathwaite CD, Poppiti RJ Jr. Malignant glomus tumor. a case report of widespread metastases in a patient with multiple glomus body hamartomas. Am J Surg Pathol. 1996;20:233-238. doi:10.1097/00000478-199602000-00012
  7. Davis DD, Kane SM. Neurilemmoma. StatPearls [Internet]. StatPearls Publishing; 2022.
  8. Kühnel T, Wirsching K, Wohlgemuth W, et al. Hereditary hemorrhagic telangiectasia. Otolaryngol Clin North Am. 2018;51:237-254. doi:10.1016/j.otc.2017.09.017
  9. Radu O, Pantanowitz L. Kaposi sarcoma. Arch Pathol Lab Med. 2013;137:289-294. doi:10.5858/arpa.2012-0101-RS
  10. Bernett CN, Mammino JJ. Cutaneous leiomyomas. StatPearls [Internet]. StatPearls Publishing; 2022.
  11. Antony FC, Cliff S, Cowley N. Complete pain relief following treatment of a glomangiomyoma with the pulsed dye laser. Clin Exp Dermatol. 2003;28:617-619. doi:10.1046/j.1365-2230.2003.01403.x
References
  1. Lee DY, Hwang SC, Jeong ST, et al. The value of diagnostic ultrasonography in the assessment of a glomus tumor of the subcutaneous layer of the forearm mimicking a hemangioma: a case report. J Med Case Rep. 2015;9:191. doi:10.1186/s13256-015-0672-y
  2. Li L, Bardsley V, Grainger A, et al. Extradigital glomangiomyoma of the forearm mimicking peripheral nerve sheath tumour and thrombosed varicose vein. BMJ Case Rep. 2021;14:E241221. doi: 10.1136 /bcr-2020-241221
  3. Calduch L, Monteagudo C, Martínez-Ruiz E, et al. Familial generalized multiple glomangiomyoma: report of a new family, with immunohistochemical and ultrastructural studies and review of the literature. Pediatr Dermatol. 2002;19:402-408. doi:10.1046/j.1525-1470.2002.00114.x
  4. Mohammadi O, Suarez M. Glomus cancer. StatPearls [Internet]. StatPearls Publishing; 2021.
  5. Maxey ML, Houghton CC, Mastriani KS, et al. Large prepatellar glomangioma: a case report [published online July 10, 2015]. Int J Surg Case Rep. 2015;14:80-84. doi:10.1016/j.ijscr.2015.07.002
  6. Brathwaite CD, Poppiti RJ Jr. Malignant glomus tumor. a case report of widespread metastases in a patient with multiple glomus body hamartomas. Am J Surg Pathol. 1996;20:233-238. doi:10.1097/00000478-199602000-00012
  7. Davis DD, Kane SM. Neurilemmoma. StatPearls [Internet]. StatPearls Publishing; 2022.
  8. Kühnel T, Wirsching K, Wohlgemuth W, et al. Hereditary hemorrhagic telangiectasia. Otolaryngol Clin North Am. 2018;51:237-254. doi:10.1016/j.otc.2017.09.017
  9. Radu O, Pantanowitz L. Kaposi sarcoma. Arch Pathol Lab Med. 2013;137:289-294. doi:10.5858/arpa.2012-0101-RS
  10. Bernett CN, Mammino JJ. Cutaneous leiomyomas. StatPearls [Internet]. StatPearls Publishing; 2022.
  11. Antony FC, Cliff S, Cowley N. Complete pain relief following treatment of a glomangiomyoma with the pulsed dye laser. Clin Exp Dermatol. 2003;28:617-619. doi:10.1046/j.1365-2230.2003.01403.x
Issue
Cutis - 112(4)
Issue
Cutis - 112(4)
Page Number
E26-E28
Page Number
E26-E28
Publications
Publications
Topics
Article Type
Display Headline
Blue Nodules on the Forearms in an Active-Duty Military Servicemember
Display Headline
Blue Nodules on the Forearms in an Active-Duty Military Servicemember
Sections
Questionnaire Body

A 31-year-old active-duty military servicemember presented to the dermatology clinic for evaluation of 0.3- to 2-cm, tender, blue nodules on the wrists and forearms. The lesions first appeared on the right volar wrist secondary to a presumed injury sustained approximately 10 years prior to presentation and spread to the proximal forearm as well as the left wrist and forearm. He denied fevers, chills, chest pain, hematochezia, hematuria, or other skin findings. Physical examination revealed blue-violaceous, firm nodules on the right volar wrist and forearm that were tender to palpation. Blue-violaceous, papulonodular lesions on the left volar wrist and dorsal hand were not tender to palpation. A punch biopsy was performed.

Eubanks_Quiz.jpg

Disallow All Ads
Content Gating
No Gating (article Unlocked/Free)
Alternative CME
Disqus Comments
Default
Gate On Date
Wed, 10/25/2023 - 06:45
Un-Gate On Date
Wed, 10/25/2023 - 06:45
Use ProPublica
CFC Schedule Remove Status
Wed, 10/25/2023 - 06:45
Hide sidebar & use full width
render the right sidebar.
Conference Recap Checkbox
Not Conference Recap
Clinical Edge
Display the Slideshow in this Article
Medscape Article
Display survey writer
Reuters content
Disable Inline Native ads
WebMD Article
Article PDF Media
Image
Teambase ID
180025E0.SIG
Disable zoom
Off

Tender Nodular Lesions in the Axilla and Vulva

Article Type
Changed
Wed, 01/03/2024 - 13:58
Display Headline
Tender Nodular Lesions in the Axilla and Vulva

The Diagnosis: Cutaneous Langerhans Cell Histiocytosis

Histopathologic findings of the left axillary lesion included a diffuse infiltrate of irregular hematolymphoid cells with reniform nuclei that strongly and diffusely stained positively with CD1a and S-100 but were negative for CD138 and CD163 (Figure). Numerous eosinophils also were present. The surrounding lymphocytic infiltrate stained positively with CD45. Polymerase chain reaction of the vaginal lesion was negative for herpes simplex virus types 1 and 2. Biopsy of the vaginal lesion revealed a mildly acanthotic epidermis and an aggregation of epithelioid cells with reniform nuclei in the papillary dermis. Positron emission tomography revealed widely disseminated disease. Sequencing of the mitogen-activated protein kinase/extracellular signalregulated kinase pathway showed amplified expression of these genes but found no mutations. These results led to a diagnosis of cutaneous Langerhans cell histiocytosis (LCH) with a background of hidradenitis suppurativa (HS). Our patient has since initiated therapy with trametinib leading to disease improvement without known recurrence.

CT112004020_e_FigAB.jpg
%3Cp%3ECutaneous%20Langerhans%20cell%20histiocytosis.%20A%2C%20Histopathology%20revealed%20a%20diffuse%20dermal%20infiltrate%20of%20mononuclear%20cells%20with%20cleaved%20nuclei%20as%20well%20as%20scattered%20lymphocytes%20and%20eosinophils%20(H%26amp%3BE%2C%20original%20magnification%20%C3%97200).%20B%2C%20Lesional%20cells%20strongly%20and%20diffusely%20expressed%20CD1a%20(original%20magnification%20%C3%97200).%3C%2Fp%3E

Langerhans cell histiocytosis is a rare disease of clonal dendritic cells (Langerhans cells) that can present in any organ.1 Most LCH diagnoses are made in pediatric patients, most often presenting in the bones, with other presentations in the skin, hypophysis, liver, lymph nodes, lungs, and spleen occurring less commonly.2 Proto-oncogene BRAF V600E mutations are a common determinant of LCH, with half of cases linked with this mutation that leads to enhanced activation of the mitogen-activated protein kinase pathway, though other mutations have been reported.3,4 These genetic alterations suggest LCH is neoplastic in nature; however, this is controversial, as spontaneous regression among pulmonary LCH has been observed, pointing to a reactive inflammatory process.5 Cutaneous LCH can present as a distinct papular or nodular lesion or multiple lesions with possible ulceration, but it is rare that LCH first presents on the skin.2,6 There is a substantial association of cutaneous LCH with the development of systemically disseminated LCH as well as other blood tumors, such as myelomonocytic leukemia, histiocytic sarcoma, and multiple lymphomas; this association is thought to be due to the common origin of LCH and other blood diseases in the bone marrow.6

Histopathology of LCH shows a diffuse papillary dermal infiltrate of clonal proliferation of reniform or cleaved histiocytes.5 Epidermal ulceration and epidermotropism also are common. Neoplastic cells are found admixed with variable levels of eosinophils, lymphocytes, plasma cells, and neutrophils, though eosinophils typically are elevated. Immunohistochemistry characteristically shows the expression of CD1a, S-100, and/or CD207, and the absence of CD163 expression.

Treatment of LCH is primarily dependent on disease dissemination status, with splenic and hepatic involvement, genetic panel results, and central nervous system risk considered in the treatment plan.5 Langerhans cell histiocytosis localized to the skin may require follow-up and monitoring, as spontaneous regression of cutaneous LCH is common. However, topical steroids or psoralen and long-wave UV radiation are potential treatments. Physicians who diagnose unifocal cutaneous LCH should have high clinical suspicion of disseminated LCH, and laboratory and radiographic evaluation may be necessary to rule out systemic disease, as more than 40% of patients with cutaneous LCH have systemic disease upon full evaluation.7 With systemic involvement, systemic chemotherapy may reduce morbidity and mortality, but clinical response should be monitored after 6 weeks of treatment, as results are variably effective. Vinblastine is the most common chemotherapy regimen, with an 84% survival rate and 51.5% event-free survival rate after 8 years.8 Targeted therapy for common genetic mutations also is possible, as vemurafenib has been used to treat patients with the BRAF V600E mutation.

Due to the variable clinical presentation of cutaneous LCH, the lesions can mimic other common skin diseases such as eczema or seborrheic dermatitis.7 However, there are limited data on LCH presenting in infiltrative skin disease. Langerhans cell histiocytosis that was misdiagnosed as HS has been reported,9-11 but LCH presenting alongside long-standing HS is rare. Although LCH often mimics infiltrative skin diseases, its simultaneous presentation with a previously confirmed diagnosis of HS was notable in our patient.

In our patient, the differential diagnosis included HS, Actinomyces infection, lymphomatoid papulosis, and dermatofibrosarcoma protuberans. Cutaneous findings in HS include chronic acneform nodules with follicular plugging, ruptured ducts leading to epithelized sinuses, inflammation, and abscesses in the axillae or inguinal and perineal areas.11 Histopathology reveals follicular occlusion and hyperkeratinization, which cause destruction of the pilosebaceous glands. Hidradenitis suppurativa features on immunohistochemistry often are conflicting, but there consistently is co-localization of keratinocyte hyperplasia with CD3-, CD4-, CD8-, and CD68-positive staining of cells that produce tumor necrosis factor α, IL-12, IL-23, and IL-32, with CD1a staining variable.12 An infection with Actinomyces, a slow-progressing anaerobic or microaerophilic bacteria, may present in the skin with chronic suppurative inflammation on the neck, trunk, and abdomen. The classic presentation is subcutaneous nodules with localized infiltration of abscesses, fistulas, and draining sinuses.13 Morphologically, Actinomyces causes chronic granulomatous infection with 0.1- to 1-mm sulfur granules, which are seen as basophilic masses with eosinophilic terminal clubs on hematoxylin and eosin staining.14 Histopathology reveals grampositive filamentous Actinomyces bacteria that branch at the edge of the granules. Lymphomatoid papulosis, a nonaggressive T-cell lymphoma, presents as papulonodular and sometimes necrotic disseminated lesions that spontaneously can regress or can cause a higher risk for the development of more aggressive lymphomas.15 Histopathology shows consistently dense, dermal, lymphocytic infiltration. Immunohistochemistry is characterized by lymphocytes expressing CD30 of varying degrees: type A with many CD30 staining cells, type B presenting similar to mycosis fungoides with little CD30 staining, and type C with lymphocytic CD30-staining plaques. Dermatofibrosarcoma protuberans is a low-grade soft-tissue malignant tumor with extensive local infiltration characterized by asymptomatic plaques on the trunk and proximal extremities that are indurated and adhered to the skin.16 Histopathology shows extensive invasion into the adjacent tissue far from the original focus of the tumor.

References
  1. Girschikofsky M, Arico M, Castillo D, et al. Management of adult patients with Langerhans cell histiocytosis: recommendations from an expert panel on behalf of Euro-Histio-Net. Orphanet J Rare Dis. 2013;8:72. doi:10.1186/1750-1172-8-72
  2. Flores-Terry MA, Sanz-Trenado JL, García-Arpa M, et al. Cutaneous Langerhans cell histiocytosis presenting in adulthood. Actas Dermosifiliogr (Engl Ed). 2019;110:167-169. doi:10.1016/j .adengl.2018.12.005
  3. Emile J-F, Abla O, Fraitag S, et al. Revised classification of histiocytoses and neoplasms of the macrophage-dendritic cell lineages. Blood. 2016;127:2672-2681. doi:10.1182/blood-2016-01-690636
  4. Badalian-Very G, Vergilio J-A, Degar BA, et al. Recurrent BRAF mutations in Langerhans cell histiocytosis. Blood. 2010;116:1919-1923. doi:10.1182/blood-2010-04-279083
  5. Bohn OL, Teruya-Feldstein J, Sanchez-Sosa S. Skin biopsy diagnosis of Langerhans cell neoplasms. In: Fernando S, ed. Skin Biopsy: Diagnosis and Treatment [Internet]. InTechOpen; 2013. http://dx.doi .org/10.5772/55893
  6. Edelbroek JR, Vermeer MH, Jansen PM, et al. Langerhans cell histiocytosis first presenting in the skin in adults: frequent association with a second haematological malignancy. Br J Dermatol. 2012;167:1287-1294. doi:10.1111/j.1365-2133.2012.11169.x
  7. Simko SJ, Garmezy B, Abhyankar H, et al. Differentiating skin-limited and multisystem Langerhans cell histiocytosis. J Pediatr. 2014;165: 990-996. doi:10.1016/j.jpeds.2014.07.063
  8. Yag˘ ci B, Varan A, Cag˘ lar M, et al. Langerhans cell histiocytosis: retrospective analysis of 217 cases in a single center. Pediatr Hematol Oncol. 2008;25:399-408. doi:10.1080/08880010802107356
  9. Kalen JE, Shokeen D, Mislankar M, et al. Langerhans cell histiocytosis with clinical and histologic features of hidradenitis suppurativa: brief report and review. Am J Dermatopathol. 2018;40:502-505. doi:10.1097/dad.0000000000001005
  10. Chertoff J, Chung J, Ataya A. Adult Langerhans cell histiocytosis masquerading as hidradenitis suppurativa. Am J Respir Crit Care Med. 2017;195:E34-E36. doi:10.1164/rccm.201610-2082IM
  11. St. Claire K, Bunney R, Ashack KA, et al. Langerhans cell histiocytosis: a great imitator. Clin Dermatol. 2020;38:223-234. doi:10.1016/j.clindermatol.2019.10.007
  12. Frew JW, Hawkes JE, Krueger JG. A systematic review and critical evaluation of immunohistochemical associations in hidradenitis suppurativa. F1000Research. 2019;7:1923. doi:10.12688/f1000research.17268.2
  13. Robati RM, Niknezhad N, Bidari-Zerehpoush F, et al. Primary cutaneous actinomycosis along with the surgical scar on the hand [published online November 9, 2016]. Case Rep Infect Dis. doi:10.1155/2016/5943932
  14. Ferry T, Valour F, Karsenty J, et al. Actinomycosis: etiology, clinical features, diagnosis, treatment, and management. Infect Drug Res. 2014;2014:183-197. doi:10.2147/idr.s39601
  15. Willemze R, Jaffe ES, Burg G, et al. WHO-EORTC classification for cutaneous lymphomas. Blood. 2005;105:3768-3785. doi:10.1182 /blood-2004-09-3502
  16. Tsai Y, Lin P, Chew K, et al. Dermatofibrosarcoma protuberans in children and adolescents: clinical presentation, histology, treatment, and review of the literature. J Plast Reconstr Aesthet Surg. 2014;67:1222-1229. doi:10.1016/j.bjps.2014.05.03
Article PDF
Author and Disclosure Information

From The Ohio State University Wexner Medical Center, Columbus. Matthew Gallardo is from the College of Medicine. Drs. Chung and Kaffenberger are from the Department of Dermatology. Dr. Chung also is from the Department of Pathology.

The authors report no conflict of interest.

Correspondence: Benjamin Kaffenberger, MD, MS, The Ohio State University Wexner Medical Center, 1328 Dublin Rd #100, Columbus, OH 43215 (Benjamin.Kaffenberger@osumc.edu).

Issue
Cutis - 112(4)
Publications
Topics
Page Number
E20-E22
Sections
Author and Disclosure Information

From The Ohio State University Wexner Medical Center, Columbus. Matthew Gallardo is from the College of Medicine. Drs. Chung and Kaffenberger are from the Department of Dermatology. Dr. Chung also is from the Department of Pathology.

The authors report no conflict of interest.

Correspondence: Benjamin Kaffenberger, MD, MS, The Ohio State University Wexner Medical Center, 1328 Dublin Rd #100, Columbus, OH 43215 (Benjamin.Kaffenberger@osumc.edu).

Author and Disclosure Information

From The Ohio State University Wexner Medical Center, Columbus. Matthew Gallardo is from the College of Medicine. Drs. Chung and Kaffenberger are from the Department of Dermatology. Dr. Chung also is from the Department of Pathology.

The authors report no conflict of interest.

Correspondence: Benjamin Kaffenberger, MD, MS, The Ohio State University Wexner Medical Center, 1328 Dublin Rd #100, Columbus, OH 43215 (Benjamin.Kaffenberger@osumc.edu).

Article PDF
Article PDF
Related Articles

The Diagnosis: Cutaneous Langerhans Cell Histiocytosis

Histopathologic findings of the left axillary lesion included a diffuse infiltrate of irregular hematolymphoid cells with reniform nuclei that strongly and diffusely stained positively with CD1a and S-100 but were negative for CD138 and CD163 (Figure). Numerous eosinophils also were present. The surrounding lymphocytic infiltrate stained positively with CD45. Polymerase chain reaction of the vaginal lesion was negative for herpes simplex virus types 1 and 2. Biopsy of the vaginal lesion revealed a mildly acanthotic epidermis and an aggregation of epithelioid cells with reniform nuclei in the papillary dermis. Positron emission tomography revealed widely disseminated disease. Sequencing of the mitogen-activated protein kinase/extracellular signalregulated kinase pathway showed amplified expression of these genes but found no mutations. These results led to a diagnosis of cutaneous Langerhans cell histiocytosis (LCH) with a background of hidradenitis suppurativa (HS). Our patient has since initiated therapy with trametinib leading to disease improvement without known recurrence.

CT112004020_e_FigAB.jpg
%3Cp%3ECutaneous%20Langerhans%20cell%20histiocytosis.%20A%2C%20Histopathology%20revealed%20a%20diffuse%20dermal%20infiltrate%20of%20mononuclear%20cells%20with%20cleaved%20nuclei%20as%20well%20as%20scattered%20lymphocytes%20and%20eosinophils%20(H%26amp%3BE%2C%20original%20magnification%20%C3%97200).%20B%2C%20Lesional%20cells%20strongly%20and%20diffusely%20expressed%20CD1a%20(original%20magnification%20%C3%97200).%3C%2Fp%3E

Langerhans cell histiocytosis is a rare disease of clonal dendritic cells (Langerhans cells) that can present in any organ.1 Most LCH diagnoses are made in pediatric patients, most often presenting in the bones, with other presentations in the skin, hypophysis, liver, lymph nodes, lungs, and spleen occurring less commonly.2 Proto-oncogene BRAF V600E mutations are a common determinant of LCH, with half of cases linked with this mutation that leads to enhanced activation of the mitogen-activated protein kinase pathway, though other mutations have been reported.3,4 These genetic alterations suggest LCH is neoplastic in nature; however, this is controversial, as spontaneous regression among pulmonary LCH has been observed, pointing to a reactive inflammatory process.5 Cutaneous LCH can present as a distinct papular or nodular lesion or multiple lesions with possible ulceration, but it is rare that LCH first presents on the skin.2,6 There is a substantial association of cutaneous LCH with the development of systemically disseminated LCH as well as other blood tumors, such as myelomonocytic leukemia, histiocytic sarcoma, and multiple lymphomas; this association is thought to be due to the common origin of LCH and other blood diseases in the bone marrow.6

Histopathology of LCH shows a diffuse papillary dermal infiltrate of clonal proliferation of reniform or cleaved histiocytes.5 Epidermal ulceration and epidermotropism also are common. Neoplastic cells are found admixed with variable levels of eosinophils, lymphocytes, plasma cells, and neutrophils, though eosinophils typically are elevated. Immunohistochemistry characteristically shows the expression of CD1a, S-100, and/or CD207, and the absence of CD163 expression.

Treatment of LCH is primarily dependent on disease dissemination status, with splenic and hepatic involvement, genetic panel results, and central nervous system risk considered in the treatment plan.5 Langerhans cell histiocytosis localized to the skin may require follow-up and monitoring, as spontaneous regression of cutaneous LCH is common. However, topical steroids or psoralen and long-wave UV radiation are potential treatments. Physicians who diagnose unifocal cutaneous LCH should have high clinical suspicion of disseminated LCH, and laboratory and radiographic evaluation may be necessary to rule out systemic disease, as more than 40% of patients with cutaneous LCH have systemic disease upon full evaluation.7 With systemic involvement, systemic chemotherapy may reduce morbidity and mortality, but clinical response should be monitored after 6 weeks of treatment, as results are variably effective. Vinblastine is the most common chemotherapy regimen, with an 84% survival rate and 51.5% event-free survival rate after 8 years.8 Targeted therapy for common genetic mutations also is possible, as vemurafenib has been used to treat patients with the BRAF V600E mutation.

Due to the variable clinical presentation of cutaneous LCH, the lesions can mimic other common skin diseases such as eczema or seborrheic dermatitis.7 However, there are limited data on LCH presenting in infiltrative skin disease. Langerhans cell histiocytosis that was misdiagnosed as HS has been reported,9-11 but LCH presenting alongside long-standing HS is rare. Although LCH often mimics infiltrative skin diseases, its simultaneous presentation with a previously confirmed diagnosis of HS was notable in our patient.

In our patient, the differential diagnosis included HS, Actinomyces infection, lymphomatoid papulosis, and dermatofibrosarcoma protuberans. Cutaneous findings in HS include chronic acneform nodules with follicular plugging, ruptured ducts leading to epithelized sinuses, inflammation, and abscesses in the axillae or inguinal and perineal areas.11 Histopathology reveals follicular occlusion and hyperkeratinization, which cause destruction of the pilosebaceous glands. Hidradenitis suppurativa features on immunohistochemistry often are conflicting, but there consistently is co-localization of keratinocyte hyperplasia with CD3-, CD4-, CD8-, and CD68-positive staining of cells that produce tumor necrosis factor α, IL-12, IL-23, and IL-32, with CD1a staining variable.12 An infection with Actinomyces, a slow-progressing anaerobic or microaerophilic bacteria, may present in the skin with chronic suppurative inflammation on the neck, trunk, and abdomen. The classic presentation is subcutaneous nodules with localized infiltration of abscesses, fistulas, and draining sinuses.13 Morphologically, Actinomyces causes chronic granulomatous infection with 0.1- to 1-mm sulfur granules, which are seen as basophilic masses with eosinophilic terminal clubs on hematoxylin and eosin staining.14 Histopathology reveals grampositive filamentous Actinomyces bacteria that branch at the edge of the granules. Lymphomatoid papulosis, a nonaggressive T-cell lymphoma, presents as papulonodular and sometimes necrotic disseminated lesions that spontaneously can regress or can cause a higher risk for the development of more aggressive lymphomas.15 Histopathology shows consistently dense, dermal, lymphocytic infiltration. Immunohistochemistry is characterized by lymphocytes expressing CD30 of varying degrees: type A with many CD30 staining cells, type B presenting similar to mycosis fungoides with little CD30 staining, and type C with lymphocytic CD30-staining plaques. Dermatofibrosarcoma protuberans is a low-grade soft-tissue malignant tumor with extensive local infiltration characterized by asymptomatic plaques on the trunk and proximal extremities that are indurated and adhered to the skin.16 Histopathology shows extensive invasion into the adjacent tissue far from the original focus of the tumor.

The Diagnosis: Cutaneous Langerhans Cell Histiocytosis

Histopathologic findings of the left axillary lesion included a diffuse infiltrate of irregular hematolymphoid cells with reniform nuclei that strongly and diffusely stained positively with CD1a and S-100 but were negative for CD138 and CD163 (Figure). Numerous eosinophils also were present. The surrounding lymphocytic infiltrate stained positively with CD45. Polymerase chain reaction of the vaginal lesion was negative for herpes simplex virus types 1 and 2. Biopsy of the vaginal lesion revealed a mildly acanthotic epidermis and an aggregation of epithelioid cells with reniform nuclei in the papillary dermis. Positron emission tomography revealed widely disseminated disease. Sequencing of the mitogen-activated protein kinase/extracellular signalregulated kinase pathway showed amplified expression of these genes but found no mutations. These results led to a diagnosis of cutaneous Langerhans cell histiocytosis (LCH) with a background of hidradenitis suppurativa (HS). Our patient has since initiated therapy with trametinib leading to disease improvement without known recurrence.

CT112004020_e_FigAB.jpg
%3Cp%3ECutaneous%20Langerhans%20cell%20histiocytosis.%20A%2C%20Histopathology%20revealed%20a%20diffuse%20dermal%20infiltrate%20of%20mononuclear%20cells%20with%20cleaved%20nuclei%20as%20well%20as%20scattered%20lymphocytes%20and%20eosinophils%20(H%26amp%3BE%2C%20original%20magnification%20%C3%97200).%20B%2C%20Lesional%20cells%20strongly%20and%20diffusely%20expressed%20CD1a%20(original%20magnification%20%C3%97200).%3C%2Fp%3E

Langerhans cell histiocytosis is a rare disease of clonal dendritic cells (Langerhans cells) that can present in any organ.1 Most LCH diagnoses are made in pediatric patients, most often presenting in the bones, with other presentations in the skin, hypophysis, liver, lymph nodes, lungs, and spleen occurring less commonly.2 Proto-oncogene BRAF V600E mutations are a common determinant of LCH, with half of cases linked with this mutation that leads to enhanced activation of the mitogen-activated protein kinase pathway, though other mutations have been reported.3,4 These genetic alterations suggest LCH is neoplastic in nature; however, this is controversial, as spontaneous regression among pulmonary LCH has been observed, pointing to a reactive inflammatory process.5 Cutaneous LCH can present as a distinct papular or nodular lesion or multiple lesions with possible ulceration, but it is rare that LCH first presents on the skin.2,6 There is a substantial association of cutaneous LCH with the development of systemically disseminated LCH as well as other blood tumors, such as myelomonocytic leukemia, histiocytic sarcoma, and multiple lymphomas; this association is thought to be due to the common origin of LCH and other blood diseases in the bone marrow.6

Histopathology of LCH shows a diffuse papillary dermal infiltrate of clonal proliferation of reniform or cleaved histiocytes.5 Epidermal ulceration and epidermotropism also are common. Neoplastic cells are found admixed with variable levels of eosinophils, lymphocytes, plasma cells, and neutrophils, though eosinophils typically are elevated. Immunohistochemistry characteristically shows the expression of CD1a, S-100, and/or CD207, and the absence of CD163 expression.

Treatment of LCH is primarily dependent on disease dissemination status, with splenic and hepatic involvement, genetic panel results, and central nervous system risk considered in the treatment plan.5 Langerhans cell histiocytosis localized to the skin may require follow-up and monitoring, as spontaneous regression of cutaneous LCH is common. However, topical steroids or psoralen and long-wave UV radiation are potential treatments. Physicians who diagnose unifocal cutaneous LCH should have high clinical suspicion of disseminated LCH, and laboratory and radiographic evaluation may be necessary to rule out systemic disease, as more than 40% of patients with cutaneous LCH have systemic disease upon full evaluation.7 With systemic involvement, systemic chemotherapy may reduce morbidity and mortality, but clinical response should be monitored after 6 weeks of treatment, as results are variably effective. Vinblastine is the most common chemotherapy regimen, with an 84% survival rate and 51.5% event-free survival rate after 8 years.8 Targeted therapy for common genetic mutations also is possible, as vemurafenib has been used to treat patients with the BRAF V600E mutation.

Due to the variable clinical presentation of cutaneous LCH, the lesions can mimic other common skin diseases such as eczema or seborrheic dermatitis.7 However, there are limited data on LCH presenting in infiltrative skin disease. Langerhans cell histiocytosis that was misdiagnosed as HS has been reported,9-11 but LCH presenting alongside long-standing HS is rare. Although LCH often mimics infiltrative skin diseases, its simultaneous presentation with a previously confirmed diagnosis of HS was notable in our patient.

In our patient, the differential diagnosis included HS, Actinomyces infection, lymphomatoid papulosis, and dermatofibrosarcoma protuberans. Cutaneous findings in HS include chronic acneform nodules with follicular plugging, ruptured ducts leading to epithelized sinuses, inflammation, and abscesses in the axillae or inguinal and perineal areas.11 Histopathology reveals follicular occlusion and hyperkeratinization, which cause destruction of the pilosebaceous glands. Hidradenitis suppurativa features on immunohistochemistry often are conflicting, but there consistently is co-localization of keratinocyte hyperplasia with CD3-, CD4-, CD8-, and CD68-positive staining of cells that produce tumor necrosis factor α, IL-12, IL-23, and IL-32, with CD1a staining variable.12 An infection with Actinomyces, a slow-progressing anaerobic or microaerophilic bacteria, may present in the skin with chronic suppurative inflammation on the neck, trunk, and abdomen. The classic presentation is subcutaneous nodules with localized infiltration of abscesses, fistulas, and draining sinuses.13 Morphologically, Actinomyces causes chronic granulomatous infection with 0.1- to 1-mm sulfur granules, which are seen as basophilic masses with eosinophilic terminal clubs on hematoxylin and eosin staining.14 Histopathology reveals grampositive filamentous Actinomyces bacteria that branch at the edge of the granules. Lymphomatoid papulosis, a nonaggressive T-cell lymphoma, presents as papulonodular and sometimes necrotic disseminated lesions that spontaneously can regress or can cause a higher risk for the development of more aggressive lymphomas.15 Histopathology shows consistently dense, dermal, lymphocytic infiltration. Immunohistochemistry is characterized by lymphocytes expressing CD30 of varying degrees: type A with many CD30 staining cells, type B presenting similar to mycosis fungoides with little CD30 staining, and type C with lymphocytic CD30-staining plaques. Dermatofibrosarcoma protuberans is a low-grade soft-tissue malignant tumor with extensive local infiltration characterized by asymptomatic plaques on the trunk and proximal extremities that are indurated and adhered to the skin.16 Histopathology shows extensive invasion into the adjacent tissue far from the original focus of the tumor.

References
  1. Girschikofsky M, Arico M, Castillo D, et al. Management of adult patients with Langerhans cell histiocytosis: recommendations from an expert panel on behalf of Euro-Histio-Net. Orphanet J Rare Dis. 2013;8:72. doi:10.1186/1750-1172-8-72
  2. Flores-Terry MA, Sanz-Trenado JL, García-Arpa M, et al. Cutaneous Langerhans cell histiocytosis presenting in adulthood. Actas Dermosifiliogr (Engl Ed). 2019;110:167-169. doi:10.1016/j .adengl.2018.12.005
  3. Emile J-F, Abla O, Fraitag S, et al. Revised classification of histiocytoses and neoplasms of the macrophage-dendritic cell lineages. Blood. 2016;127:2672-2681. doi:10.1182/blood-2016-01-690636
  4. Badalian-Very G, Vergilio J-A, Degar BA, et al. Recurrent BRAF mutations in Langerhans cell histiocytosis. Blood. 2010;116:1919-1923. doi:10.1182/blood-2010-04-279083
  5. Bohn OL, Teruya-Feldstein J, Sanchez-Sosa S. Skin biopsy diagnosis of Langerhans cell neoplasms. In: Fernando S, ed. Skin Biopsy: Diagnosis and Treatment [Internet]. InTechOpen; 2013. http://dx.doi .org/10.5772/55893
  6. Edelbroek JR, Vermeer MH, Jansen PM, et al. Langerhans cell histiocytosis first presenting in the skin in adults: frequent association with a second haematological malignancy. Br J Dermatol. 2012;167:1287-1294. doi:10.1111/j.1365-2133.2012.11169.x
  7. Simko SJ, Garmezy B, Abhyankar H, et al. Differentiating skin-limited and multisystem Langerhans cell histiocytosis. J Pediatr. 2014;165: 990-996. doi:10.1016/j.jpeds.2014.07.063
  8. Yag˘ ci B, Varan A, Cag˘ lar M, et al. Langerhans cell histiocytosis: retrospective analysis of 217 cases in a single center. Pediatr Hematol Oncol. 2008;25:399-408. doi:10.1080/08880010802107356
  9. Kalen JE, Shokeen D, Mislankar M, et al. Langerhans cell histiocytosis with clinical and histologic features of hidradenitis suppurativa: brief report and review. Am J Dermatopathol. 2018;40:502-505. doi:10.1097/dad.0000000000001005
  10. Chertoff J, Chung J, Ataya A. Adult Langerhans cell histiocytosis masquerading as hidradenitis suppurativa. Am J Respir Crit Care Med. 2017;195:E34-E36. doi:10.1164/rccm.201610-2082IM
  11. St. Claire K, Bunney R, Ashack KA, et al. Langerhans cell histiocytosis: a great imitator. Clin Dermatol. 2020;38:223-234. doi:10.1016/j.clindermatol.2019.10.007
  12. Frew JW, Hawkes JE, Krueger JG. A systematic review and critical evaluation of immunohistochemical associations in hidradenitis suppurativa. F1000Research. 2019;7:1923. doi:10.12688/f1000research.17268.2
  13. Robati RM, Niknezhad N, Bidari-Zerehpoush F, et al. Primary cutaneous actinomycosis along with the surgical scar on the hand [published online November 9, 2016]. Case Rep Infect Dis. doi:10.1155/2016/5943932
  14. Ferry T, Valour F, Karsenty J, et al. Actinomycosis: etiology, clinical features, diagnosis, treatment, and management. Infect Drug Res. 2014;2014:183-197. doi:10.2147/idr.s39601
  15. Willemze R, Jaffe ES, Burg G, et al. WHO-EORTC classification for cutaneous lymphomas. Blood. 2005;105:3768-3785. doi:10.1182 /blood-2004-09-3502
  16. Tsai Y, Lin P, Chew K, et al. Dermatofibrosarcoma protuberans in children and adolescents: clinical presentation, histology, treatment, and review of the literature. J Plast Reconstr Aesthet Surg. 2014;67:1222-1229. doi:10.1016/j.bjps.2014.05.03
References
  1. Girschikofsky M, Arico M, Castillo D, et al. Management of adult patients with Langerhans cell histiocytosis: recommendations from an expert panel on behalf of Euro-Histio-Net. Orphanet J Rare Dis. 2013;8:72. doi:10.1186/1750-1172-8-72
  2. Flores-Terry MA, Sanz-Trenado JL, García-Arpa M, et al. Cutaneous Langerhans cell histiocytosis presenting in adulthood. Actas Dermosifiliogr (Engl Ed). 2019;110:167-169. doi:10.1016/j .adengl.2018.12.005
  3. Emile J-F, Abla O, Fraitag S, et al. Revised classification of histiocytoses and neoplasms of the macrophage-dendritic cell lineages. Blood. 2016;127:2672-2681. doi:10.1182/blood-2016-01-690636
  4. Badalian-Very G, Vergilio J-A, Degar BA, et al. Recurrent BRAF mutations in Langerhans cell histiocytosis. Blood. 2010;116:1919-1923. doi:10.1182/blood-2010-04-279083
  5. Bohn OL, Teruya-Feldstein J, Sanchez-Sosa S. Skin biopsy diagnosis of Langerhans cell neoplasms. In: Fernando S, ed. Skin Biopsy: Diagnosis and Treatment [Internet]. InTechOpen; 2013. http://dx.doi .org/10.5772/55893
  6. Edelbroek JR, Vermeer MH, Jansen PM, et al. Langerhans cell histiocytosis first presenting in the skin in adults: frequent association with a second haematological malignancy. Br J Dermatol. 2012;167:1287-1294. doi:10.1111/j.1365-2133.2012.11169.x
  7. Simko SJ, Garmezy B, Abhyankar H, et al. Differentiating skin-limited and multisystem Langerhans cell histiocytosis. J Pediatr. 2014;165: 990-996. doi:10.1016/j.jpeds.2014.07.063
  8. Yag˘ ci B, Varan A, Cag˘ lar M, et al. Langerhans cell histiocytosis: retrospective analysis of 217 cases in a single center. Pediatr Hematol Oncol. 2008;25:399-408. doi:10.1080/08880010802107356
  9. Kalen JE, Shokeen D, Mislankar M, et al. Langerhans cell histiocytosis with clinical and histologic features of hidradenitis suppurativa: brief report and review. Am J Dermatopathol. 2018;40:502-505. doi:10.1097/dad.0000000000001005
  10. Chertoff J, Chung J, Ataya A. Adult Langerhans cell histiocytosis masquerading as hidradenitis suppurativa. Am J Respir Crit Care Med. 2017;195:E34-E36. doi:10.1164/rccm.201610-2082IM
  11. St. Claire K, Bunney R, Ashack KA, et al. Langerhans cell histiocytosis: a great imitator. Clin Dermatol. 2020;38:223-234. doi:10.1016/j.clindermatol.2019.10.007
  12. Frew JW, Hawkes JE, Krueger JG. A systematic review and critical evaluation of immunohistochemical associations in hidradenitis suppurativa. F1000Research. 2019;7:1923. doi:10.12688/f1000research.17268.2
  13. Robati RM, Niknezhad N, Bidari-Zerehpoush F, et al. Primary cutaneous actinomycosis along with the surgical scar on the hand [published online November 9, 2016]. Case Rep Infect Dis. doi:10.1155/2016/5943932
  14. Ferry T, Valour F, Karsenty J, et al. Actinomycosis: etiology, clinical features, diagnosis, treatment, and management. Infect Drug Res. 2014;2014:183-197. doi:10.2147/idr.s39601
  15. Willemze R, Jaffe ES, Burg G, et al. WHO-EORTC classification for cutaneous lymphomas. Blood. 2005;105:3768-3785. doi:10.1182 /blood-2004-09-3502
  16. Tsai Y, Lin P, Chew K, et al. Dermatofibrosarcoma protuberans in children and adolescents: clinical presentation, histology, treatment, and review of the literature. J Plast Reconstr Aesthet Surg. 2014;67:1222-1229. doi:10.1016/j.bjps.2014.05.03
Issue
Cutis - 112(4)
Issue
Cutis - 112(4)
Page Number
E20-E22
Page Number
E20-E22
Publications
Publications
Topics
Article Type
Display Headline
Tender Nodular Lesions in the Axilla and Vulva
Display Headline
Tender Nodular Lesions in the Axilla and Vulva
Sections
Questionnaire Body

A 28-year-old woman presented with tender burning lesions of the left axillary and vaginal skin that had worsened over the last year. Her medical history was notable for hidradenitis suppurativa, which had been present since adolescence, as well as pulmonary Langerhans cell histiocytosis diagnosed 7 years prior to the current presentation after a spontaneous pneumothorax that eventually led to a pulmonary transplantation 3 years prior. The patient’s Langerhans cell histiocytosis was believed to have resolved without treatment after smoking cessation. Physical examination revealed nodular inflammation and scarring with deep undermining along the left axilla as well as swelling of the mons pubis with erosive skin lesions in the surrounding vaginal area. Bilateral cervical, axillary, inguinal, supraclavicular, and femoral lymph node chains were negative for adenopathy. A shave biopsy was performed on the axillary nodule.

Gallardo_Quiz.jpg

Disallow All Ads
Content Gating
No Gating (article Unlocked/Free)
Alternative CME
Disqus Comments
Default
Gate On Date
Tue, 10/17/2023 - 11:15
Un-Gate On Date
Tue, 10/17/2023 - 11:15
Use ProPublica
CFC Schedule Remove Status
Tue, 10/17/2023 - 11:15
Hide sidebar & use full width
render the right sidebar.
Conference Recap Checkbox
Not Conference Recap
Clinical Edge
Display the Slideshow in this Article
Medscape Article
Display survey writer
Reuters content
Disable Inline Native ads
WebMD Article
Article PDF Media
Image
Teambase ID
180025D1.SIG
Disable zoom
Off

Nonhealing Ulcer in a Patient With Crohn Disease

Article Type
Changed
Tue, 10/17/2023 - 11:22
Display Headline
Nonhealing Ulcer in a Patient With Crohn Disease

The Diagnosis: Mycobacterium abscessus Infection

Upon further testing, cultures were positive for Mycobacterium abscessus. Our patient was referred to infectious disease for co-management, and his treatment plan consisted of intravenous amikacin 885 mg 3 times weekly, intravenous imipenem 1 g twice daily, azithromycin 500 mg/d, and omadacycline 150 mg/d for at least 3 months. Magnetic resonance imaging findings were consistent with a combination of cellulitis and osteomyelitis, and our patient was referred to plastic surgery for debridement. He subsequently was lost to follow-up.

Mycobacterium abscessus is classified as both a nontuberculous and rapidly growing mycobacterium. Mycobacterium abscessus recently has emerged as a pathogen of increasing public health concern, especially due to its high rate of antibiotic resistance.1-5 It is highly prevalent in the environment, and infection has been reported from a wide variety of environmental sources.6-8 Immunocompromised individuals, such as our patient, undergoing anti–tumor necrosis factor therapy are at increased risk for infection from all Mycobacterium species.9-11 Recognizing these infections quickly is a priority for patient care, as M abscessus can lead to disseminated infection and high mortality rates.1

Histopathology of M abscessus consists of granulomatous inflammation with mixed granulomas12; however, these findings are not always appreciable, and staining does not always reveal visible organisms. In our patient, histopathology revealed patchy plasmalymphocytic infiltrates of the dermis and subcutaneous tissue, which are signs of generalized inflammation (Figure). Therefore, cultures positive for M abscessus are the gold standard for diagnosis and established the diagnosis in this case.

CT112004010_e_Fig_AB.jpg
%3Cp%3EA%20and%20B%2C%20Histopathology%20revealed%20patchy%20plasmalymphocytic%20infiltrates%20of%20the%20dermis%20and%20subcutaneous%20tissue%20(H%26amp%3BE%2C%20original%20magnifications%20%C3%9740%20and%20%C3%97200).%3C%2Fp%3E

The differential diagnoses for our patient’s ulceration included squamous cell carcinoma, pyoderma gangrenosum, aseptic abscess ulcer, and pyodermatitispyostomatitis vegetans. Immunosuppressive therapy is a risk factor for squamous cell carcinoma13,14; however, ulcerated squamous cell carcinoma typically presents with prominent everted edges with a necrotic tumor base.15 Biopsy reveals cells with abundant eosinophilic cytoplasm, large nuclei, and variable keratin pearls.16 Pyoderma gangrenosum is an inflammatory skin condition associated with Crohn disease and often is a diagnosis of exclusion characterized by neutrophilic infiltrates on biopsy.17-19 Aseptic abscess ulcers are characterized by neutrophil-filled lesions that respond to corticosteroids but not antibiotics.20 Pyodermatitis-pyostomatitis vegetans is a rare skin manifestation of inflammatory bowel disease associated with a pustular eruption of the skin and/or mouth. Histopathology reveals pustules within or below the epidermis with many eosinophils or neutrophils. Granulomas do not occur as in M abscessus.21

Treatment of M abscessus infection requires the coadministration of several antibiotics across multiple classes to ensure complete disease resolution. High rates of antibiotic resistance are characterized by at least partial resistance to almost every antibiotic; clarithromycin has near-complete efficacy, but resistant strains have started to emerge. Amikacin and cefoxitin are other antibiotics that have reported a resistance rate of less than 50%, but they are only effective 90% and 70% of the time, respectively.1,22 The antibiotic omadacycline, which is approved by the US Food and Drug Administration to treat acute bacterial skin and soft-tissue infections, also may have utility in treating M abscessus infections.23,24 Finally, phage therapy may offer a potential mode of treatment for this bacterium and was used to treat pulmonary infection in a patient with cystic fibrosis.25 Despite these newer innovations, the current standard of care involves clarithromycin or azithromycin in combination with a parenteral antibiotic such as cefoxitin, amikacin, or imipenem for at least 4 months.1

References
  1. Griffith DE, Aksamit T, Brown-Elliott BA, et al. An official ATS/IDSA statement: diagnosis, treatment, and prevention of nontuberculous mycobacterial diseases. Am J Respir Crit Care Med. 2007;175:367-416.
  2. Jeong SH, Kim SY, Huh HJ, et al. Mycobacteriological characteristics and treatment outcomes in extrapulmonary Mycobacterium abscessus complex infections. Int J Infect Dis. 2017;60:49-56.
  3. Strnad L, Winthrop KL. Treatment of Mycobacterium abscessus complex. Semin Respir Crit Care Med. 2018;39:362-376.
  4. Cardenas DD, Yasmin T, Ahmed S. A rare insidious case of skin and soft tissue infection due to Mycobacterium abscessus: a case report. Cureus. 2022;14:E25725.
  5. Gonzalez-Santiago TM, Drage LA. Nontuberculous mycobacteria: skin and soft tissue infections. Dermatol Clin. 2015;33:563-577.
  6. Dickison P, Howard V, O’Kane G, et al. Mycobacterium abscessus infection following penetrations through wetsuits. Australas J Dermatol. 2019;60:57-59.
  7. Choi H, Kim YI, Na CH, et al. Mycobacterium abscessus skin infection associated with shaving activity in a 75-year-old man. Ann Geriatr Med Res. 2018;22:204.
  8. Costa-Silva M, Cesar A, Gomes NP, et al. Mycobacterium abscessus infection in a spa worker. Acta Dermatovenerol Alp Pannonica Adriat. 2018;27:159-161.
  9. Besada E. Rapid growing mycobacteria and TNF-α blockers: case report of a fatal lung infection with Mycobacterium abscessus. Clin Exp Rheumatol. 2011;29:705-707.
  10. Mufti AH, Toye BW, Mckendry RR, et al. Mycobacterium abscessus infection after use of tumor necrosis factor α inhibitor therapy: case report and review of infectious complications associated with tumor necrosis factor α inhibitor use. Diagn Microbiol Infect Dis. 2005;53:233-238.
  11. Lee SK, Kim SY, Kim EY, et al. Mycobacterial infections in patients treated with tumor necrosis factor antagonists in South Korea. Lung. 2013;191:565-571.
  12. Rodríguez G, Ortegón M, Camargo D, et al. Iatrogenic Mycobacterium abscessus infection: histopathology of 71 patients. Br J Dermatol. 1997;137:214-218.
  13. Firnhaber JM. Diagnosis and treatment of basal cell and squamous cell carcinoma. Am Fam Physician. 2012;86:161-168.
  14. Walker HS, Hardwicke J. Non-melanoma skin cancer. Surgery (Oxford). 2022;40:39-45.
  15. Browse NL. The skin. In: Browse NL, ed. An Introduction to the Symptoms and Signs of Surgical Disease. 3rd ed. London Arnold Publications; 2001:66-69.
  16. Weedon D. Squamous cell carcinoma. Weedon’s Skin Pathology. 3rd ed. Churchill Livingstone Elsevier; 2010;691-700.
  17. Powell F, Schroeter A, Su W, et al. Pyoderma gangrenosum: a review of 86 patients. QJM Int J Med. 1985;55:173-186.
  18. Brunsting LA, Goeckerman WH, O’Leary PA. Pyoderma (ecthyma) gangrenosum: clinical and experimental observations in five cases occurring in adults. Arch Dermatol. 1982;118:743-768.
  19. Maverakis E, Ma C, Shinkai K, et al. Diagnostic criteria of ulcerative pyoderma gangrenosum: a Delphi consensus of international experts. JAMA Dermatol. 2018;154:461-466.
  20. André MFJ, Piette JC, Kémény JL, et al. Aseptic abscesses: a study of 30 patients with or without inflammatory bowel disease and review of the literature. Medicine (Baltimore). 2007;86:145. doi:10.1097/md.0b013e18064f9f3
  21. Femiano F, Lanza A, Buonaiuto C, et al. Pyostomatitis vegetans: a review of the literature. Med Oral Patol Oral Cir Bucal. 2009;14:E114-E117.
  22. Kasperbauer SH, De Groote MA. The treatment of rapidly growing mycobacterial infections. Clin Chest Med. 2015;36:67-78.
  23. Duah M, Beshay M. Omadacycline in first-line combination therapy for pulmonary Mycobacterium abscessus infection: a case series. Int J Infect Dis. 2022;122:953-956.
  24. Minhas R, Sharma S, Kundu S. Utilizing the promise of omadacycline in a resistant, non-tubercular mycobacterial pulmonary infection. Cureus. 2019;11:E5112.
  25. Dedrick RM, Guerrero-Bustamante CA, Garlena RA, et al. Engineered bacteriophages for treatment of a patient with a disseminated drug-resistant Mycobacterium abscessus. Nat Med. 2019;25:730-733.
Article PDF
Author and Disclosure Information

Drs. Javdan and Wassef and Siddharth Marwaha are from the Department of Dermatology, Rutgers Robert Wood Johnson Medical School, Somerset, New Jersey. Dr. Ali is from the Department of Pathology, Rutgers Robert Wood Johnson Medical School, New Brunswick, New Jersey.

The authors report no conflict of interest.

Correspondence: Bahar Javdan, PhD, Rutgers Robert Wood Johnson Medical School, Rutgers Center for Dermatology, 1 Worlds Fair Dr, Somerset, NJ 08873 (bj186@rwjms.rutgers.edu).

Issue
Cutis - 112(4)
Publications
Topics
Page Number
E10-E12
Sections
Author and Disclosure Information

Drs. Javdan and Wassef and Siddharth Marwaha are from the Department of Dermatology, Rutgers Robert Wood Johnson Medical School, Somerset, New Jersey. Dr. Ali is from the Department of Pathology, Rutgers Robert Wood Johnson Medical School, New Brunswick, New Jersey.

The authors report no conflict of interest.

Correspondence: Bahar Javdan, PhD, Rutgers Robert Wood Johnson Medical School, Rutgers Center for Dermatology, 1 Worlds Fair Dr, Somerset, NJ 08873 (bj186@rwjms.rutgers.edu).

Author and Disclosure Information

Drs. Javdan and Wassef and Siddharth Marwaha are from the Department of Dermatology, Rutgers Robert Wood Johnson Medical School, Somerset, New Jersey. Dr. Ali is from the Department of Pathology, Rutgers Robert Wood Johnson Medical School, New Brunswick, New Jersey.

The authors report no conflict of interest.

Correspondence: Bahar Javdan, PhD, Rutgers Robert Wood Johnson Medical School, Rutgers Center for Dermatology, 1 Worlds Fair Dr, Somerset, NJ 08873 (bj186@rwjms.rutgers.edu).

Article PDF
Article PDF
Related Articles

The Diagnosis: Mycobacterium abscessus Infection

Upon further testing, cultures were positive for Mycobacterium abscessus. Our patient was referred to infectious disease for co-management, and his treatment plan consisted of intravenous amikacin 885 mg 3 times weekly, intravenous imipenem 1 g twice daily, azithromycin 500 mg/d, and omadacycline 150 mg/d for at least 3 months. Magnetic resonance imaging findings were consistent with a combination of cellulitis and osteomyelitis, and our patient was referred to plastic surgery for debridement. He subsequently was lost to follow-up.

Mycobacterium abscessus is classified as both a nontuberculous and rapidly growing mycobacterium. Mycobacterium abscessus recently has emerged as a pathogen of increasing public health concern, especially due to its high rate of antibiotic resistance.1-5 It is highly prevalent in the environment, and infection has been reported from a wide variety of environmental sources.6-8 Immunocompromised individuals, such as our patient, undergoing anti–tumor necrosis factor therapy are at increased risk for infection from all Mycobacterium species.9-11 Recognizing these infections quickly is a priority for patient care, as M abscessus can lead to disseminated infection and high mortality rates.1

Histopathology of M abscessus consists of granulomatous inflammation with mixed granulomas12; however, these findings are not always appreciable, and staining does not always reveal visible organisms. In our patient, histopathology revealed patchy plasmalymphocytic infiltrates of the dermis and subcutaneous tissue, which are signs of generalized inflammation (Figure). Therefore, cultures positive for M abscessus are the gold standard for diagnosis and established the diagnosis in this case.

CT112004010_e_Fig_AB.jpg
%3Cp%3EA%20and%20B%2C%20Histopathology%20revealed%20patchy%20plasmalymphocytic%20infiltrates%20of%20the%20dermis%20and%20subcutaneous%20tissue%20(H%26amp%3BE%2C%20original%20magnifications%20%C3%9740%20and%20%C3%97200).%3C%2Fp%3E

The differential diagnoses for our patient’s ulceration included squamous cell carcinoma, pyoderma gangrenosum, aseptic abscess ulcer, and pyodermatitispyostomatitis vegetans. Immunosuppressive therapy is a risk factor for squamous cell carcinoma13,14; however, ulcerated squamous cell carcinoma typically presents with prominent everted edges with a necrotic tumor base.15 Biopsy reveals cells with abundant eosinophilic cytoplasm, large nuclei, and variable keratin pearls.16 Pyoderma gangrenosum is an inflammatory skin condition associated with Crohn disease and often is a diagnosis of exclusion characterized by neutrophilic infiltrates on biopsy.17-19 Aseptic abscess ulcers are characterized by neutrophil-filled lesions that respond to corticosteroids but not antibiotics.20 Pyodermatitis-pyostomatitis vegetans is a rare skin manifestation of inflammatory bowel disease associated with a pustular eruption of the skin and/or mouth. Histopathology reveals pustules within or below the epidermis with many eosinophils or neutrophils. Granulomas do not occur as in M abscessus.21

Treatment of M abscessus infection requires the coadministration of several antibiotics across multiple classes to ensure complete disease resolution. High rates of antibiotic resistance are characterized by at least partial resistance to almost every antibiotic; clarithromycin has near-complete efficacy, but resistant strains have started to emerge. Amikacin and cefoxitin are other antibiotics that have reported a resistance rate of less than 50%, but they are only effective 90% and 70% of the time, respectively.1,22 The antibiotic omadacycline, which is approved by the US Food and Drug Administration to treat acute bacterial skin and soft-tissue infections, also may have utility in treating M abscessus infections.23,24 Finally, phage therapy may offer a potential mode of treatment for this bacterium and was used to treat pulmonary infection in a patient with cystic fibrosis.25 Despite these newer innovations, the current standard of care involves clarithromycin or azithromycin in combination with a parenteral antibiotic such as cefoxitin, amikacin, or imipenem for at least 4 months.1

The Diagnosis: Mycobacterium abscessus Infection

Upon further testing, cultures were positive for Mycobacterium abscessus. Our patient was referred to infectious disease for co-management, and his treatment plan consisted of intravenous amikacin 885 mg 3 times weekly, intravenous imipenem 1 g twice daily, azithromycin 500 mg/d, and omadacycline 150 mg/d for at least 3 months. Magnetic resonance imaging findings were consistent with a combination of cellulitis and osteomyelitis, and our patient was referred to plastic surgery for debridement. He subsequently was lost to follow-up.

Mycobacterium abscessus is classified as both a nontuberculous and rapidly growing mycobacterium. Mycobacterium abscessus recently has emerged as a pathogen of increasing public health concern, especially due to its high rate of antibiotic resistance.1-5 It is highly prevalent in the environment, and infection has been reported from a wide variety of environmental sources.6-8 Immunocompromised individuals, such as our patient, undergoing anti–tumor necrosis factor therapy are at increased risk for infection from all Mycobacterium species.9-11 Recognizing these infections quickly is a priority for patient care, as M abscessus can lead to disseminated infection and high mortality rates.1

Histopathology of M abscessus consists of granulomatous inflammation with mixed granulomas12; however, these findings are not always appreciable, and staining does not always reveal visible organisms. In our patient, histopathology revealed patchy plasmalymphocytic infiltrates of the dermis and subcutaneous tissue, which are signs of generalized inflammation (Figure). Therefore, cultures positive for M abscessus are the gold standard for diagnosis and established the diagnosis in this case.

CT112004010_e_Fig_AB.jpg
%3Cp%3EA%20and%20B%2C%20Histopathology%20revealed%20patchy%20plasmalymphocytic%20infiltrates%20of%20the%20dermis%20and%20subcutaneous%20tissue%20(H%26amp%3BE%2C%20original%20magnifications%20%C3%9740%20and%20%C3%97200).%3C%2Fp%3E

The differential diagnoses for our patient’s ulceration included squamous cell carcinoma, pyoderma gangrenosum, aseptic abscess ulcer, and pyodermatitispyostomatitis vegetans. Immunosuppressive therapy is a risk factor for squamous cell carcinoma13,14; however, ulcerated squamous cell carcinoma typically presents with prominent everted edges with a necrotic tumor base.15 Biopsy reveals cells with abundant eosinophilic cytoplasm, large nuclei, and variable keratin pearls.16 Pyoderma gangrenosum is an inflammatory skin condition associated with Crohn disease and often is a diagnosis of exclusion characterized by neutrophilic infiltrates on biopsy.17-19 Aseptic abscess ulcers are characterized by neutrophil-filled lesions that respond to corticosteroids but not antibiotics.20 Pyodermatitis-pyostomatitis vegetans is a rare skin manifestation of inflammatory bowel disease associated with a pustular eruption of the skin and/or mouth. Histopathology reveals pustules within or below the epidermis with many eosinophils or neutrophils. Granulomas do not occur as in M abscessus.21

Treatment of M abscessus infection requires the coadministration of several antibiotics across multiple classes to ensure complete disease resolution. High rates of antibiotic resistance are characterized by at least partial resistance to almost every antibiotic; clarithromycin has near-complete efficacy, but resistant strains have started to emerge. Amikacin and cefoxitin are other antibiotics that have reported a resistance rate of less than 50%, but they are only effective 90% and 70% of the time, respectively.1,22 The antibiotic omadacycline, which is approved by the US Food and Drug Administration to treat acute bacterial skin and soft-tissue infections, also may have utility in treating M abscessus infections.23,24 Finally, phage therapy may offer a potential mode of treatment for this bacterium and was used to treat pulmonary infection in a patient with cystic fibrosis.25 Despite these newer innovations, the current standard of care involves clarithromycin or azithromycin in combination with a parenteral antibiotic such as cefoxitin, amikacin, or imipenem for at least 4 months.1

References
  1. Griffith DE, Aksamit T, Brown-Elliott BA, et al. An official ATS/IDSA statement: diagnosis, treatment, and prevention of nontuberculous mycobacterial diseases. Am J Respir Crit Care Med. 2007;175:367-416.
  2. Jeong SH, Kim SY, Huh HJ, et al. Mycobacteriological characteristics and treatment outcomes in extrapulmonary Mycobacterium abscessus complex infections. Int J Infect Dis. 2017;60:49-56.
  3. Strnad L, Winthrop KL. Treatment of Mycobacterium abscessus complex. Semin Respir Crit Care Med. 2018;39:362-376.
  4. Cardenas DD, Yasmin T, Ahmed S. A rare insidious case of skin and soft tissue infection due to Mycobacterium abscessus: a case report. Cureus. 2022;14:E25725.
  5. Gonzalez-Santiago TM, Drage LA. Nontuberculous mycobacteria: skin and soft tissue infections. Dermatol Clin. 2015;33:563-577.
  6. Dickison P, Howard V, O’Kane G, et al. Mycobacterium abscessus infection following penetrations through wetsuits. Australas J Dermatol. 2019;60:57-59.
  7. Choi H, Kim YI, Na CH, et al. Mycobacterium abscessus skin infection associated with shaving activity in a 75-year-old man. Ann Geriatr Med Res. 2018;22:204.
  8. Costa-Silva M, Cesar A, Gomes NP, et al. Mycobacterium abscessus infection in a spa worker. Acta Dermatovenerol Alp Pannonica Adriat. 2018;27:159-161.
  9. Besada E. Rapid growing mycobacteria and TNF-α blockers: case report of a fatal lung infection with Mycobacterium abscessus. Clin Exp Rheumatol. 2011;29:705-707.
  10. Mufti AH, Toye BW, Mckendry RR, et al. Mycobacterium abscessus infection after use of tumor necrosis factor α inhibitor therapy: case report and review of infectious complications associated with tumor necrosis factor α inhibitor use. Diagn Microbiol Infect Dis. 2005;53:233-238.
  11. Lee SK, Kim SY, Kim EY, et al. Mycobacterial infections in patients treated with tumor necrosis factor antagonists in South Korea. Lung. 2013;191:565-571.
  12. Rodríguez G, Ortegón M, Camargo D, et al. Iatrogenic Mycobacterium abscessus infection: histopathology of 71 patients. Br J Dermatol. 1997;137:214-218.
  13. Firnhaber JM. Diagnosis and treatment of basal cell and squamous cell carcinoma. Am Fam Physician. 2012;86:161-168.
  14. Walker HS, Hardwicke J. Non-melanoma skin cancer. Surgery (Oxford). 2022;40:39-45.
  15. Browse NL. The skin. In: Browse NL, ed. An Introduction to the Symptoms and Signs of Surgical Disease. 3rd ed. London Arnold Publications; 2001:66-69.
  16. Weedon D. Squamous cell carcinoma. Weedon’s Skin Pathology. 3rd ed. Churchill Livingstone Elsevier; 2010;691-700.
  17. Powell F, Schroeter A, Su W, et al. Pyoderma gangrenosum: a review of 86 patients. QJM Int J Med. 1985;55:173-186.
  18. Brunsting LA, Goeckerman WH, O’Leary PA. Pyoderma (ecthyma) gangrenosum: clinical and experimental observations in five cases occurring in adults. Arch Dermatol. 1982;118:743-768.
  19. Maverakis E, Ma C, Shinkai K, et al. Diagnostic criteria of ulcerative pyoderma gangrenosum: a Delphi consensus of international experts. JAMA Dermatol. 2018;154:461-466.
  20. André MFJ, Piette JC, Kémény JL, et al. Aseptic abscesses: a study of 30 patients with or without inflammatory bowel disease and review of the literature. Medicine (Baltimore). 2007;86:145. doi:10.1097/md.0b013e18064f9f3
  21. Femiano F, Lanza A, Buonaiuto C, et al. Pyostomatitis vegetans: a review of the literature. Med Oral Patol Oral Cir Bucal. 2009;14:E114-E117.
  22. Kasperbauer SH, De Groote MA. The treatment of rapidly growing mycobacterial infections. Clin Chest Med. 2015;36:67-78.
  23. Duah M, Beshay M. Omadacycline in first-line combination therapy for pulmonary Mycobacterium abscessus infection: a case series. Int J Infect Dis. 2022;122:953-956.
  24. Minhas R, Sharma S, Kundu S. Utilizing the promise of omadacycline in a resistant, non-tubercular mycobacterial pulmonary infection. Cureus. 2019;11:E5112.
  25. Dedrick RM, Guerrero-Bustamante CA, Garlena RA, et al. Engineered bacteriophages for treatment of a patient with a disseminated drug-resistant Mycobacterium abscessus. Nat Med. 2019;25:730-733.
References
  1. Griffith DE, Aksamit T, Brown-Elliott BA, et al. An official ATS/IDSA statement: diagnosis, treatment, and prevention of nontuberculous mycobacterial diseases. Am J Respir Crit Care Med. 2007;175:367-416.
  2. Jeong SH, Kim SY, Huh HJ, et al. Mycobacteriological characteristics and treatment outcomes in extrapulmonary Mycobacterium abscessus complex infections. Int J Infect Dis. 2017;60:49-56.
  3. Strnad L, Winthrop KL. Treatment of Mycobacterium abscessus complex. Semin Respir Crit Care Med. 2018;39:362-376.
  4. Cardenas DD, Yasmin T, Ahmed S. A rare insidious case of skin and soft tissue infection due to Mycobacterium abscessus: a case report. Cureus. 2022;14:E25725.
  5. Gonzalez-Santiago TM, Drage LA. Nontuberculous mycobacteria: skin and soft tissue infections. Dermatol Clin. 2015;33:563-577.
  6. Dickison P, Howard V, O’Kane G, et al. Mycobacterium abscessus infection following penetrations through wetsuits. Australas J Dermatol. 2019;60:57-59.
  7. Choi H, Kim YI, Na CH, et al. Mycobacterium abscessus skin infection associated with shaving activity in a 75-year-old man. Ann Geriatr Med Res. 2018;22:204.
  8. Costa-Silva M, Cesar A, Gomes NP, et al. Mycobacterium abscessus infection in a spa worker. Acta Dermatovenerol Alp Pannonica Adriat. 2018;27:159-161.
  9. Besada E. Rapid growing mycobacteria and TNF-α blockers: case report of a fatal lung infection with Mycobacterium abscessus. Clin Exp Rheumatol. 2011;29:705-707.
  10. Mufti AH, Toye BW, Mckendry RR, et al. Mycobacterium abscessus infection after use of tumor necrosis factor α inhibitor therapy: case report and review of infectious complications associated with tumor necrosis factor α inhibitor use. Diagn Microbiol Infect Dis. 2005;53:233-238.
  11. Lee SK, Kim SY, Kim EY, et al. Mycobacterial infections in patients treated with tumor necrosis factor antagonists in South Korea. Lung. 2013;191:565-571.
  12. Rodríguez G, Ortegón M, Camargo D, et al. Iatrogenic Mycobacterium abscessus infection: histopathology of 71 patients. Br J Dermatol. 1997;137:214-218.
  13. Firnhaber JM. Diagnosis and treatment of basal cell and squamous cell carcinoma. Am Fam Physician. 2012;86:161-168.
  14. Walker HS, Hardwicke J. Non-melanoma skin cancer. Surgery (Oxford). 2022;40:39-45.
  15. Browse NL. The skin. In: Browse NL, ed. An Introduction to the Symptoms and Signs of Surgical Disease. 3rd ed. London Arnold Publications; 2001:66-69.
  16. Weedon D. Squamous cell carcinoma. Weedon’s Skin Pathology. 3rd ed. Churchill Livingstone Elsevier; 2010;691-700.
  17. Powell F, Schroeter A, Su W, et al. Pyoderma gangrenosum: a review of 86 patients. QJM Int J Med. 1985;55:173-186.
  18. Brunsting LA, Goeckerman WH, O’Leary PA. Pyoderma (ecthyma) gangrenosum: clinical and experimental observations in five cases occurring in adults. Arch Dermatol. 1982;118:743-768.
  19. Maverakis E, Ma C, Shinkai K, et al. Diagnostic criteria of ulcerative pyoderma gangrenosum: a Delphi consensus of international experts. JAMA Dermatol. 2018;154:461-466.
  20. André MFJ, Piette JC, Kémény JL, et al. Aseptic abscesses: a study of 30 patients with or without inflammatory bowel disease and review of the literature. Medicine (Baltimore). 2007;86:145. doi:10.1097/md.0b013e18064f9f3
  21. Femiano F, Lanza A, Buonaiuto C, et al. Pyostomatitis vegetans: a review of the literature. Med Oral Patol Oral Cir Bucal. 2009;14:E114-E117.
  22. Kasperbauer SH, De Groote MA. The treatment of rapidly growing mycobacterial infections. Clin Chest Med. 2015;36:67-78.
  23. Duah M, Beshay M. Omadacycline in first-line combination therapy for pulmonary Mycobacterium abscessus infection: a case series. Int J Infect Dis. 2022;122:953-956.
  24. Minhas R, Sharma S, Kundu S. Utilizing the promise of omadacycline in a resistant, non-tubercular mycobacterial pulmonary infection. Cureus. 2019;11:E5112.
  25. Dedrick RM, Guerrero-Bustamante CA, Garlena RA, et al. Engineered bacteriophages for treatment of a patient with a disseminated drug-resistant Mycobacterium abscessus. Nat Med. 2019;25:730-733.
Issue
Cutis - 112(4)
Issue
Cutis - 112(4)
Page Number
E10-E12
Page Number
E10-E12
Publications
Publications
Topics
Article Type
Display Headline
Nonhealing Ulcer in a Patient With Crohn Disease
Display Headline
Nonhealing Ulcer in a Patient With Crohn Disease
Sections
Questionnaire Body

A 24-year-old man presented to our dermatology clinic with a painful lesion on the right buccal cheek of 4 months’ duration that had not changed in size or appearance. He had a history of Crohn disease that was being treated with 6-mercaptopurine and infliximab. He underwent jaw surgery 7 years prior for correction of an underbite, followed by subsequent surgery to remove the hardware 1 year after the initial procedure. He experienced recurring skin abscesses following the initial jaw surgery roughly once a year that were treated with bedside incision and drainage procedures in the emergency department followed by trimethoprim-sulfamethoxazole with complete resolution; however, treatment with mupirocin ointment 2%, trimethoprim-sulfamethoxazole, and azithromycin did not provide symptomatic relief or resolution for the current lesion. Physical examination revealed a 4-cm ulceration with actively draining serosanguineous discharge. Two punch biopsies were performed; 48-hour bacterial and fungal cultures, as well as Giemsa, acid-fast bacilli, and periodic acid–Schiff staining were negative.

Javdan_Quiz.jpg

Disallow All Ads
Content Gating
No Gating (article Unlocked/Free)
Alternative CME
Disqus Comments
Default
Gate On Date
Wed, 10/11/2023 - 09:15
Un-Gate On Date
Wed, 10/11/2023 - 09:15
Use ProPublica
CFC Schedule Remove Status
Wed, 10/11/2023 - 09:15
Hide sidebar & use full width
render the right sidebar.
Conference Recap Checkbox
Not Conference Recap
Clinical Edge
Display the Slideshow in this Article
Medscape Article
Display survey writer
Reuters content
Disable Inline Native ads
WebMD Article
Article PDF Media
Image
Teambase ID
180025CE.SIG
Disable zoom
Off

Verrucous Plaque on the Foot

Article Type
Changed
Tue, 01/09/2024 - 08:26
Display Headline
Verrucous Plaque on the Foot

The Diagnosis: Eccrine Poroma

Histopathology demonstrated epidermal thickening, epidermal protrusions, a well-defined mass of tumor cells that extended from the epidermis down to the dermis, and luminal structures. Poroid cells and ovoid nuclei with basophilic cytoplasm also were evident (Figure 1). Dermoscopy showed papillomatous growth, milky-red areas, and dotted vessels (Figure 2). Reflectance confocal microscopy (RCM) at the spinous layer showed hyporefractile, dark, roundish lumina surrounded by keratinocytes (Figure 3). Based on the histologic, dermoscopic, and RCM findings, our patient was diagnosed with eccrine poroma.

CT112004153_Fig1_AB.jpg
%3Cp%3E%3Cstrong%3EFIGURE%201.%3C%2Fstrong%3E%20A%2C%20Histopathology%20revealed%20epidermal%20thickening%2C%20a%20welldefined%20mass%20of%20tumor%20cells%20extending%20from%20the%20epidermis%20down%20into%20the%20dermis%2C%20and%20luminal%20structures%20(H%26amp%3BE%2C%20original%20magnification%20%C3%974).%20B%2C%20Poroid%20cells%20appeared%20as%20cuboidal%20keratinocytes%20and%20ovoid%20nuclei%20with%20basophilic%20cytoplasm%20(H%26amp%3BE%2C%20original%20magnification%20%C3%9720).%3C%2Fp%3E

Goldman et al1 first described poroma in 1956. Poromas, which include eccrine poroma, are a group of benign cutaneous neoplasms arising from the terminal eccrine or apocrine sweat gland ducts.2 Histologically, poroid cells appear as cuboidal keratinocytes with monomorphous ovoid nuclei and discrete nucleoli.3 They usually appear as nodules or plaques with colors varying from flesh colored to red, brown, or bluish, and they clinically mimic several benign and malignant skin tumors. The differential diagnosis may include keratoacanthoma, plantar wart, verrucous carcinoma, basal cell carcinoma, and squamous cell carcinoma. Poromas can be of eccrine or apocrine origin.4 They also belong to a broad group of neoplasms, including nodular hidradenomas, clear cell hidradenomas, hidroacanthoma simplex, dermal duct tumors, and hidradenomas.5 Four subtypes—poroma, poroid hidradenoma, hidroacanthoma simplex, and dermal duct tumor—have been documented.6 Because poromas have nonspecific and variable clinical presentations, they often are misdiagnosed as other skin neoplasms, and differentiation may be difficult. For example, some cases of poroma present with follicular, sebaceous, and/or apocrine differentiation, leading to difficulty in diagnosis.

Zhou_2.jpg
%3Cp%3E%3Cstrong%3EFIGURE%202.%3C%2Fstrong%3E%20Dermoscopy%20showed%20papillomatous%20growth%2C%20milky-red%20areas%2C%20and%20dotted%20vessels%20(original%20magnification%20%C3%9710).%3C%2Fp%3E

Characteristic features of eccrine poroma seen on dermoscopy and RCM have the potential to aid in the diagnosis compared to histopathology. Marchetti et al7 proposed 4 patterns of characteristic dermoscopic findings. Pattern 1 refers to the classic description with bleeding spots, a structureless yellow appearance, milkyred globules, and branched vessels. Patterns 2 and 3 simulate basal cell carcinoma, dermal nevus, or vascular tumors. Pattern 4 refers to tumors that are large in size and resemble keratinizing neoplasms.7 Brugués et al8 described poromas with the following RCM findings: an atypical honeycomb shape that was well separated from the normal epithelium, hyporefractile nests with atypical cells, lack of palisading, and dark holes. One study described RCM parameters as cords without palisading, dark holes, prominent vascularization, and abundant stroma—findings that were positively associated with poroma in a univariate analysis. These findings assist in distinguishing poromas from other conditions in the differential diagnosis.9

Zhou_3.jpg

There is a substantial overlap in clinical appearance with malignant conditions, including basal cell carcinoma, squamous cell carcinoma, cutaneous metastases, and Paget disease; therefore, the use of dermoscopy and RCM may be helpful in the diagnosis and recognition of specific features, as well as the corresponding patterns of poroma. Poromas commonly display vascularized features due to the variability of dermoscopic patterns of eccrine poroma, and further studies are required to establish the specificity of vascularized features.

Acral lesions are more likely to show the classic clinical features of erythema and exophytic growth. A case of a collision tumor with the verrucous changes of poroma, seborrheic keratosis, and viral wart has been described.10 The verrucous changes may lead to misdiagnosis as plantar warts or other neoplasms. Clinicians also should consider conditions that are induced by friction or trauma. In our patient, dermoscopy and RCM aided in the diagnosis of eccrine poroma due to the interference of prominent overlying verrucous changes.

Treatment of poroma is optional. Deeper lesions can be treated with surgical excision, and superficial lesions may be treated with electrosurgical destruction. Our patient was treated with surgical excision followed by repair of the surgical defect with a double V-Y flap.

References
  1. Goldman P, Pinkus H, Rogin JR. Eccrine poroma; tumors exhibiting features of the epidermal sweat duct unit. AMA Arch Derm. 1956; 74:511-521.
  2. Miller AC, Adjei S, Temiz LA, et al. Dermal duct tumor: a diagnostic dilemma [published online January 28, 2022]. Dermatopathology (Basel). 2022;9:36-47. doi:10.3390/dermatopathology9010007
  3. Ahmed Jan N, Masood S. Poroma. StatPearls [Internet]. StatPearls Publishing; 2022. https://www.ncbi.nlm.nih.gov/books/NBK560909/
  4. Casper DJ, Glass LF, Shenefelt PD. An unusually large eccrine poroma: a case report and review of the literature. Cutis. 2011; 88:227-229.
  5. Sawaya JL, Khachemoune A. Poroma: a review of eccrine, apocrine, and malignant forms. Int J Dermatol. 2014;53:1053-1061.
  6. Betti R, Bombonato C, Cerri A, et al. Unusual sites for poromas are not very unusual: a survey of 101 cases. Clin Exp Dermatol. 2014; 39:119-122.
  7. Marchetti MA, Marino ML, Virmani P, et al. Dermoscopic features and patterns of poromas: a multicenter observational case-control study conducted by the International Dermoscopy Society (IDS). J Eur Acad Dermatol Venereol. 2018;32:1263-1271.
  8. Brugués A, Gamboa M, Alós L, et al. The challenging diagnosis of eccrine poromas. J Am Acad Dermatol. 2016;74:E113-E115.
  9. Di Tullio F, Mandel VD, Ignazio S, et al. The role of reflectance confocal microscopy in the diagnosis of eccrine poroma: a retrospective casecontrol study. Exp Dermatol. 2022;31:1779-1790.
  10. Bloom BS, Kamino H, Hale CS, et al. Collision tumor of eccrine poroma, seborrheic keratosis, and a viral wart. Dermatol Online J. 2014;20:13030/qt8tm0r9b9.
Article PDF
Author and Disclosure Information

From the Wenzhou Hospital of Integrated Traditional Chinese and Western Medicine, Affiliated Hospital of Integrated Traditional Chinese and Western Medicine of Shanghai University, China.

The authors report no conflict of interest.

Correspondence: Wei Wang, MD, Wenzhou Hospital of Integrated Traditional Chinese and Western Medicine, Affiliated Hospital of Integrated Traditional Chinese and Western Medicine of Shanghai University, Wenzhou 325000, China (w2wangwei@126.com).

Issue
Cutis - 112(4)
Publications
Topics
Page Number
153,178-179
Sections
Author and Disclosure Information

From the Wenzhou Hospital of Integrated Traditional Chinese and Western Medicine, Affiliated Hospital of Integrated Traditional Chinese and Western Medicine of Shanghai University, China.

The authors report no conflict of interest.

Correspondence: Wei Wang, MD, Wenzhou Hospital of Integrated Traditional Chinese and Western Medicine, Affiliated Hospital of Integrated Traditional Chinese and Western Medicine of Shanghai University, Wenzhou 325000, China (w2wangwei@126.com).

Author and Disclosure Information

From the Wenzhou Hospital of Integrated Traditional Chinese and Western Medicine, Affiliated Hospital of Integrated Traditional Chinese and Western Medicine of Shanghai University, China.

The authors report no conflict of interest.

Correspondence: Wei Wang, MD, Wenzhou Hospital of Integrated Traditional Chinese and Western Medicine, Affiliated Hospital of Integrated Traditional Chinese and Western Medicine of Shanghai University, Wenzhou 325000, China (w2wangwei@126.com).

Article PDF
Article PDF
Related Articles

The Diagnosis: Eccrine Poroma

Histopathology demonstrated epidermal thickening, epidermal protrusions, a well-defined mass of tumor cells that extended from the epidermis down to the dermis, and luminal structures. Poroid cells and ovoid nuclei with basophilic cytoplasm also were evident (Figure 1). Dermoscopy showed papillomatous growth, milky-red areas, and dotted vessels (Figure 2). Reflectance confocal microscopy (RCM) at the spinous layer showed hyporefractile, dark, roundish lumina surrounded by keratinocytes (Figure 3). Based on the histologic, dermoscopic, and RCM findings, our patient was diagnosed with eccrine poroma.

CT112004153_Fig1_AB.jpg
%3Cp%3E%3Cstrong%3EFIGURE%201.%3C%2Fstrong%3E%20A%2C%20Histopathology%20revealed%20epidermal%20thickening%2C%20a%20welldefined%20mass%20of%20tumor%20cells%20extending%20from%20the%20epidermis%20down%20into%20the%20dermis%2C%20and%20luminal%20structures%20(H%26amp%3BE%2C%20original%20magnification%20%C3%974).%20B%2C%20Poroid%20cells%20appeared%20as%20cuboidal%20keratinocytes%20and%20ovoid%20nuclei%20with%20basophilic%20cytoplasm%20(H%26amp%3BE%2C%20original%20magnification%20%C3%9720).%3C%2Fp%3E

Goldman et al1 first described poroma in 1956. Poromas, which include eccrine poroma, are a group of benign cutaneous neoplasms arising from the terminal eccrine or apocrine sweat gland ducts.2 Histologically, poroid cells appear as cuboidal keratinocytes with monomorphous ovoid nuclei and discrete nucleoli.3 They usually appear as nodules or plaques with colors varying from flesh colored to red, brown, or bluish, and they clinically mimic several benign and malignant skin tumors. The differential diagnosis may include keratoacanthoma, plantar wart, verrucous carcinoma, basal cell carcinoma, and squamous cell carcinoma. Poromas can be of eccrine or apocrine origin.4 They also belong to a broad group of neoplasms, including nodular hidradenomas, clear cell hidradenomas, hidroacanthoma simplex, dermal duct tumors, and hidradenomas.5 Four subtypes—poroma, poroid hidradenoma, hidroacanthoma simplex, and dermal duct tumor—have been documented.6 Because poromas have nonspecific and variable clinical presentations, they often are misdiagnosed as other skin neoplasms, and differentiation may be difficult. For example, some cases of poroma present with follicular, sebaceous, and/or apocrine differentiation, leading to difficulty in diagnosis.

Zhou_2.jpg
%3Cp%3E%3Cstrong%3EFIGURE%202.%3C%2Fstrong%3E%20Dermoscopy%20showed%20papillomatous%20growth%2C%20milky-red%20areas%2C%20and%20dotted%20vessels%20(original%20magnification%20%C3%9710).%3C%2Fp%3E

Characteristic features of eccrine poroma seen on dermoscopy and RCM have the potential to aid in the diagnosis compared to histopathology. Marchetti et al7 proposed 4 patterns of characteristic dermoscopic findings. Pattern 1 refers to the classic description with bleeding spots, a structureless yellow appearance, milkyred globules, and branched vessels. Patterns 2 and 3 simulate basal cell carcinoma, dermal nevus, or vascular tumors. Pattern 4 refers to tumors that are large in size and resemble keratinizing neoplasms.7 Brugués et al8 described poromas with the following RCM findings: an atypical honeycomb shape that was well separated from the normal epithelium, hyporefractile nests with atypical cells, lack of palisading, and dark holes. One study described RCM parameters as cords without palisading, dark holes, prominent vascularization, and abundant stroma—findings that were positively associated with poroma in a univariate analysis. These findings assist in distinguishing poromas from other conditions in the differential diagnosis.9

Zhou_3.jpg

There is a substantial overlap in clinical appearance with malignant conditions, including basal cell carcinoma, squamous cell carcinoma, cutaneous metastases, and Paget disease; therefore, the use of dermoscopy and RCM may be helpful in the diagnosis and recognition of specific features, as well as the corresponding patterns of poroma. Poromas commonly display vascularized features due to the variability of dermoscopic patterns of eccrine poroma, and further studies are required to establish the specificity of vascularized features.

Acral lesions are more likely to show the classic clinical features of erythema and exophytic growth. A case of a collision tumor with the verrucous changes of poroma, seborrheic keratosis, and viral wart has been described.10 The verrucous changes may lead to misdiagnosis as plantar warts or other neoplasms. Clinicians also should consider conditions that are induced by friction or trauma. In our patient, dermoscopy and RCM aided in the diagnosis of eccrine poroma due to the interference of prominent overlying verrucous changes.

Treatment of poroma is optional. Deeper lesions can be treated with surgical excision, and superficial lesions may be treated with electrosurgical destruction. Our patient was treated with surgical excision followed by repair of the surgical defect with a double V-Y flap.

The Diagnosis: Eccrine Poroma

Histopathology demonstrated epidermal thickening, epidermal protrusions, a well-defined mass of tumor cells that extended from the epidermis down to the dermis, and luminal structures. Poroid cells and ovoid nuclei with basophilic cytoplasm also were evident (Figure 1). Dermoscopy showed papillomatous growth, milky-red areas, and dotted vessels (Figure 2). Reflectance confocal microscopy (RCM) at the spinous layer showed hyporefractile, dark, roundish lumina surrounded by keratinocytes (Figure 3). Based on the histologic, dermoscopic, and RCM findings, our patient was diagnosed with eccrine poroma.

CT112004153_Fig1_AB.jpg
%3Cp%3E%3Cstrong%3EFIGURE%201.%3C%2Fstrong%3E%20A%2C%20Histopathology%20revealed%20epidermal%20thickening%2C%20a%20welldefined%20mass%20of%20tumor%20cells%20extending%20from%20the%20epidermis%20down%20into%20the%20dermis%2C%20and%20luminal%20structures%20(H%26amp%3BE%2C%20original%20magnification%20%C3%974).%20B%2C%20Poroid%20cells%20appeared%20as%20cuboidal%20keratinocytes%20and%20ovoid%20nuclei%20with%20basophilic%20cytoplasm%20(H%26amp%3BE%2C%20original%20magnification%20%C3%9720).%3C%2Fp%3E

Goldman et al1 first described poroma in 1956. Poromas, which include eccrine poroma, are a group of benign cutaneous neoplasms arising from the terminal eccrine or apocrine sweat gland ducts.2 Histologically, poroid cells appear as cuboidal keratinocytes with monomorphous ovoid nuclei and discrete nucleoli.3 They usually appear as nodules or plaques with colors varying from flesh colored to red, brown, or bluish, and they clinically mimic several benign and malignant skin tumors. The differential diagnosis may include keratoacanthoma, plantar wart, verrucous carcinoma, basal cell carcinoma, and squamous cell carcinoma. Poromas can be of eccrine or apocrine origin.4 They also belong to a broad group of neoplasms, including nodular hidradenomas, clear cell hidradenomas, hidroacanthoma simplex, dermal duct tumors, and hidradenomas.5 Four subtypes—poroma, poroid hidradenoma, hidroacanthoma simplex, and dermal duct tumor—have been documented.6 Because poromas have nonspecific and variable clinical presentations, they often are misdiagnosed as other skin neoplasms, and differentiation may be difficult. For example, some cases of poroma present with follicular, sebaceous, and/or apocrine differentiation, leading to difficulty in diagnosis.

Zhou_2.jpg
%3Cp%3E%3Cstrong%3EFIGURE%202.%3C%2Fstrong%3E%20Dermoscopy%20showed%20papillomatous%20growth%2C%20milky-red%20areas%2C%20and%20dotted%20vessels%20(original%20magnification%20%C3%9710).%3C%2Fp%3E

Characteristic features of eccrine poroma seen on dermoscopy and RCM have the potential to aid in the diagnosis compared to histopathology. Marchetti et al7 proposed 4 patterns of characteristic dermoscopic findings. Pattern 1 refers to the classic description with bleeding spots, a structureless yellow appearance, milkyred globules, and branched vessels. Patterns 2 and 3 simulate basal cell carcinoma, dermal nevus, or vascular tumors. Pattern 4 refers to tumors that are large in size and resemble keratinizing neoplasms.7 Brugués et al8 described poromas with the following RCM findings: an atypical honeycomb shape that was well separated from the normal epithelium, hyporefractile nests with atypical cells, lack of palisading, and dark holes. One study described RCM parameters as cords without palisading, dark holes, prominent vascularization, and abundant stroma—findings that were positively associated with poroma in a univariate analysis. These findings assist in distinguishing poromas from other conditions in the differential diagnosis.9

Zhou_3.jpg

There is a substantial overlap in clinical appearance with malignant conditions, including basal cell carcinoma, squamous cell carcinoma, cutaneous metastases, and Paget disease; therefore, the use of dermoscopy and RCM may be helpful in the diagnosis and recognition of specific features, as well as the corresponding patterns of poroma. Poromas commonly display vascularized features due to the variability of dermoscopic patterns of eccrine poroma, and further studies are required to establish the specificity of vascularized features.

Acral lesions are more likely to show the classic clinical features of erythema and exophytic growth. A case of a collision tumor with the verrucous changes of poroma, seborrheic keratosis, and viral wart has been described.10 The verrucous changes may lead to misdiagnosis as plantar warts or other neoplasms. Clinicians also should consider conditions that are induced by friction or trauma. In our patient, dermoscopy and RCM aided in the diagnosis of eccrine poroma due to the interference of prominent overlying verrucous changes.

Treatment of poroma is optional. Deeper lesions can be treated with surgical excision, and superficial lesions may be treated with electrosurgical destruction. Our patient was treated with surgical excision followed by repair of the surgical defect with a double V-Y flap.

References
  1. Goldman P, Pinkus H, Rogin JR. Eccrine poroma; tumors exhibiting features of the epidermal sweat duct unit. AMA Arch Derm. 1956; 74:511-521.
  2. Miller AC, Adjei S, Temiz LA, et al. Dermal duct tumor: a diagnostic dilemma [published online January 28, 2022]. Dermatopathology (Basel). 2022;9:36-47. doi:10.3390/dermatopathology9010007
  3. Ahmed Jan N, Masood S. Poroma. StatPearls [Internet]. StatPearls Publishing; 2022. https://www.ncbi.nlm.nih.gov/books/NBK560909/
  4. Casper DJ, Glass LF, Shenefelt PD. An unusually large eccrine poroma: a case report and review of the literature. Cutis. 2011; 88:227-229.
  5. Sawaya JL, Khachemoune A. Poroma: a review of eccrine, apocrine, and malignant forms. Int J Dermatol. 2014;53:1053-1061.
  6. Betti R, Bombonato C, Cerri A, et al. Unusual sites for poromas are not very unusual: a survey of 101 cases. Clin Exp Dermatol. 2014; 39:119-122.
  7. Marchetti MA, Marino ML, Virmani P, et al. Dermoscopic features and patterns of poromas: a multicenter observational case-control study conducted by the International Dermoscopy Society (IDS). J Eur Acad Dermatol Venereol. 2018;32:1263-1271.
  8. Brugués A, Gamboa M, Alós L, et al. The challenging diagnosis of eccrine poromas. J Am Acad Dermatol. 2016;74:E113-E115.
  9. Di Tullio F, Mandel VD, Ignazio S, et al. The role of reflectance confocal microscopy in the diagnosis of eccrine poroma: a retrospective casecontrol study. Exp Dermatol. 2022;31:1779-1790.
  10. Bloom BS, Kamino H, Hale CS, et al. Collision tumor of eccrine poroma, seborrheic keratosis, and a viral wart. Dermatol Online J. 2014;20:13030/qt8tm0r9b9.
References
  1. Goldman P, Pinkus H, Rogin JR. Eccrine poroma; tumors exhibiting features of the epidermal sweat duct unit. AMA Arch Derm. 1956; 74:511-521.
  2. Miller AC, Adjei S, Temiz LA, et al. Dermal duct tumor: a diagnostic dilemma [published online January 28, 2022]. Dermatopathology (Basel). 2022;9:36-47. doi:10.3390/dermatopathology9010007
  3. Ahmed Jan N, Masood S. Poroma. StatPearls [Internet]. StatPearls Publishing; 2022. https://www.ncbi.nlm.nih.gov/books/NBK560909/
  4. Casper DJ, Glass LF, Shenefelt PD. An unusually large eccrine poroma: a case report and review of the literature. Cutis. 2011; 88:227-229.
  5. Sawaya JL, Khachemoune A. Poroma: a review of eccrine, apocrine, and malignant forms. Int J Dermatol. 2014;53:1053-1061.
  6. Betti R, Bombonato C, Cerri A, et al. Unusual sites for poromas are not very unusual: a survey of 101 cases. Clin Exp Dermatol. 2014; 39:119-122.
  7. Marchetti MA, Marino ML, Virmani P, et al. Dermoscopic features and patterns of poromas: a multicenter observational case-control study conducted by the International Dermoscopy Society (IDS). J Eur Acad Dermatol Venereol. 2018;32:1263-1271.
  8. Brugués A, Gamboa M, Alós L, et al. The challenging diagnosis of eccrine poromas. J Am Acad Dermatol. 2016;74:E113-E115.
  9. Di Tullio F, Mandel VD, Ignazio S, et al. The role of reflectance confocal microscopy in the diagnosis of eccrine poroma: a retrospective casecontrol study. Exp Dermatol. 2022;31:1779-1790.
  10. Bloom BS, Kamino H, Hale CS, et al. Collision tumor of eccrine poroma, seborrheic keratosis, and a viral wart. Dermatol Online J. 2014;20:13030/qt8tm0r9b9.
Issue
Cutis - 112(4)
Issue
Cutis - 112(4)
Page Number
153,178-179
Page Number
153,178-179
Publications
Publications
Topics
Article Type
Display Headline
Verrucous Plaque on the Foot
Display Headline
Verrucous Plaque on the Foot
Sections
Questionnaire Body

A 62-year-old man presented with an enlarging plaque on the foot of 3 years’ duration. He experienced minor pain while walking but reported no other symptoms. His family history was negative for similar anomalies, and his medical history was negative for the presence of malignant tumors. Physical examination revealed a 2-mm erythematous plaque on the plantar aspect of the right foot with prominent overlying verrucous changes and no ulceration or regional lymphadenopathy. Dermoscopy and reflectance confocal microscopy of the lesion were performed along with a histopathologic examination after complete surgical excision.

Zhou_Quiz.jpg

Disallow All Ads
Content Gating
No Gating (article Unlocked/Free)
Alternative CME
Disqus Comments
Default
Gate On Date
Tue, 10/03/2023 - 06:30
Un-Gate On Date
Tue, 10/03/2023 - 06:30
Use ProPublica
CFC Schedule Remove Status
Tue, 10/03/2023 - 06:30
Hide sidebar & use full width
render the right sidebar.
Conference Recap Checkbox
Not Conference Recap
Clinical Edge
Display the Slideshow in this Article
Medscape Article
Display survey writer
Reuters content
Disable Inline Native ads
WebMD Article
Article PDF Media
Image
Teambase ID
180025B1.SIG
Disable zoom
Off

Hypotrichosis and Hair Loss on the Occipital Scalp

Article Type
Changed
Wed, 09/27/2023 - 12:52
Display Headline
Hypotrichosis and Hair Loss on the Occipital Scalp

The Diagnosis: Monilethrix

A diagnosis of monilethrix was rendered based on the clinical and trichoscopic findings. Simple surveillance of the patient’s condition and prevention of further hair trauma were proposed as management options.

Monilethrix is a hair shaft disorder that is inherited in a predominantly autosomal-dominant pattern with variable expressiveness and penetrance resulting from heterozygous mutations in hair keratin genes KRT81, KRT83, and KRT86 in a region of chromosome 12q13.13.1,2 An autosomalrecessive form has been described with mutation in desmoglein 4, but it differs from the classical form by the variable periodicity of the region between the nodules.3,4

The morphologic alteration consists of the formation of fusiform nodules of normal structure alternated with narrow and dystrophic constrictions (Figure). These internodes are fragile areas that cause breakage at constricted points.5 Clinically, monilethrix presents as areas of focal or diffuse alopecia with frequent involvement of the terminal follicles, mainly in areas of friction. The hair is normal at birth due to the predominance of lanugo in the neonatal period, but it subsequently is replaced by abnormal hairs in the first months of life.6 Initial clinical signs begin to appear when the terminal hairs begin to form.7 Although rarer, the eyebrows and eyelashes, as well as the axillary, pubic, and body hair, may be involved.5

CT112003027_e_AB.jpg
%3Cp%3EA%20and%20B%2C%20Optical%20microscopy%20showed%20uniform%20elliptical%20nodes%20separated%20by%20intermittent%20constrictions%20and%20broken%20hair%20shafts%20at%20internode%20levels%2C%20respectively%20(original%20magnifications%20%C3%97100).%3C%2Fp%3E

Other hair shaft anomalies merit consideration in the differential diagnosis of monilethrix, including pseudomonilethrix, pressure alopecia, trichorrhexis invaginata, ectodermal dysplasia, tinea capitis, and trichothiodystrophy.6 The diagnosis is reached by clinical history and physical examination. Trichoscopy and light microscopy are used to confirm the diagnosis. Trichoscopic examination shows markedly higher rates of anagen hair. The shafts examined in our patient revealed 0.7- to 1-mm intervals between nodes. Hair can be better visualized under a polarized microscope, and the condition can be distinguished from pseudomonilethrix using this approach.5,6 In our patient, the diagnosis was made based on light microscopy and trichoscopic findings with no genetic testing; however, genetic testing for the classic mutations of the keratin genes would be desirable to confirm the diagnosis but was not done in our patient.6 The prognosis of monilethrix is variable; most cases persist into adulthood, though spontaneous improvement may occur with advancing age, during summer, and during pregnancy.8

There is no definitive therapy for monilethrix. Although there have been reports of cases treated with systemic corticosteroids, oral retinoids, topical minoxidil, vitamins, and peeling ointments (desquamative oil), the cornerstone of management is protecting the hair against traumatic procedures such as excessive combing, brushing, and friction, as well as parent and patient education about the benign nature of the condition.9 Additionally, some cases have shown improvement with minoxidil solution at 2% and 5% concentrations, oral minoxidil, or acitretin.7-9

References
  1. Fontenelle de Oliveira E, Cotta de Alencar Araripe AL. Monilethrix: a typical case report with microscopic and dermatoscopic findings. An Bras Dermatol. 2015;90:126-127.
  2. de Cruz R, Horev L, Green J, et al. A novel monilethrix mutation in coil 2A of KRT86 causing autosomal dominant monilethrix with incomplete penetrance. Br J Dermatol. 2012;166(suppl 2):20-26.
  3. Baltazard T, Dhaille F, Chaby G, et al. Value of dermoscopy for the diagnosis of monilethrix. Dermatol Online J. 2017;23:13030 /qt9hf1p3xm.
  4. Kato M, Shimizu A, Yokoyama Y, et al. An autosomal recessive mutation of DSG4 causes monilethrix through the ER stress response. J Invest Dermatol. 2015;135:1253-1260.
  5. Gummer CL, Dawber RP, Swift JA. Monilethrix: an electron microscopic and electron histochemical study. Br J Dermatol. 1981;105:529-541.
  6. Sharma VK, Chiramel MJ, Rao A. Dermoscopy: a rapid bedside tool to assess monilethrix. Indian J Dermatol Venereol Leprol. 2016;82:73-74.
  7. Sinclair R. Treatment of monilethrix with oral minoxidil. JAAD Case Rep. 2016;2:212-215.
  8. Rakowska A, Slowinska M, Czuwara J, et al. Dermoscopy as a tool for rapid diagnosis of monilethrix. J Drugs Dermatol. 2007;6:222-224.
  9. Karincaoglu Y, Coskun BK, Seyhan ME, et al. Monilethrix. Am J Clin Dermatol. 2005;6:407-410.
Article PDF
Author and Disclosure Information

From the School of Medicine, Dermatology Department, Federal University of Minas Gerais, Belo Horizonte, Brazil.

The authors report no conflict of interest.

Correspondence: Erica Possa de Abreu, MD, Tomé de Souza St, 950, Ap 905 - Savassi, 30140-36 Minas Gerais, Brazil (abreuerica20@gmail.com).

Issue
Cutis - 112(3)
Publications
Topics
Page Number
E27-E29
Sections
Author and Disclosure Information

From the School of Medicine, Dermatology Department, Federal University of Minas Gerais, Belo Horizonte, Brazil.

The authors report no conflict of interest.

Correspondence: Erica Possa de Abreu, MD, Tomé de Souza St, 950, Ap 905 - Savassi, 30140-36 Minas Gerais, Brazil (abreuerica20@gmail.com).

Author and Disclosure Information

From the School of Medicine, Dermatology Department, Federal University of Minas Gerais, Belo Horizonte, Brazil.

The authors report no conflict of interest.

Correspondence: Erica Possa de Abreu, MD, Tomé de Souza St, 950, Ap 905 - Savassi, 30140-36 Minas Gerais, Brazil (abreuerica20@gmail.com).

Article PDF
Article PDF
Related Articles

The Diagnosis: Monilethrix

A diagnosis of monilethrix was rendered based on the clinical and trichoscopic findings. Simple surveillance of the patient’s condition and prevention of further hair trauma were proposed as management options.

Monilethrix is a hair shaft disorder that is inherited in a predominantly autosomal-dominant pattern with variable expressiveness and penetrance resulting from heterozygous mutations in hair keratin genes KRT81, KRT83, and KRT86 in a region of chromosome 12q13.13.1,2 An autosomalrecessive form has been described with mutation in desmoglein 4, but it differs from the classical form by the variable periodicity of the region between the nodules.3,4

The morphologic alteration consists of the formation of fusiform nodules of normal structure alternated with narrow and dystrophic constrictions (Figure). These internodes are fragile areas that cause breakage at constricted points.5 Clinically, monilethrix presents as areas of focal or diffuse alopecia with frequent involvement of the terminal follicles, mainly in areas of friction. The hair is normal at birth due to the predominance of lanugo in the neonatal period, but it subsequently is replaced by abnormal hairs in the first months of life.6 Initial clinical signs begin to appear when the terminal hairs begin to form.7 Although rarer, the eyebrows and eyelashes, as well as the axillary, pubic, and body hair, may be involved.5

CT112003027_e_AB.jpg
%3Cp%3EA%20and%20B%2C%20Optical%20microscopy%20showed%20uniform%20elliptical%20nodes%20separated%20by%20intermittent%20constrictions%20and%20broken%20hair%20shafts%20at%20internode%20levels%2C%20respectively%20(original%20magnifications%20%C3%97100).%3C%2Fp%3E

Other hair shaft anomalies merit consideration in the differential diagnosis of monilethrix, including pseudomonilethrix, pressure alopecia, trichorrhexis invaginata, ectodermal dysplasia, tinea capitis, and trichothiodystrophy.6 The diagnosis is reached by clinical history and physical examination. Trichoscopy and light microscopy are used to confirm the diagnosis. Trichoscopic examination shows markedly higher rates of anagen hair. The shafts examined in our patient revealed 0.7- to 1-mm intervals between nodes. Hair can be better visualized under a polarized microscope, and the condition can be distinguished from pseudomonilethrix using this approach.5,6 In our patient, the diagnosis was made based on light microscopy and trichoscopic findings with no genetic testing; however, genetic testing for the classic mutations of the keratin genes would be desirable to confirm the diagnosis but was not done in our patient.6 The prognosis of monilethrix is variable; most cases persist into adulthood, though spontaneous improvement may occur with advancing age, during summer, and during pregnancy.8

There is no definitive therapy for monilethrix. Although there have been reports of cases treated with systemic corticosteroids, oral retinoids, topical minoxidil, vitamins, and peeling ointments (desquamative oil), the cornerstone of management is protecting the hair against traumatic procedures such as excessive combing, brushing, and friction, as well as parent and patient education about the benign nature of the condition.9 Additionally, some cases have shown improvement with minoxidil solution at 2% and 5% concentrations, oral minoxidil, or acitretin.7-9

The Diagnosis: Monilethrix

A diagnosis of monilethrix was rendered based on the clinical and trichoscopic findings. Simple surveillance of the patient’s condition and prevention of further hair trauma were proposed as management options.

Monilethrix is a hair shaft disorder that is inherited in a predominantly autosomal-dominant pattern with variable expressiveness and penetrance resulting from heterozygous mutations in hair keratin genes KRT81, KRT83, and KRT86 in a region of chromosome 12q13.13.1,2 An autosomalrecessive form has been described with mutation in desmoglein 4, but it differs from the classical form by the variable periodicity of the region between the nodules.3,4

The morphologic alteration consists of the formation of fusiform nodules of normal structure alternated with narrow and dystrophic constrictions (Figure). These internodes are fragile areas that cause breakage at constricted points.5 Clinically, monilethrix presents as areas of focal or diffuse alopecia with frequent involvement of the terminal follicles, mainly in areas of friction. The hair is normal at birth due to the predominance of lanugo in the neonatal period, but it subsequently is replaced by abnormal hairs in the first months of life.6 Initial clinical signs begin to appear when the terminal hairs begin to form.7 Although rarer, the eyebrows and eyelashes, as well as the axillary, pubic, and body hair, may be involved.5

CT112003027_e_AB.jpg
%3Cp%3EA%20and%20B%2C%20Optical%20microscopy%20showed%20uniform%20elliptical%20nodes%20separated%20by%20intermittent%20constrictions%20and%20broken%20hair%20shafts%20at%20internode%20levels%2C%20respectively%20(original%20magnifications%20%C3%97100).%3C%2Fp%3E

Other hair shaft anomalies merit consideration in the differential diagnosis of monilethrix, including pseudomonilethrix, pressure alopecia, trichorrhexis invaginata, ectodermal dysplasia, tinea capitis, and trichothiodystrophy.6 The diagnosis is reached by clinical history and physical examination. Trichoscopy and light microscopy are used to confirm the diagnosis. Trichoscopic examination shows markedly higher rates of anagen hair. The shafts examined in our patient revealed 0.7- to 1-mm intervals between nodes. Hair can be better visualized under a polarized microscope, and the condition can be distinguished from pseudomonilethrix using this approach.5,6 In our patient, the diagnosis was made based on light microscopy and trichoscopic findings with no genetic testing; however, genetic testing for the classic mutations of the keratin genes would be desirable to confirm the diagnosis but was not done in our patient.6 The prognosis of monilethrix is variable; most cases persist into adulthood, though spontaneous improvement may occur with advancing age, during summer, and during pregnancy.8

There is no definitive therapy for monilethrix. Although there have been reports of cases treated with systemic corticosteroids, oral retinoids, topical minoxidil, vitamins, and peeling ointments (desquamative oil), the cornerstone of management is protecting the hair against traumatic procedures such as excessive combing, brushing, and friction, as well as parent and patient education about the benign nature of the condition.9 Additionally, some cases have shown improvement with minoxidil solution at 2% and 5% concentrations, oral minoxidil, or acitretin.7-9

References
  1. Fontenelle de Oliveira E, Cotta de Alencar Araripe AL. Monilethrix: a typical case report with microscopic and dermatoscopic findings. An Bras Dermatol. 2015;90:126-127.
  2. de Cruz R, Horev L, Green J, et al. A novel monilethrix mutation in coil 2A of KRT86 causing autosomal dominant monilethrix with incomplete penetrance. Br J Dermatol. 2012;166(suppl 2):20-26.
  3. Baltazard T, Dhaille F, Chaby G, et al. Value of dermoscopy for the diagnosis of monilethrix. Dermatol Online J. 2017;23:13030 /qt9hf1p3xm.
  4. Kato M, Shimizu A, Yokoyama Y, et al. An autosomal recessive mutation of DSG4 causes monilethrix through the ER stress response. J Invest Dermatol. 2015;135:1253-1260.
  5. Gummer CL, Dawber RP, Swift JA. Monilethrix: an electron microscopic and electron histochemical study. Br J Dermatol. 1981;105:529-541.
  6. Sharma VK, Chiramel MJ, Rao A. Dermoscopy: a rapid bedside tool to assess monilethrix. Indian J Dermatol Venereol Leprol. 2016;82:73-74.
  7. Sinclair R. Treatment of monilethrix with oral minoxidil. JAAD Case Rep. 2016;2:212-215.
  8. Rakowska A, Slowinska M, Czuwara J, et al. Dermoscopy as a tool for rapid diagnosis of monilethrix. J Drugs Dermatol. 2007;6:222-224.
  9. Karincaoglu Y, Coskun BK, Seyhan ME, et al. Monilethrix. Am J Clin Dermatol. 2005;6:407-410.
References
  1. Fontenelle de Oliveira E, Cotta de Alencar Araripe AL. Monilethrix: a typical case report with microscopic and dermatoscopic findings. An Bras Dermatol. 2015;90:126-127.
  2. de Cruz R, Horev L, Green J, et al. A novel monilethrix mutation in coil 2A of KRT86 causing autosomal dominant monilethrix with incomplete penetrance. Br J Dermatol. 2012;166(suppl 2):20-26.
  3. Baltazard T, Dhaille F, Chaby G, et al. Value of dermoscopy for the diagnosis of monilethrix. Dermatol Online J. 2017;23:13030 /qt9hf1p3xm.
  4. Kato M, Shimizu A, Yokoyama Y, et al. An autosomal recessive mutation of DSG4 causes monilethrix through the ER stress response. J Invest Dermatol. 2015;135:1253-1260.
  5. Gummer CL, Dawber RP, Swift JA. Monilethrix: an electron microscopic and electron histochemical study. Br J Dermatol. 1981;105:529-541.
  6. Sharma VK, Chiramel MJ, Rao A. Dermoscopy: a rapid bedside tool to assess monilethrix. Indian J Dermatol Venereol Leprol. 2016;82:73-74.
  7. Sinclair R. Treatment of monilethrix with oral minoxidil. JAAD Case Rep. 2016;2:212-215.
  8. Rakowska A, Slowinska M, Czuwara J, et al. Dermoscopy as a tool for rapid diagnosis of monilethrix. J Drugs Dermatol. 2007;6:222-224.
  9. Karincaoglu Y, Coskun BK, Seyhan ME, et al. Monilethrix. Am J Clin Dermatol. 2005;6:407-410.
Issue
Cutis - 112(3)
Issue
Cutis - 112(3)
Page Number
E27-E29
Page Number
E27-E29
Publications
Publications
Topics
Article Type
Display Headline
Hypotrichosis and Hair Loss on the Occipital Scalp
Display Headline
Hypotrichosis and Hair Loss on the Occipital Scalp
Sections
Questionnaire Body

A 6-month-old infant girl was referred to the dermatology service with hypotrichosis and hair loss on the occipital region of the scalp of 4 months’ duration (top). The patient was born at full term by cesarean delivery without complications. There were no comorbidities or family history of alopecia. Clinical examination revealed an alopecic plaque in the occipital region with broken hairs and some dystrophic hairs associated with follicular papules and perifollicular hyperkeratosis. A hair pull test was positive for telogen hairs. Trichoscopy revealed black dots and broken hairs resembling Morse code (bottom). Hair microscopy showed regular alternation of constriction zones separated by intervals of normal thickness.

CT112003027_e_Quiz_top_bottom.jpg

Disallow All Ads
Content Gating
No Gating (article Unlocked/Free)
Alternative CME
Disqus Comments
Default
Gate On Date
Wed, 09/27/2023 - 11:45
Un-Gate On Date
Wed, 09/27/2023 - 11:45
Use ProPublica
CFC Schedule Remove Status
Wed, 09/27/2023 - 11:45
Hide sidebar & use full width
render the right sidebar.
Conference Recap Checkbox
Not Conference Recap
Clinical Edge
Display the Slideshow in this Article
Medscape Article
Display survey writer
Reuters content
Disable Inline Native ads
WebMD Article
Article PDF Media
Image
Teambase ID
180025A7.SIG
Disable zoom
Off

Diffuse Pruritic Eruption in an Immunocompromised Patient

Article Type
Changed
Mon, 09/25/2023 - 07:56
Display Headline
Diffuse Pruritic Eruption in an Immunocompromised Patient

The Diagnosis: Scabies Infestation

Direct microscopy revealed the presence of a live scabies mite and numerous eggs (Figure), confirming the diagnosis of a scabies infestation. Scabies, caused by the Sarcoptes scabiei var hominis mite, characteristically presents in adults as pruritic hyperkeratotic plaques of the interdigital web spaces of the hands, flexor surfaces of the wrists and elbows, axillae, male genitalia, and breasts; however, an atypical presentation is common in immunocompromised or immunosuppressed individuals, such as our patient. In children, the palms, soles, and head (ie, face, scalp, neck) are common sites of involvement. Although dermatologists generally are familiar with severe atypical presentations such as Norwegian crusted scabies or bullous scabies, it is important that they are aware of other atypical presentations, such as the diffuse papulonodular variant observed in our patient.1 As such, a low threshold of suspicion for scabies infestations should be employed in immunocompromised patients with new-onset pruritic eruptions.

Nelson_diffuse.jpg
%3Cp%3EA%20live%20scabies%20mite%20(%3Cem%3ESarcoptes%20scabiei%3C%2Fem%3E%20var%20%3Cem%3Ehominis%3C%2Fem%3E)%20obtained%20from%20a%20skin%20scraping%20of%20a%20lesion%20on%20the%20right%20flank%20prepared%20with%20mineral%20oil%20and%20viewed%20under%20direct%20microscopy%20(original%20magnification%20%C3%9720).%3C%2Fp%3E

Direct microscopy is widely accepted as the gold standard for the diagnosis of scabies infestations; it is a fast and low-cost diagnostic tool. However, this technique displays variable sensitivity in clinical practice, requiring experience and a skilled hand.1,2 Other more sensitive diagnostic options for suspected scabies infestations include histopathology, serology, and molecular-based techniques such as DNA isolation and polymerase chain reaction. Although these tests do demonstrate greater sensitivity, they also are more invasive, time intensive, and costly.2 Therefore, they typically are not the first choice for a suspected scabies infestation. Dermoscopy has emerged as another tool to aid in the diagnosis of a suspected scabies infestation, enabling visualization of scaly burrows, eggs, and live mites. Classically, findings resembling a delta wing with contrail are seen on dermoscopic examination. The delta wing represents the brown triangular structure of the pigmented scabies mite head and anterior legs; the contrail is the lighter linear structures streaming behind the scabies mite (similar to visible vapor streams occurring behind flying jets), representing the burrow of the mite.

Although treatment of scabies infestations typically can be accomplished with permethrin cream 5%, the diffuse nature of our patient’s lesions in combination with his immunocompromised state made oral therapy a more appropriate choice. Based on Centers for Disease Control and Prevention recommendations, the patient received 2 doses of oral weight-based ivermectin (200 μg/kg per dose) administered 1 week apart.1,3 The initial dose at day 1 serves to eliminate any scabies mites that are present, while the second dose 1 week later eliminates any residual eggs. Our patient experienced complete resolution of the symptoms following this treatment regimen.

It was important to differentiate our patient’s scabies infestation from other intensely pruritic conditions and morphologic mimics including papular urticaria, lichenoid drug eruptions, tinea corporis, and prurigo nodularis. Papular urticaria is an intensely pruritic hypersensitivity reaction to insect bites that commonly affects the extremities or other exposed areas. Visible puncta may be present.4 Our patient’s lesion distribution involved areas covered by clothing, no puncta were present, and he had no history of a recent arthropod assault, making the diagnosis of papular urticaria less likely.

Lichenoid drug eruptions classically present with symmetric, diffuse, pruritic, violaceous, scaling papules and plaques that present 2 to 3 months after exposure to an offending agent.5 Our patient’s eruption was papulonodular with no violaceous plaques, and he did not report changes to his medications, making a lichenoid drug eruption less likely.

Tinea corporis is another intensely pruritic condition that should be considered, especially in immunocompromised patients. It is caused by dermatophytes and classically presents as erythematous pruritic plaques with an annular, advancing, scaling border.6 Although immunocompromised patients may display extensive involvement, our patient’s lesions were papulonodular with no annular morphology or scale, rendering tinea corporis less likely.

Prurigo nodularis is a chronic condition characterized by pruritic, violaceous, dome-shaped, smooth or crusted nodules secondary to repeated scratching or pressure. Although prurigo nodules can develop as a secondary change due to chronic excoriations in scabies infestations, prurigo nodules usually do not develop in areas such as the midline of the back that are not easily reached by the fingernails,7 which made prurigo nodularis less likely in our patient.

This case describes a unique papulonodular variant of scabies presenting in an immunocompromised cancer patient. Timely recognition and diagnosis of atypical scabies infestations can decrease morbidity and improve the quality of life of these patients.

References
  1. Chandler DJ, Fuller LC. A review of scabies: an infestation more than skin deep. Dermatology. 2019;235:79-90. doi:10.1159/000495290
  2. Siddig EE, Hay R. Laboratory-based diagnosis of scabies: a review of the current status. Trans R Soc Trop Med Hyg. 2022;116:4-9. doi:10.1093/trstmh/trab049
  3. Centers for Disease Control and Prevention. Parasites—scabies. medications. Accessed September 19, 2023. https://www.cdc.gov/parasites/ scabies/health_professionals/meds.html
  4. Örnek S, Zuberbier T, Kocatürk E. Annular urticarial lesions. Clin Dermatol. 2022;40:480-504. doi:10.1016/j.clindermatol .2021.12.010
  5. Cheraghlou S, Levy LL. Fixed drug eruptions, bullous drug eruptions, and lichenoid drug eruptions. Clin Dermatol. 2020;38:679-692. doi:10.1016/j.clindermatol.2020.06.010
  6. Leung AK, Lam JM, Leong KF, et al. Tinea corporis: an updated review. Drugs Context. 2020;9:2020-5-6. doi:10.7573/dic.2020-5-6
  7. Kwon CD, Khanna R, Williams KA, et al. Diagnostic workup and evaluation of patients with prurigo nodularis. Medicines (Basel). 2019;6:97. doi:10.3390/medicines6040097
Article PDF
Author and Disclosure Information

Emelie E. Nelson and Dr. Ryan are from the John P. and Katherine G. McGovern Medical School, University of Texas Health Science Center at Houston. Dr. Nelson is from the University of Texas MD Anderson Cancer Center, Houston.

The authors report no conflict of interest.

Correspondence: Kelly C. Nelson, MD, 1515 Holcombe Blvd, Faculty Center Tower, Floor 11, Box 1452, Houston, TX 77030 (kcnelson1@mdanderson.org).

Issue
Cutis - 112(3)
Publications
Topics
Page Number
E21-E23
Sections
Author and Disclosure Information

Emelie E. Nelson and Dr. Ryan are from the John P. and Katherine G. McGovern Medical School, University of Texas Health Science Center at Houston. Dr. Nelson is from the University of Texas MD Anderson Cancer Center, Houston.

The authors report no conflict of interest.

Correspondence: Kelly C. Nelson, MD, 1515 Holcombe Blvd, Faculty Center Tower, Floor 11, Box 1452, Houston, TX 77030 (kcnelson1@mdanderson.org).

Author and Disclosure Information

Emelie E. Nelson and Dr. Ryan are from the John P. and Katherine G. McGovern Medical School, University of Texas Health Science Center at Houston. Dr. Nelson is from the University of Texas MD Anderson Cancer Center, Houston.

The authors report no conflict of interest.

Correspondence: Kelly C. Nelson, MD, 1515 Holcombe Blvd, Faculty Center Tower, Floor 11, Box 1452, Houston, TX 77030 (kcnelson1@mdanderson.org).

Article PDF
Article PDF
Related Articles

The Diagnosis: Scabies Infestation

Direct microscopy revealed the presence of a live scabies mite and numerous eggs (Figure), confirming the diagnosis of a scabies infestation. Scabies, caused by the Sarcoptes scabiei var hominis mite, characteristically presents in adults as pruritic hyperkeratotic plaques of the interdigital web spaces of the hands, flexor surfaces of the wrists and elbows, axillae, male genitalia, and breasts; however, an atypical presentation is common in immunocompromised or immunosuppressed individuals, such as our patient. In children, the palms, soles, and head (ie, face, scalp, neck) are common sites of involvement. Although dermatologists generally are familiar with severe atypical presentations such as Norwegian crusted scabies or bullous scabies, it is important that they are aware of other atypical presentations, such as the diffuse papulonodular variant observed in our patient.1 As such, a low threshold of suspicion for scabies infestations should be employed in immunocompromised patients with new-onset pruritic eruptions.

Nelson_diffuse.jpg
%3Cp%3EA%20live%20scabies%20mite%20(%3Cem%3ESarcoptes%20scabiei%3C%2Fem%3E%20var%20%3Cem%3Ehominis%3C%2Fem%3E)%20obtained%20from%20a%20skin%20scraping%20of%20a%20lesion%20on%20the%20right%20flank%20prepared%20with%20mineral%20oil%20and%20viewed%20under%20direct%20microscopy%20(original%20magnification%20%C3%9720).%3C%2Fp%3E

Direct microscopy is widely accepted as the gold standard for the diagnosis of scabies infestations; it is a fast and low-cost diagnostic tool. However, this technique displays variable sensitivity in clinical practice, requiring experience and a skilled hand.1,2 Other more sensitive diagnostic options for suspected scabies infestations include histopathology, serology, and molecular-based techniques such as DNA isolation and polymerase chain reaction. Although these tests do demonstrate greater sensitivity, they also are more invasive, time intensive, and costly.2 Therefore, they typically are not the first choice for a suspected scabies infestation. Dermoscopy has emerged as another tool to aid in the diagnosis of a suspected scabies infestation, enabling visualization of scaly burrows, eggs, and live mites. Classically, findings resembling a delta wing with contrail are seen on dermoscopic examination. The delta wing represents the brown triangular structure of the pigmented scabies mite head and anterior legs; the contrail is the lighter linear structures streaming behind the scabies mite (similar to visible vapor streams occurring behind flying jets), representing the burrow of the mite.

Although treatment of scabies infestations typically can be accomplished with permethrin cream 5%, the diffuse nature of our patient’s lesions in combination with his immunocompromised state made oral therapy a more appropriate choice. Based on Centers for Disease Control and Prevention recommendations, the patient received 2 doses of oral weight-based ivermectin (200 μg/kg per dose) administered 1 week apart.1,3 The initial dose at day 1 serves to eliminate any scabies mites that are present, while the second dose 1 week later eliminates any residual eggs. Our patient experienced complete resolution of the symptoms following this treatment regimen.

It was important to differentiate our patient’s scabies infestation from other intensely pruritic conditions and morphologic mimics including papular urticaria, lichenoid drug eruptions, tinea corporis, and prurigo nodularis. Papular urticaria is an intensely pruritic hypersensitivity reaction to insect bites that commonly affects the extremities or other exposed areas. Visible puncta may be present.4 Our patient’s lesion distribution involved areas covered by clothing, no puncta were present, and he had no history of a recent arthropod assault, making the diagnosis of papular urticaria less likely.

Lichenoid drug eruptions classically present with symmetric, diffuse, pruritic, violaceous, scaling papules and plaques that present 2 to 3 months after exposure to an offending agent.5 Our patient’s eruption was papulonodular with no violaceous plaques, and he did not report changes to his medications, making a lichenoid drug eruption less likely.

Tinea corporis is another intensely pruritic condition that should be considered, especially in immunocompromised patients. It is caused by dermatophytes and classically presents as erythematous pruritic plaques with an annular, advancing, scaling border.6 Although immunocompromised patients may display extensive involvement, our patient’s lesions were papulonodular with no annular morphology or scale, rendering tinea corporis less likely.

Prurigo nodularis is a chronic condition characterized by pruritic, violaceous, dome-shaped, smooth or crusted nodules secondary to repeated scratching or pressure. Although prurigo nodules can develop as a secondary change due to chronic excoriations in scabies infestations, prurigo nodules usually do not develop in areas such as the midline of the back that are not easily reached by the fingernails,7 which made prurigo nodularis less likely in our patient.

This case describes a unique papulonodular variant of scabies presenting in an immunocompromised cancer patient. Timely recognition and diagnosis of atypical scabies infestations can decrease morbidity and improve the quality of life of these patients.

The Diagnosis: Scabies Infestation

Direct microscopy revealed the presence of a live scabies mite and numerous eggs (Figure), confirming the diagnosis of a scabies infestation. Scabies, caused by the Sarcoptes scabiei var hominis mite, characteristically presents in adults as pruritic hyperkeratotic plaques of the interdigital web spaces of the hands, flexor surfaces of the wrists and elbows, axillae, male genitalia, and breasts; however, an atypical presentation is common in immunocompromised or immunosuppressed individuals, such as our patient. In children, the palms, soles, and head (ie, face, scalp, neck) are common sites of involvement. Although dermatologists generally are familiar with severe atypical presentations such as Norwegian crusted scabies or bullous scabies, it is important that they are aware of other atypical presentations, such as the diffuse papulonodular variant observed in our patient.1 As such, a low threshold of suspicion for scabies infestations should be employed in immunocompromised patients with new-onset pruritic eruptions.

Nelson_diffuse.jpg
%3Cp%3EA%20live%20scabies%20mite%20(%3Cem%3ESarcoptes%20scabiei%3C%2Fem%3E%20var%20%3Cem%3Ehominis%3C%2Fem%3E)%20obtained%20from%20a%20skin%20scraping%20of%20a%20lesion%20on%20the%20right%20flank%20prepared%20with%20mineral%20oil%20and%20viewed%20under%20direct%20microscopy%20(original%20magnification%20%C3%9720).%3C%2Fp%3E

Direct microscopy is widely accepted as the gold standard for the diagnosis of scabies infestations; it is a fast and low-cost diagnostic tool. However, this technique displays variable sensitivity in clinical practice, requiring experience and a skilled hand.1,2 Other more sensitive diagnostic options for suspected scabies infestations include histopathology, serology, and molecular-based techniques such as DNA isolation and polymerase chain reaction. Although these tests do demonstrate greater sensitivity, they also are more invasive, time intensive, and costly.2 Therefore, they typically are not the first choice for a suspected scabies infestation. Dermoscopy has emerged as another tool to aid in the diagnosis of a suspected scabies infestation, enabling visualization of scaly burrows, eggs, and live mites. Classically, findings resembling a delta wing with contrail are seen on dermoscopic examination. The delta wing represents the brown triangular structure of the pigmented scabies mite head and anterior legs; the contrail is the lighter linear structures streaming behind the scabies mite (similar to visible vapor streams occurring behind flying jets), representing the burrow of the mite.

Although treatment of scabies infestations typically can be accomplished with permethrin cream 5%, the diffuse nature of our patient’s lesions in combination with his immunocompromised state made oral therapy a more appropriate choice. Based on Centers for Disease Control and Prevention recommendations, the patient received 2 doses of oral weight-based ivermectin (200 μg/kg per dose) administered 1 week apart.1,3 The initial dose at day 1 serves to eliminate any scabies mites that are present, while the second dose 1 week later eliminates any residual eggs. Our patient experienced complete resolution of the symptoms following this treatment regimen.

It was important to differentiate our patient’s scabies infestation from other intensely pruritic conditions and morphologic mimics including papular urticaria, lichenoid drug eruptions, tinea corporis, and prurigo nodularis. Papular urticaria is an intensely pruritic hypersensitivity reaction to insect bites that commonly affects the extremities or other exposed areas. Visible puncta may be present.4 Our patient’s lesion distribution involved areas covered by clothing, no puncta were present, and he had no history of a recent arthropod assault, making the diagnosis of papular urticaria less likely.

Lichenoid drug eruptions classically present with symmetric, diffuse, pruritic, violaceous, scaling papules and plaques that present 2 to 3 months after exposure to an offending agent.5 Our patient’s eruption was papulonodular with no violaceous plaques, and he did not report changes to his medications, making a lichenoid drug eruption less likely.

Tinea corporis is another intensely pruritic condition that should be considered, especially in immunocompromised patients. It is caused by dermatophytes and classically presents as erythematous pruritic plaques with an annular, advancing, scaling border.6 Although immunocompromised patients may display extensive involvement, our patient’s lesions were papulonodular with no annular morphology or scale, rendering tinea corporis less likely.

Prurigo nodularis is a chronic condition characterized by pruritic, violaceous, dome-shaped, smooth or crusted nodules secondary to repeated scratching or pressure. Although prurigo nodules can develop as a secondary change due to chronic excoriations in scabies infestations, prurigo nodules usually do not develop in areas such as the midline of the back that are not easily reached by the fingernails,7 which made prurigo nodularis less likely in our patient.

This case describes a unique papulonodular variant of scabies presenting in an immunocompromised cancer patient. Timely recognition and diagnosis of atypical scabies infestations can decrease morbidity and improve the quality of life of these patients.

References
  1. Chandler DJ, Fuller LC. A review of scabies: an infestation more than skin deep. Dermatology. 2019;235:79-90. doi:10.1159/000495290
  2. Siddig EE, Hay R. Laboratory-based diagnosis of scabies: a review of the current status. Trans R Soc Trop Med Hyg. 2022;116:4-9. doi:10.1093/trstmh/trab049
  3. Centers for Disease Control and Prevention. Parasites—scabies. medications. Accessed September 19, 2023. https://www.cdc.gov/parasites/ scabies/health_professionals/meds.html
  4. Örnek S, Zuberbier T, Kocatürk E. Annular urticarial lesions. Clin Dermatol. 2022;40:480-504. doi:10.1016/j.clindermatol .2021.12.010
  5. Cheraghlou S, Levy LL. Fixed drug eruptions, bullous drug eruptions, and lichenoid drug eruptions. Clin Dermatol. 2020;38:679-692. doi:10.1016/j.clindermatol.2020.06.010
  6. Leung AK, Lam JM, Leong KF, et al. Tinea corporis: an updated review. Drugs Context. 2020;9:2020-5-6. doi:10.7573/dic.2020-5-6
  7. Kwon CD, Khanna R, Williams KA, et al. Diagnostic workup and evaluation of patients with prurigo nodularis. Medicines (Basel). 2019;6:97. doi:10.3390/medicines6040097
References
  1. Chandler DJ, Fuller LC. A review of scabies: an infestation more than skin deep. Dermatology. 2019;235:79-90. doi:10.1159/000495290
  2. Siddig EE, Hay R. Laboratory-based diagnosis of scabies: a review of the current status. Trans R Soc Trop Med Hyg. 2022;116:4-9. doi:10.1093/trstmh/trab049
  3. Centers for Disease Control and Prevention. Parasites—scabies. medications. Accessed September 19, 2023. https://www.cdc.gov/parasites/ scabies/health_professionals/meds.html
  4. Örnek S, Zuberbier T, Kocatürk E. Annular urticarial lesions. Clin Dermatol. 2022;40:480-504. doi:10.1016/j.clindermatol .2021.12.010
  5. Cheraghlou S, Levy LL. Fixed drug eruptions, bullous drug eruptions, and lichenoid drug eruptions. Clin Dermatol. 2020;38:679-692. doi:10.1016/j.clindermatol.2020.06.010
  6. Leung AK, Lam JM, Leong KF, et al. Tinea corporis: an updated review. Drugs Context. 2020;9:2020-5-6. doi:10.7573/dic.2020-5-6
  7. Kwon CD, Khanna R, Williams KA, et al. Diagnostic workup and evaluation of patients with prurigo nodularis. Medicines (Basel). 2019;6:97. doi:10.3390/medicines6040097
Issue
Cutis - 112(3)
Issue
Cutis - 112(3)
Page Number
E21-E23
Page Number
E21-E23
Publications
Publications
Topics
Article Type
Display Headline
Diffuse Pruritic Eruption in an Immunocompromised Patient
Display Headline
Diffuse Pruritic Eruption in an Immunocompromised Patient
Sections
Questionnaire Body

A 54-year-old man presented to our dermatology clinic for evaluation of a widespread intensely pruritic rash of 4 weeks’ duration. Calamine lotion and oral hydroxyzine provided minimal relief. He was being treated for a myeloproliferative disorder with immunosuppressive therapy consisting of a combination of cladribine, low-dose cytarabine, and fedratinib. Physical examination revealed multiple excoriated papules and indurated nodules on the extensor and flexor surfaces of the arms and legs (top), chest, midline of the back (bottom), and groin. No lesions were noted on the volar aspect of the patient’s wrists or interdigital spaces, and no central puncta or scales were present. He denied any preceding arthropod bites, trauma, new environmental exposures, or changes to his medications. Scrapings from several representative lesions were obtained for mineral oil preparation and microscopic evaluation.

CT112003021_e_Quiz_top_bottom.jpg

Disallow All Ads
Content Gating
No Gating (article Unlocked/Free)
Alternative CME
Disqus Comments
Default
Gate On Date
Wed, 09/20/2023 - 10:00
Un-Gate On Date
Wed, 09/20/2023 - 10:00
Use ProPublica
CFC Schedule Remove Status
Wed, 09/20/2023 - 10:00
Hide sidebar & use full width
render the right sidebar.
Conference Recap Checkbox
Not Conference Recap
Clinical Edge
Display the Slideshow in this Article
Medscape Article
Display survey writer
Reuters content
Disable Inline Native ads
WebMD Article
Article PDF Media
Image
Teambase ID
180025A5.SIG
Disable zoom
Off

Transient Skin Rippling in an Infant

Article Type
Changed
Wed, 09/13/2023 - 12:29
Display Headline
Transient Skin Rippling in an Infant

The Diagnosis: Infantile Transient Smooth Muscle Contraction of the Skin

A diagnosis of infantile transient smooth muscle contraction of the skin (ITSMC) was made based on our patient’s clinical presentation and eliminating the diagnoses in the differential. No treatment ultimately was indicated, as episodes became less frequent over time.

The term infantile transient smooth muscle contraction of the skin was first proposed in 2013 by Torrelo et al,1 who described 9 newborns with episodic skin rippling occasionally associated with exposure to cold or friction. The authors postulated that ITSMC was the result of a transient contraction of the arrector pili smooth muscle fibers of the skin, secondary to autonomic immaturity, primitive reflexes, or smooth muscle hypersensitivity.1 Since this first description, ITSMC has remained a rarely reported and poorly understood phenomenon with rare identified cases in the literature.2,3 Clinical history and examination of infants with intermittent transient skin rippling help to distinguish ITSMC from other diagnoses without the need for biopsy, which is particularly undesirable in the pediatric population.

Congenital smooth muscle hamartoma is a benign proliferation of mature smooth muscle that also can arise from the arrector pili muscles.4 In contrast to ITSMC, a hamartoma does not clear; rather, it persists and grows proportionally with the child and is associated with overlying hyperpigmentation and hypertrichosis. The transient nature of ITSMC may be worrisome for mastocytoma; however, this condition presents as erythematous, yellow, red, or brown macules, papules, plaques, or nodules with a positive Darier sign.5 Although the differential diagnosis includes the shagreen patch characteristic of tuberous sclerosis, this irregular plaque typically is located on the lower back with overlying peau d’orange skin changes, and our patient lacked other features indicative of this condition.6 Becker nevus also remains a consideration in patients with rippled skin, but this entity typically becomes more notable at puberty and is associated with hyperpigmentation and hypertrichosis and is a type of smooth muscle hamartoma.4

Our case highlighted the unusual presentation of ITSMC, a condition that can easily go unrecognized, leading to unnecessary referrals and concern. Familiarity with this benign diagnosis is essential to inform prognosis and guide management.

References
  1. Torrelo A, Moreno S, Castro C, et al. Infantile transient smooth muscle contraction of the skin. J Am Acad Dermatol. 2013;69:498-500. doi:10.1016/j.jaad.2013.04.029
  2. Theodosiou G, Belfrage E, Berggård K, et al. Infantile transient smooth muscle contraction of the skin: a case report and literature review. Eur J Dermatol. 2021;31:260-261. doi:10.1684/ejd.2021.3996
  3. Topham C, Deacon DC, Bowen A, et al. More than goosebumps: a case of marked skin dimpling in an infant. Pediatr Dermatol. 2019;36:E71-E72. doi:10.1111/pde.13791
  4. Raboudi A, Litaiem N. Congenital smooth muscle hamartoma. StatPearls. StatPearls Publishing; 2022.
  5. Leung AKC, Lam JM, Leong KF. Childhood solitary cutaneous mastocytoma: clinical manifestations, diagnosis, evaluation, and management. Curr Pediatr Rev. 2019;15:42-46. doi:10.2174/1573396315666 181120163952
  6. Bongiorno MA, Nathan N, Oyerinde O, et al. Clinical characteristics of connective tissue nevi in tuberous sclerosis complex with special emphasis on shagreen patches. JAMA Dermatol. 2017;153:660-665. doi:10.1001/jamadermatol.2017.0298
Article PDF
Author and Disclosure Information

From the Department of Dermatology, Boston Children’s Hospital, Massachusetts, and the Department of Dermatology, Massachusetts General Hospital, Boston. Mia A. Mologousis also is from Tufts University School of Medicine, Boston. Dr. Hawryluk also is from Harvard Medical School, Boston.

The authors report no conflict of interest.

Correspondence: Elena B. Hawryluk, MD, PhD, Massachusetts General Hospital, 50 Staniford St, Ste 200, Boston MA 02114 (ehawryluk@mgh.harvard.edu).

Issue
Cutis - 112(3)
Publications
Topics
Page Number
E11-E12
Sections
Author and Disclosure Information

From the Department of Dermatology, Boston Children’s Hospital, Massachusetts, and the Department of Dermatology, Massachusetts General Hospital, Boston. Mia A. Mologousis also is from Tufts University School of Medicine, Boston. Dr. Hawryluk also is from Harvard Medical School, Boston.

The authors report no conflict of interest.

Correspondence: Elena B. Hawryluk, MD, PhD, Massachusetts General Hospital, 50 Staniford St, Ste 200, Boston MA 02114 (ehawryluk@mgh.harvard.edu).

Author and Disclosure Information

From the Department of Dermatology, Boston Children’s Hospital, Massachusetts, and the Department of Dermatology, Massachusetts General Hospital, Boston. Mia A. Mologousis also is from Tufts University School of Medicine, Boston. Dr. Hawryluk also is from Harvard Medical School, Boston.

The authors report no conflict of interest.

Correspondence: Elena B. Hawryluk, MD, PhD, Massachusetts General Hospital, 50 Staniford St, Ste 200, Boston MA 02114 (ehawryluk@mgh.harvard.edu).

Article PDF
Article PDF
Related Articles

The Diagnosis: Infantile Transient Smooth Muscle Contraction of the Skin

A diagnosis of infantile transient smooth muscle contraction of the skin (ITSMC) was made based on our patient’s clinical presentation and eliminating the diagnoses in the differential. No treatment ultimately was indicated, as episodes became less frequent over time.

The term infantile transient smooth muscle contraction of the skin was first proposed in 2013 by Torrelo et al,1 who described 9 newborns with episodic skin rippling occasionally associated with exposure to cold or friction. The authors postulated that ITSMC was the result of a transient contraction of the arrector pili smooth muscle fibers of the skin, secondary to autonomic immaturity, primitive reflexes, or smooth muscle hypersensitivity.1 Since this first description, ITSMC has remained a rarely reported and poorly understood phenomenon with rare identified cases in the literature.2,3 Clinical history and examination of infants with intermittent transient skin rippling help to distinguish ITSMC from other diagnoses without the need for biopsy, which is particularly undesirable in the pediatric population.

Congenital smooth muscle hamartoma is a benign proliferation of mature smooth muscle that also can arise from the arrector pili muscles.4 In contrast to ITSMC, a hamartoma does not clear; rather, it persists and grows proportionally with the child and is associated with overlying hyperpigmentation and hypertrichosis. The transient nature of ITSMC may be worrisome for mastocytoma; however, this condition presents as erythematous, yellow, red, or brown macules, papules, plaques, or nodules with a positive Darier sign.5 Although the differential diagnosis includes the shagreen patch characteristic of tuberous sclerosis, this irregular plaque typically is located on the lower back with overlying peau d’orange skin changes, and our patient lacked other features indicative of this condition.6 Becker nevus also remains a consideration in patients with rippled skin, but this entity typically becomes more notable at puberty and is associated with hyperpigmentation and hypertrichosis and is a type of smooth muscle hamartoma.4

Our case highlighted the unusual presentation of ITSMC, a condition that can easily go unrecognized, leading to unnecessary referrals and concern. Familiarity with this benign diagnosis is essential to inform prognosis and guide management.

The Diagnosis: Infantile Transient Smooth Muscle Contraction of the Skin

A diagnosis of infantile transient smooth muscle contraction of the skin (ITSMC) was made based on our patient’s clinical presentation and eliminating the diagnoses in the differential. No treatment ultimately was indicated, as episodes became less frequent over time.

The term infantile transient smooth muscle contraction of the skin was first proposed in 2013 by Torrelo et al,1 who described 9 newborns with episodic skin rippling occasionally associated with exposure to cold or friction. The authors postulated that ITSMC was the result of a transient contraction of the arrector pili smooth muscle fibers of the skin, secondary to autonomic immaturity, primitive reflexes, or smooth muscle hypersensitivity.1 Since this first description, ITSMC has remained a rarely reported and poorly understood phenomenon with rare identified cases in the literature.2,3 Clinical history and examination of infants with intermittent transient skin rippling help to distinguish ITSMC from other diagnoses without the need for biopsy, which is particularly undesirable in the pediatric population.

Congenital smooth muscle hamartoma is a benign proliferation of mature smooth muscle that also can arise from the arrector pili muscles.4 In contrast to ITSMC, a hamartoma does not clear; rather, it persists and grows proportionally with the child and is associated with overlying hyperpigmentation and hypertrichosis. The transient nature of ITSMC may be worrisome for mastocytoma; however, this condition presents as erythematous, yellow, red, or brown macules, papules, plaques, or nodules with a positive Darier sign.5 Although the differential diagnosis includes the shagreen patch characteristic of tuberous sclerosis, this irregular plaque typically is located on the lower back with overlying peau d’orange skin changes, and our patient lacked other features indicative of this condition.6 Becker nevus also remains a consideration in patients with rippled skin, but this entity typically becomes more notable at puberty and is associated with hyperpigmentation and hypertrichosis and is a type of smooth muscle hamartoma.4

Our case highlighted the unusual presentation of ITSMC, a condition that can easily go unrecognized, leading to unnecessary referrals and concern. Familiarity with this benign diagnosis is essential to inform prognosis and guide management.

References
  1. Torrelo A, Moreno S, Castro C, et al. Infantile transient smooth muscle contraction of the skin. J Am Acad Dermatol. 2013;69:498-500. doi:10.1016/j.jaad.2013.04.029
  2. Theodosiou G, Belfrage E, Berggård K, et al. Infantile transient smooth muscle contraction of the skin: a case report and literature review. Eur J Dermatol. 2021;31:260-261. doi:10.1684/ejd.2021.3996
  3. Topham C, Deacon DC, Bowen A, et al. More than goosebumps: a case of marked skin dimpling in an infant. Pediatr Dermatol. 2019;36:E71-E72. doi:10.1111/pde.13791
  4. Raboudi A, Litaiem N. Congenital smooth muscle hamartoma. StatPearls. StatPearls Publishing; 2022.
  5. Leung AKC, Lam JM, Leong KF. Childhood solitary cutaneous mastocytoma: clinical manifestations, diagnosis, evaluation, and management. Curr Pediatr Rev. 2019;15:42-46. doi:10.2174/1573396315666 181120163952
  6. Bongiorno MA, Nathan N, Oyerinde O, et al. Clinical characteristics of connective tissue nevi in tuberous sclerosis complex with special emphasis on shagreen patches. JAMA Dermatol. 2017;153:660-665. doi:10.1001/jamadermatol.2017.0298
References
  1. Torrelo A, Moreno S, Castro C, et al. Infantile transient smooth muscle contraction of the skin. J Am Acad Dermatol. 2013;69:498-500. doi:10.1016/j.jaad.2013.04.029
  2. Theodosiou G, Belfrage E, Berggård K, et al. Infantile transient smooth muscle contraction of the skin: a case report and literature review. Eur J Dermatol. 2021;31:260-261. doi:10.1684/ejd.2021.3996
  3. Topham C, Deacon DC, Bowen A, et al. More than goosebumps: a case of marked skin dimpling in an infant. Pediatr Dermatol. 2019;36:E71-E72. doi:10.1111/pde.13791
  4. Raboudi A, Litaiem N. Congenital smooth muscle hamartoma. StatPearls. StatPearls Publishing; 2022.
  5. Leung AKC, Lam JM, Leong KF. Childhood solitary cutaneous mastocytoma: clinical manifestations, diagnosis, evaluation, and management. Curr Pediatr Rev. 2019;15:42-46. doi:10.2174/1573396315666 181120163952
  6. Bongiorno MA, Nathan N, Oyerinde O, et al. Clinical characteristics of connective tissue nevi in tuberous sclerosis complex with special emphasis on shagreen patches. JAMA Dermatol. 2017;153:660-665. doi:10.1001/jamadermatol.2017.0298
Issue
Cutis - 112(3)
Issue
Cutis - 112(3)
Page Number
E11-E12
Page Number
E11-E12
Publications
Publications
Topics
Article Type
Display Headline
Transient Skin Rippling in an Infant
Display Headline
Transient Skin Rippling in an Infant
Sections
Questionnaire Body

A healthy, full-term, 5-month-old infant boy presented to dermatology for evaluation of an intermittent, asymptomatic, rippled skin texture of the left thigh that resolved completely between flares. The parents noted fewer than 10 intermittent flares prior to the initial presentation at 5 months. Physical examination of the patient’s skin revealed no epidermal abnormalities, dermatographism, or subcutaneous nodules, and there was no positive Darier sign. A subsequent flare at 9 months of age occurred concurrently with fevers up to 39.4 °C (103 °F), and a corresponding photograph (quiz image) provided by the parents due to the intermittent and transient nature of the condition demonstrated an ill-defined, raised, rippled plaque on the left lateral thigh.

Mologousis_Quiz.jpg

Disallow All Ads
Content Gating
No Gating (article Unlocked/Free)
Alternative CME
Disqus Comments
Default
Gate On Date
Wed, 09/13/2023 - 06:45
Un-Gate On Date
Wed, 09/13/2023 - 06:45
Use ProPublica
CFC Schedule Remove Status
Wed, 09/13/2023 - 06:45
Hide sidebar & use full width
render the right sidebar.
Conference Recap Checkbox
Not Conference Recap
Clinical Edge
Display the Slideshow in this Article
Medscape Article
Display survey writer
Reuters content
Disable Inline Native ads
WebMD Article
Article PDF Media

Disseminated Papules and Nodules on the Skin and Oral Mucosa in an Infant

Article Type
Changed
Tue, 09/05/2023 - 10:11
Display Headline
Disseminated Papules and Nodules on the Skin and Oral Mucosa in an Infant

The Diagnosis: Congenital Cutaneous Langerhans Cell Histiocytosis

Although the infectious workup was positive for herpes simplex virus type 1 and cytomegalovirus antibodies, serologies for the rest of the TORCH (toxoplasmosis, other agents [syphilis, hepatitis B virus], rubella, cytomegalovirus) group of infections, as well as other bacterial, fungal, and viral infections, were negative. A skin biopsy from the right fifth toe showed a dense infiltrate of CD1a+ histiocytic cells with folded or kidney-shaped nuclei mixed with eosinophils, which was consistent with Langerhans cell histiocytosis (LCH) (Figure 1). Skin lesions were treated with hydrocortisone cream 2.5% and progressively faded over a few weeks.

Ong_1.jpg
%3Cp%3E%3Cstrong%3EFIGURE%201.%3C%2Fstrong%3E%20A%20dense%20infiltrate%20of%20histiocytic%20cells%20with%20folded%20or%20kidney-shaped%20nuclei%20mixed%20with%20eosinophils%20(H%26amp%3BE%2C%20original%20magnification%20%C3%9740).%3C%2Fp%3E

Langerhans cell histiocytosis is a rare disorder with a variable clinical presentation depending on the sites affected and the extent of involvement. It can involve multiple organ systems, most commonly the skeletal system and the skin. Organ involvement is characterized by histiocyte infiltration. Acute disseminated multisystem disease most commonly is seen in children younger than 3 years.1

Congenital cutaneous LCH presents with variable skin lesions ranging from papules to vesicles, pustules, and ulcers, with onset at birth or in the neonatal period. Various morphologic traits of skin lesions have been described; the most common presentation is multiple red to yellow-brown, crusted papules with accompanying hemorrhage or erosion.1 Other cases have described an eczematous, seborrheic, diffuse eruption or erosive intertrigo. One case of a child with a solitary necrotic nodule on the scalp has been reported.2

Our patient presented with disseminated, nonblanching, purple to dark red papules and nodules of the skin and oral mucosa, as well as nail dystrophy (Figure 2). However, LCH in a neonate can mimic other causes of congenital papulonodular eruptions. Red-brown papules and nodules with or without crusting in a newborn can be mistaken for erythema toxicum neonatorum, transient neonatal pustular melanosis, congenital leukemia cutis, neonatal erythropoiesis, disseminated neonatal hemangiomatosis, infantile acropustulosis, or congenital TORCH infections such as rubella or syphilis. When LCH presents as vesicles or eroded papules or nodules in a newborn, the differential diagnosis includes incontinentia pigmenti and hereditary epidermolysis bullosa.

CT112003131_Fig2_AB.jpg
%3Cp%3E%3Cstrong%3EFIGURE%202.%3C%2Fstrong%3E%20The%20clinical%20presentation%20of%20Langerhans%20cell%20histiocytosis%20in%20an%20infant.%20A%2C%20Disseminated%2C%20nonblanching%2C%20purple%20to%20dark%20red%20papules%20and%20nodules%20were%20present%20on%20the%20oral%20mucosa.%20B%2C%20Nail%20dystrophy%20also%20was%20present.%3C%2Fp%3E

Langerhans cell histiocytosis may even present with a classic blueberry muffin rash that can lead clinicians to consider cutaneous metastasis from various hematologic malignancies or the more common TORCH infections. Several diagnostic tests can be performed to clarify the diagnosis, including bacterial and viral cultures and stains, serology, immunohistochemistry, flow cytometry, bone marrow aspiration, or skin biopsy.3 Langerhans cell histiocytosis is diagnosed with a combination of histology, immunohistochemistry, and clinical presentation; however, a skin biopsy is crucial. Tissue should be taken from the most easily accessible yet representative lesion. The characteristic appearance of LCH lesions is described as a dense infiltrate of histiocytic cells mixed with numerous eosinophils in the dermis.1 Histiocytes usually have folded nuclei and eosinophilic cytoplasm or kidney-shaped nuclei with prominent nucleoli. Positive CD1a and/or CD207 (Langerin) staining of the cells is required for definitive diagnosis.4 After diagnosis, it is important to obtain baseline laboratory and radiographic studies to determine the extent of systemic involvement.

Treatment of congenital LCH is tailored to the extent of organ involvement. The dermatologic manifestations resolve without medications in many cases. However, true self-resolving LCH can only be diagnosed retrospectively after a full evaluation for other sites of disease. Disseminated disease can be life-threatening and requires more active management. In cases of skin-limited disease, therapies include topical steroids, nitrogen mustard, or imiquimod; surgical resection of isolated lesions; phototherapy; or systemic therapies such as methotrexate, 6-mercaptopurine, vinblastine/vincristine, cladribine, and/or cytarabine. Symptomatic patients initially are treated with methotrexate and 6-mercaptopurine.5 Asymptomatic infants with skin-limited involvement can be managed with topical treatments.

Our patient had skin-limited disease. Abdominal ultrasonography, skeletal survey, and magnetic resonance imaging of the brain revealed no abnormalities. The patient’s family was advised to monitor him for reoccurrence of the skin lesions and to continue close follow-up with hematology and dermatology. Although congenital LCH often is self-resolving, extensive skin involvement increases the risk for internal organ involvement for several years.6 These patients require long-term follow-up for potential musculoskeletal, ophthalmologic, endocrine, hepatic, and/or pulmonary disease.

References
  1. Pan Y, Zeng X, Ge J, et al. Congenital self-healing Langerhans cell histiocytosis: clinical and pathological characteristics. Int J Clin Exp Pathol. 2019;12:2275-2278.
  2. Morren MA, Vanden Broecke K, Vangeebergen L, et al. Diverse cutaneous presentations of Langerhans cell histiocytosis in children: a retrospective cohort study. Pediatr Blood Cancer. 2016;63:486-492. doi:10.1002/pbc.25834
  3. Krooks J, Minkov M, Weatherall AG. Langerhans cell histiocytosis in children: diagnosis, differential diagnosis, treatment, sequelae, and standardized follow-up. J Am Acad Dermatol. 2018;78:1047-1056. doi:10.1016/j.jaad.2017.05.060
  4. Haupt R, Minkov M, Astigarraga I, et al. Langerhans cell histiocytosis (LCH): guidelines for diagnosis, clinical work-up, and treatment for patients till the age of 18 years. Pediatr Blood Cancer. 2013;60:175-184. doi:10.1002/pbc.24367
  5. Allen CE, Ladisch S, McClain KL. How I treat Langerhans cell histiocytosis. Blood. 2015;126:26-35. doi:10.1182/blood-2014-12-569301
  6. Jezierska M, Stefanowicz J, Romanowicz G, et al. Langerhans cell histiocytosis in children—a disease with many faces. recent advances in pathogenesis, diagnostic examinations and treatment. Postepy Dermatol Alergol. 2018;35:6-17. doi:10.5114/pdia.2017.67095
Article PDF
Author and Disclosure Information

From the Department of Dermatology, Saint Louis University School of Medicine, Missouri. Dr. Siegfried also is from the Department of Pediatrics.

The authors report no conflict of interest.

Correspondence: Ramona Behshad, MD, Department of Dermatology, Center for Specialized Medicine, 1225 S Grand Blvd, St. Louis, MO 63104 (ramona.behshad@health.slu.edu).

Issue
Cutis - 112(3)
Publications
Topics
Page Number
131,135-136
Sections
Author and Disclosure Information

From the Department of Dermatology, Saint Louis University School of Medicine, Missouri. Dr. Siegfried also is from the Department of Pediatrics.

The authors report no conflict of interest.

Correspondence: Ramona Behshad, MD, Department of Dermatology, Center for Specialized Medicine, 1225 S Grand Blvd, St. Louis, MO 63104 (ramona.behshad@health.slu.edu).

Author and Disclosure Information

From the Department of Dermatology, Saint Louis University School of Medicine, Missouri. Dr. Siegfried also is from the Department of Pediatrics.

The authors report no conflict of interest.

Correspondence: Ramona Behshad, MD, Department of Dermatology, Center for Specialized Medicine, 1225 S Grand Blvd, St. Louis, MO 63104 (ramona.behshad@health.slu.edu).

Article PDF
Article PDF
Related Articles

The Diagnosis: Congenital Cutaneous Langerhans Cell Histiocytosis

Although the infectious workup was positive for herpes simplex virus type 1 and cytomegalovirus antibodies, serologies for the rest of the TORCH (toxoplasmosis, other agents [syphilis, hepatitis B virus], rubella, cytomegalovirus) group of infections, as well as other bacterial, fungal, and viral infections, were negative. A skin biopsy from the right fifth toe showed a dense infiltrate of CD1a+ histiocytic cells with folded or kidney-shaped nuclei mixed with eosinophils, which was consistent with Langerhans cell histiocytosis (LCH) (Figure 1). Skin lesions were treated with hydrocortisone cream 2.5% and progressively faded over a few weeks.

Ong_1.jpg
%3Cp%3E%3Cstrong%3EFIGURE%201.%3C%2Fstrong%3E%20A%20dense%20infiltrate%20of%20histiocytic%20cells%20with%20folded%20or%20kidney-shaped%20nuclei%20mixed%20with%20eosinophils%20(H%26amp%3BE%2C%20original%20magnification%20%C3%9740).%3C%2Fp%3E

Langerhans cell histiocytosis is a rare disorder with a variable clinical presentation depending on the sites affected and the extent of involvement. It can involve multiple organ systems, most commonly the skeletal system and the skin. Organ involvement is characterized by histiocyte infiltration. Acute disseminated multisystem disease most commonly is seen in children younger than 3 years.1

Congenital cutaneous LCH presents with variable skin lesions ranging from papules to vesicles, pustules, and ulcers, with onset at birth or in the neonatal period. Various morphologic traits of skin lesions have been described; the most common presentation is multiple red to yellow-brown, crusted papules with accompanying hemorrhage or erosion.1 Other cases have described an eczematous, seborrheic, diffuse eruption or erosive intertrigo. One case of a child with a solitary necrotic nodule on the scalp has been reported.2

Our patient presented with disseminated, nonblanching, purple to dark red papules and nodules of the skin and oral mucosa, as well as nail dystrophy (Figure 2). However, LCH in a neonate can mimic other causes of congenital papulonodular eruptions. Red-brown papules and nodules with or without crusting in a newborn can be mistaken for erythema toxicum neonatorum, transient neonatal pustular melanosis, congenital leukemia cutis, neonatal erythropoiesis, disseminated neonatal hemangiomatosis, infantile acropustulosis, or congenital TORCH infections such as rubella or syphilis. When LCH presents as vesicles or eroded papules or nodules in a newborn, the differential diagnosis includes incontinentia pigmenti and hereditary epidermolysis bullosa.

CT112003131_Fig2_AB.jpg
%3Cp%3E%3Cstrong%3EFIGURE%202.%3C%2Fstrong%3E%20The%20clinical%20presentation%20of%20Langerhans%20cell%20histiocytosis%20in%20an%20infant.%20A%2C%20Disseminated%2C%20nonblanching%2C%20purple%20to%20dark%20red%20papules%20and%20nodules%20were%20present%20on%20the%20oral%20mucosa.%20B%2C%20Nail%20dystrophy%20also%20was%20present.%3C%2Fp%3E

Langerhans cell histiocytosis may even present with a classic blueberry muffin rash that can lead clinicians to consider cutaneous metastasis from various hematologic malignancies or the more common TORCH infections. Several diagnostic tests can be performed to clarify the diagnosis, including bacterial and viral cultures and stains, serology, immunohistochemistry, flow cytometry, bone marrow aspiration, or skin biopsy.3 Langerhans cell histiocytosis is diagnosed with a combination of histology, immunohistochemistry, and clinical presentation; however, a skin biopsy is crucial. Tissue should be taken from the most easily accessible yet representative lesion. The characteristic appearance of LCH lesions is described as a dense infiltrate of histiocytic cells mixed with numerous eosinophils in the dermis.1 Histiocytes usually have folded nuclei and eosinophilic cytoplasm or kidney-shaped nuclei with prominent nucleoli. Positive CD1a and/or CD207 (Langerin) staining of the cells is required for definitive diagnosis.4 After diagnosis, it is important to obtain baseline laboratory and radiographic studies to determine the extent of systemic involvement.

Treatment of congenital LCH is tailored to the extent of organ involvement. The dermatologic manifestations resolve without medications in many cases. However, true self-resolving LCH can only be diagnosed retrospectively after a full evaluation for other sites of disease. Disseminated disease can be life-threatening and requires more active management. In cases of skin-limited disease, therapies include topical steroids, nitrogen mustard, or imiquimod; surgical resection of isolated lesions; phototherapy; or systemic therapies such as methotrexate, 6-mercaptopurine, vinblastine/vincristine, cladribine, and/or cytarabine. Symptomatic patients initially are treated with methotrexate and 6-mercaptopurine.5 Asymptomatic infants with skin-limited involvement can be managed with topical treatments.

Our patient had skin-limited disease. Abdominal ultrasonography, skeletal survey, and magnetic resonance imaging of the brain revealed no abnormalities. The patient’s family was advised to monitor him for reoccurrence of the skin lesions and to continue close follow-up with hematology and dermatology. Although congenital LCH often is self-resolving, extensive skin involvement increases the risk for internal organ involvement for several years.6 These patients require long-term follow-up for potential musculoskeletal, ophthalmologic, endocrine, hepatic, and/or pulmonary disease.

The Diagnosis: Congenital Cutaneous Langerhans Cell Histiocytosis

Although the infectious workup was positive for herpes simplex virus type 1 and cytomegalovirus antibodies, serologies for the rest of the TORCH (toxoplasmosis, other agents [syphilis, hepatitis B virus], rubella, cytomegalovirus) group of infections, as well as other bacterial, fungal, and viral infections, were negative. A skin biopsy from the right fifth toe showed a dense infiltrate of CD1a+ histiocytic cells with folded or kidney-shaped nuclei mixed with eosinophils, which was consistent with Langerhans cell histiocytosis (LCH) (Figure 1). Skin lesions were treated with hydrocortisone cream 2.5% and progressively faded over a few weeks.

Ong_1.jpg
%3Cp%3E%3Cstrong%3EFIGURE%201.%3C%2Fstrong%3E%20A%20dense%20infiltrate%20of%20histiocytic%20cells%20with%20folded%20or%20kidney-shaped%20nuclei%20mixed%20with%20eosinophils%20(H%26amp%3BE%2C%20original%20magnification%20%C3%9740).%3C%2Fp%3E

Langerhans cell histiocytosis is a rare disorder with a variable clinical presentation depending on the sites affected and the extent of involvement. It can involve multiple organ systems, most commonly the skeletal system and the skin. Organ involvement is characterized by histiocyte infiltration. Acute disseminated multisystem disease most commonly is seen in children younger than 3 years.1

Congenital cutaneous LCH presents with variable skin lesions ranging from papules to vesicles, pustules, and ulcers, with onset at birth or in the neonatal period. Various morphologic traits of skin lesions have been described; the most common presentation is multiple red to yellow-brown, crusted papules with accompanying hemorrhage or erosion.1 Other cases have described an eczematous, seborrheic, diffuse eruption or erosive intertrigo. One case of a child with a solitary necrotic nodule on the scalp has been reported.2

Our patient presented with disseminated, nonblanching, purple to dark red papules and nodules of the skin and oral mucosa, as well as nail dystrophy (Figure 2). However, LCH in a neonate can mimic other causes of congenital papulonodular eruptions. Red-brown papules and nodules with or without crusting in a newborn can be mistaken for erythema toxicum neonatorum, transient neonatal pustular melanosis, congenital leukemia cutis, neonatal erythropoiesis, disseminated neonatal hemangiomatosis, infantile acropustulosis, or congenital TORCH infections such as rubella or syphilis. When LCH presents as vesicles or eroded papules or nodules in a newborn, the differential diagnosis includes incontinentia pigmenti and hereditary epidermolysis bullosa.

CT112003131_Fig2_AB.jpg
%3Cp%3E%3Cstrong%3EFIGURE%202.%3C%2Fstrong%3E%20The%20clinical%20presentation%20of%20Langerhans%20cell%20histiocytosis%20in%20an%20infant.%20A%2C%20Disseminated%2C%20nonblanching%2C%20purple%20to%20dark%20red%20papules%20and%20nodules%20were%20present%20on%20the%20oral%20mucosa.%20B%2C%20Nail%20dystrophy%20also%20was%20present.%3C%2Fp%3E

Langerhans cell histiocytosis may even present with a classic blueberry muffin rash that can lead clinicians to consider cutaneous metastasis from various hematologic malignancies or the more common TORCH infections. Several diagnostic tests can be performed to clarify the diagnosis, including bacterial and viral cultures and stains, serology, immunohistochemistry, flow cytometry, bone marrow aspiration, or skin biopsy.3 Langerhans cell histiocytosis is diagnosed with a combination of histology, immunohistochemistry, and clinical presentation; however, a skin biopsy is crucial. Tissue should be taken from the most easily accessible yet representative lesion. The characteristic appearance of LCH lesions is described as a dense infiltrate of histiocytic cells mixed with numerous eosinophils in the dermis.1 Histiocytes usually have folded nuclei and eosinophilic cytoplasm or kidney-shaped nuclei with prominent nucleoli. Positive CD1a and/or CD207 (Langerin) staining of the cells is required for definitive diagnosis.4 After diagnosis, it is important to obtain baseline laboratory and radiographic studies to determine the extent of systemic involvement.

Treatment of congenital LCH is tailored to the extent of organ involvement. The dermatologic manifestations resolve without medications in many cases. However, true self-resolving LCH can only be diagnosed retrospectively after a full evaluation for other sites of disease. Disseminated disease can be life-threatening and requires more active management. In cases of skin-limited disease, therapies include topical steroids, nitrogen mustard, or imiquimod; surgical resection of isolated lesions; phototherapy; or systemic therapies such as methotrexate, 6-mercaptopurine, vinblastine/vincristine, cladribine, and/or cytarabine. Symptomatic patients initially are treated with methotrexate and 6-mercaptopurine.5 Asymptomatic infants with skin-limited involvement can be managed with topical treatments.

Our patient had skin-limited disease. Abdominal ultrasonography, skeletal survey, and magnetic resonance imaging of the brain revealed no abnormalities. The patient’s family was advised to monitor him for reoccurrence of the skin lesions and to continue close follow-up with hematology and dermatology. Although congenital LCH often is self-resolving, extensive skin involvement increases the risk for internal organ involvement for several years.6 These patients require long-term follow-up for potential musculoskeletal, ophthalmologic, endocrine, hepatic, and/or pulmonary disease.

References
  1. Pan Y, Zeng X, Ge J, et al. Congenital self-healing Langerhans cell histiocytosis: clinical and pathological characteristics. Int J Clin Exp Pathol. 2019;12:2275-2278.
  2. Morren MA, Vanden Broecke K, Vangeebergen L, et al. Diverse cutaneous presentations of Langerhans cell histiocytosis in children: a retrospective cohort study. Pediatr Blood Cancer. 2016;63:486-492. doi:10.1002/pbc.25834
  3. Krooks J, Minkov M, Weatherall AG. Langerhans cell histiocytosis in children: diagnosis, differential diagnosis, treatment, sequelae, and standardized follow-up. J Am Acad Dermatol. 2018;78:1047-1056. doi:10.1016/j.jaad.2017.05.060
  4. Haupt R, Minkov M, Astigarraga I, et al. Langerhans cell histiocytosis (LCH): guidelines for diagnosis, clinical work-up, and treatment for patients till the age of 18 years. Pediatr Blood Cancer. 2013;60:175-184. doi:10.1002/pbc.24367
  5. Allen CE, Ladisch S, McClain KL. How I treat Langerhans cell histiocytosis. Blood. 2015;126:26-35. doi:10.1182/blood-2014-12-569301
  6. Jezierska M, Stefanowicz J, Romanowicz G, et al. Langerhans cell histiocytosis in children—a disease with many faces. recent advances in pathogenesis, diagnostic examinations and treatment. Postepy Dermatol Alergol. 2018;35:6-17. doi:10.5114/pdia.2017.67095
References
  1. Pan Y, Zeng X, Ge J, et al. Congenital self-healing Langerhans cell histiocytosis: clinical and pathological characteristics. Int J Clin Exp Pathol. 2019;12:2275-2278.
  2. Morren MA, Vanden Broecke K, Vangeebergen L, et al. Diverse cutaneous presentations of Langerhans cell histiocytosis in children: a retrospective cohort study. Pediatr Blood Cancer. 2016;63:486-492. doi:10.1002/pbc.25834
  3. Krooks J, Minkov M, Weatherall AG. Langerhans cell histiocytosis in children: diagnosis, differential diagnosis, treatment, sequelae, and standardized follow-up. J Am Acad Dermatol. 2018;78:1047-1056. doi:10.1016/j.jaad.2017.05.060
  4. Haupt R, Minkov M, Astigarraga I, et al. Langerhans cell histiocytosis (LCH): guidelines for diagnosis, clinical work-up, and treatment for patients till the age of 18 years. Pediatr Blood Cancer. 2013;60:175-184. doi:10.1002/pbc.24367
  5. Allen CE, Ladisch S, McClain KL. How I treat Langerhans cell histiocytosis. Blood. 2015;126:26-35. doi:10.1182/blood-2014-12-569301
  6. Jezierska M, Stefanowicz J, Romanowicz G, et al. Langerhans cell histiocytosis in children—a disease with many faces. recent advances in pathogenesis, diagnostic examinations and treatment. Postepy Dermatol Alergol. 2018;35:6-17. doi:10.5114/pdia.2017.67095
Issue
Cutis - 112(3)
Issue
Cutis - 112(3)
Page Number
131,135-136
Page Number
131,135-136
Publications
Publications
Topics
Article Type
Display Headline
Disseminated Papules and Nodules on the Skin and Oral Mucosa in an Infant
Display Headline
Disseminated Papules and Nodules on the Skin and Oral Mucosa in an Infant
Sections
Questionnaire Body

A 38-week-old infant boy presented at birth with disseminated, nonblanching, purple to dark red papules and nodules on the skin and oral mucosa. He was born spontaneously after an uncomplicated pregnancy. The mother experienced an episode of oral herpes simplex virus during pregnancy. The infant was otherwise healthy. Laboratory tests including a complete blood cell count and routine serum biochemical analyses were within reference range; however, an infectious workup was positive for herpes simplex virus type 1 and cytomegalovirus antibodies. Ophthalmologic and auditory screenings were normal.

Ong_Quiz.jpg

Disallow All Ads
Content Gating
No Gating (article Unlocked/Free)
Alternative CME
Disqus Comments
Default
Gate On Date
Tue, 09/05/2023 - 07:30
Un-Gate On Date
Tue, 09/05/2023 - 07:30
Use ProPublica
CFC Schedule Remove Status
Tue, 09/05/2023 - 07:30
Hide sidebar & use full width
render the right sidebar.
Conference Recap Checkbox
Not Conference Recap
Clinical Edge
Display the Slideshow in this Article
Medscape Article
Display survey writer
Reuters content
Disable Inline Native ads
WebMD Article
Article PDF Media
Image
Teambase ID
1800256C.SIG
Disable zoom
Off